рдПрдбреАрдПрд╕ рд╡реИрдХреНрд╕реАрди рдХреЗ рдмрд╛рдж рдкреНрд░рддрд┐рд░рдХреНрд╖рд╛ред рд╡реИрдХреНрд╕реАрди рдХреЗ "рдЖрд╡рд┐рд╖реНрдХрд╛рд░" рд╕реЗ рдкрд╣рд▓реЗ рдФрд░ рдмрд╛рдж рдореЗрдВред рд░реЛрдЧ рдкреНрд░рддрд┐рд░реЛрдзрдХ рдХреНрд╖рдорддрд╛ рдмрдврд╝рд╛рдиреЗ рдХрд╛ рд╕рдмрд╕реЗ рдЕрдЪреНрдЫрд╛ рддрд░реАрдХрд╛ рд╣реИ рдмреАрдорд╛рд░ рд╣реЛрдирд╛? рдПрдХ рдЦрддрд░рдирд╛рдХ рднреНрд░рдо!

рдирд┐рд╡рд╛рд░рдХ рдЯреАрдХрд╛рдХрд░рдг DTP (adsorbed diphtheria-tetanus pertussis) рдХрд╛ рдкрд╣рд▓реА рдмрд╛рд░ рд╡рд┐рджреЗрд╢реЛрдВ рдореЗрдВ рдкрд┐рдЫрд▓реА рд╕рджреА рдХреЗ 40 рдХреЗ рджрд╢рдХ рдХреЗ рдЕрдВрдд рдореЗрдВ рдЙрдкрдпреЛрдЧ рдХрд┐рдпрд╛ рдЧрдпрд╛ рдерд╛ред DPT рд╡реИрдХреНрд╕реАрди рдХрд╛ рд╡рд┐рджреЗрд╢реА рдПрдирд╛рд▓реЙрдЧ Infanrix рд╣реИред рджреЛрдиреЛрдВ рд╕рдВрдпреЛрдЬрди рдЯреАрдХреЛрдВ рдХреЛ рдкреВрд░реЗ рд╕реЗрд▓ рдЯреАрдХреЛрдВ рдХреЗ рд░реВрдк рдореЗрдВ рд╡рд░реНрдЧреАрдХреГрдд рдХрд┐рдпрд╛ рдЬрд╛рддрд╛ рд╣реИ, рдЕрд░реНрдерд╛рддред рдХрд╛рд▓реА рдЦрд╛рдВрд╕реА (4 IU *), рдЯреЗрдЯрдирд╕ (40 IU рдпрд╛ 60 IU) рдФрд░ рдбрд┐рдкреНрдереАрд░рд┐рдпрд╛ (30 IU) рдХреЗ рдкреНрд░реЗрд░рдХ рдПрдЬреЗрдВрдЯреЛрдВ рдХреА рдореГрдд (рдирд┐рд╖реНрдХреНрд░рд┐рдп) рдХреЛрд╢рд┐рдХрд╛рдУрдВ рд╕реЗ рдпреБрдХреНрддред рдЯреЗрдЯрдирд╕ рдФрд░ рдбрд┐рдкреНрдереАрд░рд┐рдпрд╛ рдЯреЙрдХреНрд╕реЛрдЗрдбреНрд╕ рдХреА рдРрд╕реА рдЦреБрд░рд╛рдХ рдмрдЪреНрдЪреЗ рдХреА рдкреНрд░рддрд┐рд░рдХреНрд╖рд╛ рдкреНрд░рдгрд╛рд▓реА рдХреА рдкреНрд░рддрд┐рдХреНрд░рд┐рдпрд╛ рдХреА рд╡рд╛рдВрдЫрд┐рдд рддреАрд╡реНрд░рддрд╛ рдХреЛ рдкреНрд░рд╛рдкреНрдд рдХрд░рдиреЗ рдХреА рдЖрд╡рд╢реНрдпрдХрддрд╛ рдХреЗ рдХрд╛рд░рдг рд╣реЛрддреА рд╣реИ, рдЬреЛ рдЕрднреА рднреА рдЕрдкреВрд░реНрдг рд╣реИ рдФрд░ рдЕрднреА рдмрди рд░рд╣реА рд╣реИред

*) рдПрдордИ - рдЕрдВрддрд░реНрд░рд╛рд╖реНрдЯреНрд░реАрдп рдЗрдХрд╛рдИ

рддреАрди рдШрд╛рддрдХ рдмреАрдорд╛рд░рд┐рдпреЛрдВ рд╕реЗ рдмрдЪрд╛рдиреЗ рдХреЗ рд▓рд┐рдП 2-3 рдорд╣реАрдиреЗ рдХреА рдЙрдореНрд░ рд╕реЗ рдмрдЪреНрдЪреЛрдВ рдХреЛ рдбреАрдкреАрдЯреА рдЯреАрдХрд╛рдХрд░рдг рджрд┐рдпрд╛ рдЬрд╛рддрд╛ рд╣реИ: рдХрд╛рд▓реА рдЦрд╛рдВрд╕реА, рдбрд┐рдкреНрдереАрд░рд┐рдпрд╛ рдФрд░ рдЯреЗрдЯрдирд╕ (рдПрдХ рд╕рд╛рде рд╡реЗ "рд░рд╣рд╕реНрдпрдордп" рдбреАрдкреАрдЯреА рдХреЛрдб рдЬреЛрдбрд╝рддреЗ рд╣реИрдВ)ред

рдХрд╛рд▓реА рдЦрд╛рдВрд╕реА, рдбрд┐рдкреНрдереАрд░рд┐рдпрд╛ рдФрд░ рдЯрд┐рдЯрдиреЗрд╕ рдмрд╣реБрдд рдЦрддрд░рдирд╛рдХ рд╣реЛрддреЗ рд╣реИрдВ рдФрд░ рдЫреЛрдЯреЗ рдмрдЪреНрдЪреЛрдВ рдореЗрдВ рдпреЗ рдореБрд╢реНрдХрд┐рд▓ рд╣реЛрддреЗ рд╣реИрдВред рдкрд░реНрдЯреБрд╕рд┐рд╕ рдЧрдВрднреАрд░ рдЬрдЯрд┐рд▓рддрд╛рдУрдВ рдХреЗ рд╕рд╛рде рдХрдкрдЯреА рд╣реИ: рдирд┐рдореЛрдирд┐рдпрд╛ (рдирд┐рдореЛрдирд┐рдпрд╛) рдФрд░ рдПрдиреНрд╕реЗрдлреИрд▓реЛрдкреИрдереА (рдорд╕реНрддрд┐рд╖реНрдХ рдХреНрд╖рддрд┐)ред рдПрдХ рдРрдВрдарди рд╡рд╛рд▓реА рдЦрд╛рдВрд╕реА рдкреВрд░реА рддрд░рд╣ рд╕реЗ рд╕рд╛рдВрд╕ рд▓реЗрдирд╛ рдмрдВрдж рдХрд░ рд╕рдХрддреА рд╣реИред рд╡реИрдХреНрд╕реАрди рдХреА рд╢реБрд░реБрдЖрдд рдХреЗ рдмрд╛рдж рд░реЛрдЧ рдкреНрд░рддрд┐рд░реЛрдзрдХ рддрдВрддреНрд░рдПрдВрдЯреАрдмреЙрдбреА рдЙрддреНрдкрдиреНрди рдХрд░рддрд╛ рд╣реИ рдЬрд┐рд╕рд╕реЗ рд╕реНрдореГрддрд┐ рдХреЛрд╢рд┐рдХрд╛рдПрдВ рдмрдирддреА рд╣реИрдВред рдпрджрд┐ рднрд╡рд┐рд╖реНрдп рдореЗрдВ рд╢рд░реАрд░ рдлрд┐рд░ рд╕реЗ рд░реЛрдЧ рдХреЗ рдкреНрд░реЗрд░рдХ рдПрдЬреЗрдВрдЯ (рдХрд╛рд▓реА рдЦрд╛рдВрд╕реА) рдХрд╛ рд╕рд╛рдордирд╛ рдХрд░рддрд╛ рд╣реИ, рддреЛ рдкреНрд░рддрд┐рд░рдХреНрд╖рд╛ рдкреНрд░рдгрд╛рд▓реА, рдЬреИрд╕рд╛ рдХрд┐ рдпрд╣ рдерд╛, "рдпрд╛рдж рд░рдЦрддрд╛ рд╣реИ" рдХрд┐ рдпрд╣ рдкрд╣рд▓реЗ рд╕реЗ рд╣реА рд╡рд╛рдпрд░рд╕ рд╕реЗ рдкрд░рд┐рдЪрд┐рдд рд╣реИ, рдФрд░ рд╕рдХреНрд░рд┐рдп рд░реВрдк рд╕реЗ рд╕реБрд░рдХреНрд╖рд╛рддреНрдордХ рдкреНрд░рддрд┐рдХреНрд░рд┐рдпрд╛рдУрдВ рдХреЛ рд╢рд╛рдорд┐рд▓ рдХрд░рдирд╛ рд╢реБрд░реВ рдХрд░ рджреЗрддрд╛ рд╣реИред рдЯреЗрдЯрдирд╕ рдФрд░ рдбрд┐рдкреНрдереАрд░рд┐рдпрд╛ рдХреА рдЦрд╝рд╛рд╕рд┐рдпрдд рдпрд╣ рд╣реИ рдХрд┐ рд░реЛрдЧ рдХрд╛ рд╡рд┐рдХрд╛рд╕, рдкрд╛рдареНрдпрдХреНрд░рдо рдФрд░ рдЬрдЯрд┐рд▓рддрд╛рдПрдВ рд░реЛрдЧрд╛рдгреБрдУрдВ рд╕реЗ рдирд╣реАрдВ, рдмрд▓реНрдХрд┐ рдЗрд╕рдХреЗ рд╡рд┐рд╖рд╛рдХреНрдд рдкрджрд╛рд░реНрдереЛрдВ рд╕реЗ рдЬреБрдбрд╝реА рд╣реИрдВред рджреВрд╕рд░реЗ рд╢рдмреНрджреЛрдВ рдореЗрдВ, рдмреАрдорд╛рд░реА рдХреЗ рдЧрдВрднреАрд░ рд░реВрдк рд╕реЗ рдмрдЪрдиреЗ рдХреЗ рд▓рд┐рдП, рд╢рд░реАрд░ рдореЗрдВ рд╡рд┐рд╖ рдХреЗ рдЦрд┐рд▓рд╛рдл рдкреНрд░рддрд┐рд░рдХреНрд╖рд╛ рдмрдирд╛рдирд╛ рдЖрд╡рд╢реНрдпрдХ рд╣реИ, рди рдХрд┐ рдкреВрд░реЗ рд╡рд╛рдпрд░рд╕ рдХреЗ рдЦрд┐рд▓рд╛рдлред рдЗрд╕ рдкреНрд░рдХрд╛рд░, рд╡реИрдХреНрд╕реАрди рдХреЛ рд╢рд░реАрд░ рдХреА рдПрдВрдЯреА-рдЯреЙрдХреНрд╕рд┐рдХ рдЗрдореНрдпреБрдирд┐рдЯреА рдмрдирд╛рдиреЗ рдХреЗ рд▓рд┐рдП рдбрд┐рдЬрд╝рд╛рдЗрди рдХрд┐рдпрд╛ рдЧрдпрд╛ рд╣реИред

рдЯреАрдХрд╛ рдПрдХ рдмрд╛рд░ рдирд╣реАрдВ, рдмрд▓реНрдХрд┐ рдЬреАрд╡рди рднрд░ рдХреЗ рдЕрдВрддрд░рд╛рд▓ рдкрд░ рд▓рдЧрд╛рдпрд╛ рдЬрд╛рддрд╛ рд╣реИ (рдпрджрд┐ рдХреЛрдИ рдЧрдВрднреАрд░ рдЪрд┐рдХрд┐рддреНрд╕рд╛ рд╕реНрдерд┐рддрд┐ рдмрдЪреНрдЪреЛрдВ рдФрд░ рд╡рдпрд╕реНрдХреЛрдВ рдХреЗ рд▓рд┐рдП рдЗрд╖реНрдЯрддрдо рдбреАрдЯреАрдкреА рдЯреАрдХрд╛рдХрд░рдг рдХрд╛рд░реНрдпрдХреНрд░рдо рдХреЛ рдкреНрд░рднрд╛рд╡рд┐рдд рдирд╣реАрдВ рдХрд░рддреА рд╣реИ, рдЬреЛ рдЗрд╕ рддрд░рд╣ рджрд┐рдЦрддрд╛ рд╣реИ:

  • 1 - рдмреА 2-3 рдорд╣реАрдиреЗред
  • 2 - рдмреА 4-5 рдорд╣реАрдиреЗред
  • 3 - рдмреА 6 рдорд╣реАрдиреЗред

рдпреЗ рддреАрди рдЦреБрд░рд╛рдХ рдкреНрд░рд╛рдердорд┐рдХ рдкрд╛рдареНрдпрдХреНрд░рдо рдмрдирд╛рддреА рд╣реИрдВред рдпрд╣ рдорд╣рддреНрд╡рдкреВрд░реНрдг рд╣реИ рдХрд┐ рдкреНрд░рд╛рдердорд┐рдХ рдкрд╛рдареНрдпрдХреНрд░рдо рдХреЗ рд╣рд┐рд╕реНрд╕реЗ рдХреЗ рд░реВрдк рдореЗрдВ рдЯреАрдХрд╛рдХрд░рдг рдХреЗ рдмреАрдЪ рдХрд╛ рдЕрдВрддрд░рд╛рд▓ рдХрдо рд╕реЗ рдХрдо 30 рджрд┐рдиреЛрдВ рдХрд╛ рд╣реЛред

  • 4 - рдмреА 18 рдорд╣реАрдиреЗред

рдпреЗ 4 рдбреАрдЯреАрдкреА рдЯреАрдХрд╛рдХрд░рдг рдПрдХ рдкреВрд░реНрдг рдЯреАрдХрд╛рдХрд░рдг рдХрд╛ рдЧрдарди рдХрд░рддреЗ рд╣реИрдВ, рдЬреЛ рд╢рд░реАрд░ рдХреЛ рдбрд┐рдкреНрдереАрд░рд┐рдпрд╛, рдкрд░реНрдЯреБрд╕рд┐рд╕ рдФрд░ рдЯреЗрдЯрдирд╕ рд╕реЗ рдордЬрд╝рдмреВрддреА рд╕реЗ рдмрдЪрд╛рдиреЗ рдХреА рдЕрдиреБрдорддрд┐ рджреЗрддрд╛ рд╣реИред рдЗрд╕рдХреЗ рдЕрд▓рд╛рд╡рд╛, рдПрдХ рдЯреАрдХрд╛рдХрд░рдг (рдЯреАрдХреЗ рдЬреЛ рдЧрддрд┐рд╡рд┐рдзрд┐ рдХреЗ рдЖрд╡рд╢реНрдпрдХ рд╕реНрддрд░ рдкрд░ рдкреНрд░рддрд┐рд░рдХреНрд╖рд╛ рдмрдирд╛рдП рд░рдЦрддреЗ рд╣реИрдВ) рдХреЗ рд░реВрдк рдореЗрдВ, рдПрдХ рдЯреАрдХреЗ рдХрд╛ рдЙрдкрдпреЛрдЧ рдкрд╣рд▓реЗ рд╕реЗ рд╣реА рддрдерд╛рдХрдерд┐рдд рдЕрдХреЛрд╢рд┐рдХреАрдп (рдЕрд░реНрдерд╛рдд, рдЕрдХреЛрд╢рд┐рдХреАрдп) рдкрд░реНрдЯреБрд╕рд┐рд╕ рдШрдЯрдХ рдХреЗ рд╕рд╛рде рдХрд┐рдпрд╛ рдЬрд╛рддрд╛ рд╣реИред рдФрд░ рдЗрд╕ рдорд╛рдорд▓реЗ рдореЗрдВ рдЗрд╕реЗ рдХрд╣рд╛ рдЬрд╛рддрд╛ рд╣реИ - рдПрдбреАрдПрд╕):

  • рд╡реА 6-7 рд╡рд░реНрд╖реЛрдВред
  • рд╡реА 14 рд╕рд╛рд▓ рдФрд░ рдЖрдЧреЗ - рд╣рд░ 10 рд╕рд╛рд▓ рдореЗрдВ: 24, 34, 44, 54, 64 рдФрд░ 74 рд╕рд╛рд▓ рдореЗрдВред

рдбрдмреНрд▓реНрдпреВрдПрдЪрдУ рдХреА рд╕рд┐рдлрд╛рд░рд┐рд╢реЛрдВ рдХреЗ рдмрд╛рд╡рдЬреВрдж, рд░реВрд╕ рдХреА рддреАрди рдЪреМрдерд╛рдИ рд╡рдпрд╕реНрдХ рдЖрдмрд╛рджреА рдХреЛ рдбреАрдЯреАрдкреА рдкреНрд░рддрд┐рд░рдХреНрд╖рдг (рдпрд╛, рдЪрд┐рдХрд┐рддреНрд╕рдХреАрдп рд░реВрдк рд╕реЗ, рдбреАрдЯреАрдкреА) рдкреНрд░рд╛рдкреНрдд рдирд╣реАрдВ рд╣реЛрддрд╛ рд╣реИред рдЗрди рддреАрди рддрд┐рдорд╛рд╣рд┐рдпреЛрдВ рдХреЛ рдпрд╣ рднреА рд╕рдВрджреЗрд╣ рдирд╣реАрдВ рд╣реИ рдХрд┐ рдЙрдиреНрд╣реЗрдВ рдЗрд╕ рддрд░рд╣ рдХреЗ рдЯреАрдХрд╛рдХрд░рдг рдХреА рдЖрд╡рд╢реНрдпрдХрддрд╛ рд╣реИред рдмреВрд╕реНрдЯрд░ рдЯреАрдХрд╛рдХрд░рдг рдХреЗ рдкрдХреНрд╖ рдореЗрдВ рд╕рдмрд╕реЗ рдорд╣рддреНрд╡рдкреВрд░реНрдг рдФрд░ рд╢рдХреНрддрд┐рд╢рд╛рд▓реА рддрд░реНрдХ рдпрд╣ рд╣реИ рдХрд┐ рдпрд╣ рдЯреАрдХрд╛ рд╣рдореЗрдВ рдЯреЗрдЯрдирд╕ рд╕реЗ рдмрдЪрд╛рддрд╛ рд╣реИ, рдореЛрдЯреЗ рддреМрд░ рдкрд░ рдЦрддрд░рдирд╛рдХ рдмреАрдорд╛рд░реАрдЖрдЬ рднреАред рдЬреЛ рд▓реЛрдЧ рд╡рд┐рджреЗрд╢ рдпрд╛рддреНрд░рд╛ рдХрд░рдирд╛ рдкрд╕рдВрдж рдХрд░рддреЗ рд╣реИрдВ, рдЙрдирдХреЗ рд▓рд┐рдП рдЕрд╡рд┐рдХрд╕рд┐рдд рд▓реЛрдЧреЛрдВ рдиреЗ рд╡рд┐рд╢реЗрд╖ рд░реВрдк рд╕реЗ рдЯрд┐рдЯрдиреЗрд╕ рд╣реЛрдиреЗ рдХрд╛ рдЬреЛрдЦрд┐рдо рджрд┐рдпрд╛ред

рдорддрднреЗрдж

рд╕рд╛рдорд╛рдиреНрдп contraindications рдореЗрдВ рдПрдХ рдкреБрд░рд╛рдиреА рдмреАрдорд╛рд░реА рдХрд╛ рддреЗрдЬ рд╣реЛрдирд╛, рдмреБрдЦрд╛рд░, рд╡реИрдХреНрд╕реАрди рдШрдЯрдХреЛрдВ рд╕реЗ рдПрд▓рд░реНрдЬреА рдФрд░ рдЧрдВрднреАрд░ рдЗрдореНрдпреБрдиреЛрдбреЗрдлрд┐рд╢рд┐рдПрдВрд╕реА рд╢рд╛рдорд┐рд▓ рд╣реИрдВред рдбреАрдЯреАрдкреА рдЯреАрдХрд╛рдХрд░рдг рдЕрд╕реНрдерд╛рдпреА рд░реВрдк рд╕реЗ рдпрд╛ рдкреВрд░реА рддрд░рд╣ рд╕реЗ contraindicated рд╣реИ рдпрджрд┐ рдмрдЪреНрдЪреЗ рдХреЛ рдЖрдХреНрд╖реЗрдк рд╣реИ рдЬреЛ рддрд╛рдкрдорд╛рди рдореЗрдВ рд╡реГрджреНрдзрд┐ рд╕реЗ рдЬреБрдбрд╝рд╛ рдирд╣реАрдВ рд╣реИ, рдпрд╛ рдПрдХ рдкреНрд░рдЧрддрд┐рд╢реАрд▓ рд╡рд┐рдХреГрддрд┐ рд╣реИ рддрдВрддреНрд░рд┐рдХрд╛ рдкреНрд░рдгрд╛рд▓реА... рдлрд┐рд░ рдмрдЪреНрдЪреЛрдВ рдХреЛ рдПрдХ рдЯреАрдХрд╛ рд▓рдЧрд╛рдпрд╛ рдЬрд╛рддрд╛ рд╣реИ рдЬрд┐рд╕рдореЗрдВ рдЗрд╕рдХреА рд╕рдВрд░рдЪрдирд╛ рдореЗрдВ рдкрд░реНрдЯреБрд╕рд┐рд╕ рдШрдЯрдХ рдирд╣реАрдВ рд╣реЛрддрд╛ рд╣реИред

рдХреНрдпрд╛ рдбреАрдкреАрдЯреА рд╡реИрдХреНрд╕реАрди рдПрдХ рд╣реИ, рд▓реЗрдХрд┐рди рд╕рднреА рдЯреАрдХреЗ рдЕрд▓рдЧ-рдЕрд▓рдЧ рд╣реЛрддреЗ рд╣реИрдВ?

рдХрдИ рдбрдмреНрд▓реНрдпреВрдПрдЪрдУ-рдкреНрд░рдорд╛рдгрд┐рдд рджрд╡рд╛рдПрдВ рд╣реИрдВ - рдбреАрдкреАрдЯреА рдЯреАрдХреЗред рдЕрдХреНрд╕рд░ рдРрд╕реА рд╕реНрдерд┐рддрд┐рдпрд╛рдВ рд╣реЛрддреА рд╣реИрдВ рдЬрдм рдПрдХ рдмрдЪреНрдЪреЗ рдХреЛ рдПрдХ рдирд┐рд░реНрдорд╛рддрд╛ рд╕реЗ рджреВрд╕рд░реЗ рдЯреАрдХреЗ рдХреЗ рд╕рд╛рде рдЗрдВрдЬреЗрдХреНрд╢рди рджрд┐рдпрд╛ рдЬрд╛рддрд╛ рд╣реИ, рдФрд░ рдмрд╛рдж рдореЗрдВ рдХрд┐рд╕реА рдЕрдиреНрдп рдирд┐рд░реНрдорд╛рддрд╛ рд╕реЗ рджрд╡рд╛ рдХреЗ рд╕рд╛рде рдЯреАрдХрд╛рдХрд░рдг рдХрд┐рдпрд╛ рдЬрд╛рддрд╛ рд╣реИред рдХреНрдпрд╛ рдЗрд╕рдореЗрдВ рдХреЛрдИ рдЬреЛрдЦрд┐рдо рдпрд╛ рдкрдХрдбрд╝ рд╣реИ?

рд╡рд┐рд╢реНрд╡ рд╕реНрд╡рд╛рд╕реНрдереНрдп рд╕рдВрдЧрдарди рджреНрд╡рд╛рд░рд╛ рдкреНрд░рдорд╛рдгрд┐рдд рдбреАрдЯреАрдкреА рдЯреАрдХрд╛рдХрд░рдг рдХреА рд╕рднреА рддреИрдпрд╛рд░рд┐рдпрд╛рдВ рдЕрджрд▓рд╛-рдмрджрд▓реА рдХреА рдЬрд╛ рд╕рдХрддреА рд╣реИрдВ! рдХрд┐рд╕реА рд╡рд┐рд╢реЗрд╖ рдирд┐рд░реНрдорд╛рддрд╛ рдХреЛ "рдкрдХрдбрд╝рдиреЗ" рдХрд╛ рдХреЛрдИ рдорддрд▓рдм рдирд╣реАрдВ рд╣реИ рдХреНрдпреЛрдВрдХрд┐ рдЖрдкрдиреЗ "рд╢реБрд░реВ рдХрд┐рдпрд╛"ред

рдЯреАрдХреЛрдВ рдХреА рдЧреБрдгрд╡рддреНрддрд╛ рдХреЗ рдмрд╛рд░реЗ рдореЗрдВ рдХреБрдЫ рд╢рдмреНрдж: рдЖрдзреБрдирд┐рдХ рджреБрдирд┐рдпрд╛рдЯрд┐рдЯрдиреЗрд╕, рдбрд┐рдкреНрдереАрд░рд┐рдпрд╛ рдФрд░ рдХрд╛рд▓реА рдЦрд╛рдВрд╕реА рдХреЗ рд▓рд┐рдП рджреЛ рдореБрдЦреНрдп рдЯреАрдХреЗ рд╣реИрдВред рдПрдХ рдХреНрд▓рд╛рд╕рд┐рдХ, рд╕рд╕реНрддрд╛ рдФрд░ рдмрд╣реБрдд (рдЗрд╕рдХреА рдХрдо рд▓рд╛рдЧрдд рдХреЗ рдХрд╛рд░рдг) рдЦрд░рд╛рдм рд╡рд┐рдХрд╕рд┐рдд рджреЗрд╢реЛрдВ рдореЗрдВ рд╡реНрдпрд╛рдкрдХ рд╣реИ, рдЕрдореАрд░ рджреЗрд╢реЛрдВ рдореЗрдВ рдирд╣реАрдВред рдЗрд╕рдореЗрдВ рдареАрдХ рдЙрд╕реА рдкреНрд░рдХрд╛рд░ рдХрд╛ рдкрд░реНрдЯреБрд╕рд┐рд╕ рдШрдЯрдХ (рд╡рд┐рднрд╛рдЬрд┐рдд рдирд╣реАрдВ, рд╢реБрджреНрдз рдирд╣реАрдВ) рд╣реЛрддрд╛ рд╣реИ, рдЬреЛ рдЙрди рд╕рднреА рдХрдИ рдирдХрд╛рд░рд╛рддреНрдордХ рдкрдХреНрд╖ рдкреНрд░рддрд┐рдХреНрд░рд┐рдпрд╛рдУрдВ рдХрд╛ рдХрд╛рд░рдг рдмрдирддрд╛ рд╣реИ рдЬрд┐рдирдХрд╛ рдЕрдХреНрд╕рд░ рдбреАрдкреАрдЯреА рдЯреАрдХрд╛рдХрд░рдг рдХреЗ рд╕рдВрдмрдВрдз рдореЗрдВ рдЙрд▓реНрд▓реЗрдЦ рдХрд┐рдпрд╛ рдЬрд╛рддрд╛ рд╣реИред

рджреВрд╕рд░рд╛ рдкреНрд░рдХрд╛рд░ - рддрдерд╛рдХрдерд┐рдд рдПрдПрдбреАрдПрд╕ рд╡реИрдХреНрд╕реАрди - рдХреНрд▓рд╛рд╕рд┐рдХ рдбреАрдкреАрдЯреА рд╡реИрдХреНрд╕реАрди рдХрд╛ рдПрдХ рдЕрдзрд┐рдХ рдЖрдзреБрдирд┐рдХ рдФрд░ рдорд╣рдВрдЧрд╛ рдПрдирд╛рд▓реЙрдЧ рд╣реИ, рдЬрд┐рд╕рдореЗрдВ рдкрд░реНрдЯреБрд╕рд┐рд╕ рдШрдЯрдХ рдХреЛ рдШрдЯрдХреЛрдВ рдореЗрдВ рд╡рд┐рднрд╛рдЬрд┐рдд рдХрд┐рдпрд╛ рдЬрд╛рддрд╛ рд╣реИ рдФрд░ рд╢реБрджреНрдз рдХрд┐рдпрд╛ рдЬрд╛рддрд╛ рд╣реИред рдЗрд╕ рдЯреАрдХреЗ рдХреЛ рд╕рд╣рди рдХрд░рдирд╛ рдмрд╣реБрдд рдЖрд╕рд╛рди рд╣реИ, рд╡реНрдпрд╛рд╡рд╣рд╛рд░рд┐рдХ рд░реВрдк рд╕реЗ рдирдХрд╛рд░рд╛рддреНрдордХ рдкрдХреНрд╖ рдкреНрд░рддрд┐рдХреНрд░рд┐рдпрд╛ рдкреИрджрд╛ рдХрд┐рдП рдмрд┐рдирд╛ред

рд╣рд╛рд▓рд╛рдВрдХрд┐, рдХрд┐рд╕реА рдХреЛ рдпрд╣ рд╕рдордЭрдирд╛ рдЪрд╛рд╣рд┐рдП рдХрд┐ рдкреНрд░рддрд┐рдХреВрд▓ рдкреНрд░рддрд┐рдХреНрд░рд┐рдпрд╛рдПрдВ рдХрд▓реНрдпрд╛рдг рдХреЗ рдЕрд╕реНрдерд╛рдпреА рд╡рд┐рдХрд╛рд░ рд╣реИрдВ рдЬреЛ рд╕рд╛рдорд╛рдиреНрдп рд░реВрдк рд╕реЗ рд╕реНрд╡рд╛рд╕реНрдереНрдп рдХреЛ рдЦрддрд░рд╛ рдирд╣реАрдВ рджреЗрддреЗ рд╣реИрдВ рдФрд░ рдмрд┐рдирд╛ рдХрд┐рд╕реА рдирд┐рд╢рд╛рди рдХреЗ рдЧреБрдЬрд░рддреЗ рд╣реИрдВред рдЬрдЯрд┐рд▓рддрд╛рдУрдВ рдХреЗ рд╡рд┐рдкрд░реАрдд - рдЬреЛ рд╕реНрд╡рд╛рд╕реНрдереНрдп рдХреЛ рдкреНрд░рднрд╛рд╡рд┐рдд рдХрд░рддреА рд╣реИрдВ рдФрд░ рджреАрд░реНрдШрдХрд╛рд▓рд┐рдХ рдЙрдкрдЪрд╛рд░ рдХреА рдЖрд╡рд╢реНрдпрдХрддрд╛ рд╣реЛрддреА рд╣реИред рдЗрд╕рд▓рд┐рдП, рдЬрд╣рд╛рдВ рддрдХ тАЛтАЛрдЬрдЯрд┐рд▓рддрд╛рдУрдВ рдХрд╛ рд╕рдВрдмрдВрдз рд╣реИ, рдбреАрдкреАрдЯреА рдЯреАрдХреЗ рдХреА рдХреЛрдИ рднреА рдХрд┐рд╕реНрдо, рдЪрд╛рд╣реЗ рд╡рд╣ рдкреБрд░рд╛рдиреА рд╣реЛ рдпрд╛ рдирдИ, рдореЗрдВ рдЗрди рдЬрдЯрд┐рд▓рддрд╛рдУрдВ рдХрд╛ рдЙрдЪреНрдЪ рдЬреЛрдЦрд┐рдо рдирд╣реАрдВ рд╣реИ (рдЙрдирдХреЗ рдмрд╛рд░реЗ рдореЗрдВ рдЕрдзрд┐рдХ рдиреАрдЪреЗ)ред

рд╕реНрд╡рд╛рд╕реНрдереНрдп рджреЗрдЦрднрд╛рд▓ рдкреЗрд╢реЗрд╡рд░ рдХреЛ рдЗрдВрдЬреЗрдХреНрд╢рди рдХрд╣рд╛рдБ рджреЗрдирд╛ рдЪрд╛рд╣рд┐рдП?

рд╕рд┐рджреНрдзрд╛рдВрдд рд░реВрдк рдореЗрдВ, рдЯреАрдХреЗ рдХреЛ рдХрд┐рд╕реА рднреА рдорд╛рдВрд╕рдкреЗрд╢реА рдКрддрдХ рдореЗрдВ рдЗрдВрдЬреЗрдХреНрдЯ рдХрд┐рдпрд╛ рдЬрд╛ рд╕рдХрддрд╛ рд╣реИред рдкрд░рдВрддреБ! рд╡рд┐рд╢реНрд╡ рд╕реНрд╡рд╛рд╕реНрдереНрдп рд╕рдВрдЧрдарди рдХреА рд╕рд┐рдлрд╛рд░рд┐рд╢ рдЫреЛрдЯреЗ рдмрдЪреНрдЪреЛрдВ рдХреЛ рдЬрд╛рдВрдШ рдореЗрдВ рд╣реА рдЗрдВрдЬреЗрдХреНрд╢рди рджреЗрдВ... рддрдереНрдп рдпрд╣ рд╣реИ рдХрд┐ 2 рдорд╣реАрдиреЗ рдХреЗ рдмрдЪреНрдЪреЗ рдореЗрдВ рднреА, рдЬрд╛рдВрдШ рдкрд░ рдорд╛рдВрд╕рдкреЗрд╢рд┐рдпрд╛рдВ рдЕрдЪреНрдЫреА рддрд░рд╣ рд╕реЗ рд╡рд┐рдХрд╕рд┐рдд рд╣реЛрддреА рд╣реИрдВ, рдХреБрдЫ рд░рдХреНрдд рд╡рд╛рд╣рд┐рдХрд╛рдПрдВ рдФрд░ рдЪрдордбрд╝реЗ рдХреЗ рдиреАрдЪреЗ рдХреЗ рд╡рд╕рд╛ рдКрддрдХ (рдЧреНрд▓реВрдЯрд┐рдпрд▓ рдорд╛рдВрд╕рдкреЗрд╢рд┐рдпреЛрдВ рдХреЗ рд╡рд┐рдкрд░реАрдд) рд╣реЛрддреЗ рд╣реИрдВред

рдпрджрд┐ рдПрдХ рдирд░реНрд╕ рдпрд╛ рдбреЙрдХреНрдЯрд░, рдЯреАрдХрд╛рдХрд░рдг рдХреЗ рджреМрд░рд╛рди, рдЖрдкрдХреЗ рдмрдЪреНрдЪреЗ рдХреЗ рдиреАрдЪреЗ "рд▓рдХреНрд╖реНрдп" рдХрд░рддрд╛ рд╣реИ, рддреЛ рдЙрд╕реЗ рд░реЛрдХреЗрдВ, 2008 рд╕реЗ рд░реВрд╕ рдореЗрдВ рд▓рд╛рдЧреВ рдПрдХ рдЖрдзрд┐рдХрд╛рд░рд┐рдХ рджрд╕реНрддрд╛рд╡реЗрдЬ рдХрд╛ рдЬрд┐рдХреНрд░ рдХрд░рддреЗ рд╣реБрдП, рдЬрд┐рд╕реЗ "рд╕реНрд╡рдЪреНрдЫрддрд╛ рдФрд░ рдорд╣рд╛рдорд╛рд░реА рд╡рд┐рдЬреНрдЮрд╛рди рдирд┐рдпрдо" рдХрд╣рд╛ рдЬрд╛рддрд╛ рд╣реИред рдЯреАрдХрд╛рдХрд░рдг рдХреА рд╕реБрд░рдХреНрд╖рд╛ рд╕реБрдирд┐рд╢реНрдЪрд┐рдд рдХрд░рдирд╛ тАЭред рдЗрд╕ рджрд╕реНрддрд╛рд╡реЗрдЬрд╝ рдХреЗ рдЕрдиреБрд╕рд╛рд░ рд╡рд┐рдХрд╕рд┐рдд рдХрд┐рдпрд╛ рдЧрдпрд╛ рд╣реИ рд╕рдВрдШреАрдп рдХрд╛рдиреВрдирдПрди 52-рдПрдлрдЬреЗрдб "рдЬрдирд╕рдВрдЦреНрдпрд╛ рдХреЗ рд╕реНрд╡рдЪреНрдЫрддрд╛ рдФрд░ рдорд╣рд╛рдорд╛рд░реА рд╡рд┐рдЬреНрдЮрд╛рди рдХрд▓реНрдпрд╛рдг рдкрд░", рдФрд░ рдкреИрд░рд╛рдЧреНрд░рд╛рдл 3.37 рдореЗрдВ рдПрдХ рдиреБрд╕реНрдЦрд╛ рд╣реИ: "рдЬреАрд╡рди рдХреЗ рдкрд╣рд▓реЗ рд╡рд░реНрд╖реЛрдВ рдореЗрдВ рдмрдЪреНрдЪреЛрдВ рдХреЗ рд▓рд┐рдП рдЗрдВрдЯреНрд░рд╛рдорд╕реНрдХреНрдпреБрд▓рд░ рдЗрдВрдЬреЗрдХреНрд╢рди рдХреЗрд╡рд▓ рдордзреНрдп рднрд╛рдЧ рдХреА рдКрдкрд░реА рдмрд╛рд╣рд░реА рд╕рддрд╣ рдореЗрдВ рдХрд┐рдпрд╛ рдЬрд╛рддрд╛ рд╣реИред рдЬрд╛рдВрдШред"

рд▓реЗрдХрд┐рди 6-7 рд╕рд╛рд▓ рдХреЗ рдмрдЪреНрдЪреЛрдВ рдФрд░ рд╡рдпрд╕реНрдХреЛрдВ рдХреЗ рд▓рд┐рдП, рдЖрдорддреМрд░ рдкрд░ рдХрдВрдзреЗ рдХреЗ рдХреНрд╖реЗрддреНрд░ рдореЗрдВ рдЯреАрдХрд╛ рд▓рдЧрд╛рдпрд╛ рдЬрд╛рддрд╛ рд╣реИред

рдбреАрдЯреАрдкреА рдЯреАрдХрд╛рдХрд░рдг рдЯреАрдХрд╛рдХрд░рдг рдХреИрд▓реЗрдВрдбрд░ рдореЗрдВ рд╕рдмрд╕реЗ рдЕрдзрд┐рдХ рдкреНрд░рддрд┐рдХреНрд░рд┐рдпрд╛рд╢реАрд▓ рдЯреАрдХрд╛ рд╣реИред рдЧреНрд░рд╛рдлреНрдЯрд┐рдВрдЧ рдХреА рдкреНрд░рддрд┐рдХреНрд░рд┐рдпрд╛рд╢реАрд▓рддрд╛ рдореБрдЦреНрдп рд░реВрдк рд╕реЗ рдкрд░реНрдЯреБрд╕рд┐рд╕ рдШрдЯрдХ, рдореБрдЦреНрдп рд░реВрдк рд╕реЗ рдкрд░реНрдЯреБрд╕рд┐рд╕ рдЯреЙрдХреНрд╕рд┐рди рдФрд░ рд▓рд┐рдкреЛрдкреЙрд▓реАрд╕реЗрдХреЗрд░рд╛рдЗрдб рдХреЗ рдХрд╛рд░рдг рд╣реЛрддреА рд╣реИред рд╡рд┐рд╖ рд░рдХреНрддрд╕реНрд░рд╛рд╡, рд▓рдЧрд╛рддрд╛рд░ рд╡рд╛рд╣рд┐рдХрд╛рд╕рдВрдХреАрд░реНрдгрди, рдпрдХреГрдд, рдкреНрд▓реАрд╣рд╛ рдФрд░ рдЧреБрд░реНрджреЗ рдореЗрдВ рдЕрдкрдХреНрд╖рдпреА рдкрд░рд┐рд╡рд░реНрддрди рдХрд╛ рдХрд╛рд░рдг рдмрдирддрд╛ рд╣реИред рдпрд╣ рдЗрди рдкрджрд╛рд░реНрдереЛрдВ рдХреА рдХреНрд░рд┐рдпрд╛ рд╣реИ, рдЬреЛ рдХрднреА-рдХрднреА рдЯреАрдХреЗ рдореЗрдВ рдЯреНрд░реЗрд╕ рдорд╛рддреНрд░рд╛ рдореЗрдВ рдореМрдЬреВрдж рд╣реЛрддреЗ рд╣реИрдВ, рдЬреЛ рдбреАрдЯреАрдкреА рдЯреАрдХрд╛рдХрд░рдг рдХреЗ рдкреНрд░рд╢рд╛рд╕рди рдХреЗ рд▓рд┐рдП рд╢реБрд░реБрдЖрддреА рд╣рд▓реНрдХреА рдкреНрд░рддрд┐рдХреНрд░рд┐рдпрд╛рдУрдВ рдХреА рдЙрдкрд╕реНрдерд┐рддрд┐ рдХреА рд╡реНрдпрд╛рдЦреНрдпрд╛ рдХрд░рддрд╛ рд╣реИред рдбреАрдЯреАрдкреА рд╡реИрдХреНрд╕реАрди рдХреЗ рдкреНрд░рд╢рд╛рд╕рди рдХреЗ рд▓рд┐рдП 95% рд╡рд┐рд╖рд╛рдХреНрдд рдкреНрд░рддрд┐рдХреНрд░рд┐рдпрд╛рдПрдВ рдкрд╣рд▓реЗ рджрд┐рди рдХреЗ рднреАрддрд░ рд╡рд┐рдХрд╕рд┐рдд рд╣реЛрддреА рд╣реИрдВред рддрд╛рдкрдорд╛рди 39-40 рдбрд┐рдЧреНрд░реА рд╕реЗрд▓реНрд╕рд┐рдпрд╕ рддрдХ рдмрдврд╝ рдЬрд╛рддрд╛ рд╣реИ, рдЙрд▓реНрдЯреА, рд╕реБрд╕реНрддреА, рдиреАрдВрдж рдореЗрдВ рдЦрд▓рд▓ рд╣реЛ рд╕рдХрддрд╛ рд╣реИред 1-3 рджрд┐рдиреЛрдВ рдореЗрдВ рд▓рдХреНрд╖рдг рдЧрд╛рдпрдм рд╣реЛ рдЬрд╛рддреЗ рд╣реИрдВред

рд╕реНрдерд╛рдиреАрдп рдФрд░ рд╕рд╛рдорд╛рдиреНрдп рдПрд▓рд░реНрдЬреА рдкреНрд░рддрд┐рдХреНрд░рд┐рдпрд╛рдУрдВ рдХрд╛ рдХрд╛рд░рдг рдЕрдзрд┐рдХ рдмрд╛рд░ рдЯреЙрдХреНрд╕реЛрдЗрдб рд╣реЛрддрд╛ рд╣реИ, рдкреНрд░рддрд┐рдХреНрд░рд┐рдпрд╛рдПрдВ рдбреАрдЯреАрдкреА рд╡реИрдХреНрд╕реАрди рдХреЗ рдкреНрд░рд╢рд╛рд╕рди рдХреЗ рдХреБрдЫ рдШрдВрдЯреЛрдВ рдХреЗ рднреАрддрд░ рд╡рд┐рдХрд╕рд┐рдд рд╣реЛрддреА рд╣реИрдВред рдЗрдВрдЬреЗрдХреНрд╢рди рд╕реНрдерд▓ рдкрд░, рд╣рд╛рдЗрдкрд░рдорд┐рдпрд╛, рджрд░реНрдж, рдорд╣рддреНрд╡рдкреВрд░реНрдг рд╕реВрдЬрди рджрд┐рдЦрд╛рдИ рджреЗрддреА рд╣реИред рддрд╛рдкрдорд╛рди рдЕрдХреНрд╕рд░ рдмрдврд╝ рдЬрд╛рддрд╛ рд╣реИред рд╕рд╛рдорд╛рдиреНрдп рдкреНрд░рддрд┐рдХреНрд░рд┐рдпрд╛рдУрдВ рдореЗрдВ рдХреНрд╡рд┐рдиреНрдХреЗ рдХреА рдПрдбрд┐рдорд╛, рдкрд┐рддреНрддреА, рдмрд╣реБрд░реВрдкреА рдФрд░ рд░рдХреНрддрд╕реНрд░рд╛рд╡реА рджрд╛рдиреЗ рд╢рд╛рдорд┐рд▓ рд╣реИрдВред рд╕рд╣рд╡рд░реНрддреА рддреНрд╡рдЪрд╛ рд░реЛрдЧ (рдПрдХреНрдЬрд┐рдорд╛, рдЬрд┐рд▓реНрдж рдХреА рд╕реВрдЬрди, рдиреНрдпреВрд░реЛрдбрд░реНрдорд╛реЗрдЯрд╛рдЗрдЯрд┐рд╕) рдмрдврд╝ рдЬрд╛рддреЗ рд╣реИрдВред рджреБрд░реНрд▓рдн рдорд╛рдорд▓реЛрдВ рдореЗрдВ, рдЯреАрдХрд╛рдХрд░рдг рдмреНрд░реЛрдВрдХреЛрд╕реНрдкрдЬрд╝рдо, рд╡рд┐рд╖рд╛рдХреНрдд-рдПрд▓рд░реНрдЬреА рдХреА рд╕реНрдерд┐рддрд┐, рд╣реЗрдореЛрд▓рд┐рдЯрд┐рдХ-рдпреВрд░реЗрдорд┐рдХ рд╕рд┐рдВрдбреНрд░реЛрдо рдХреЛ рднрдбрд╝рдХрд╛рддрд╛ рд╣реИред рдпрд╣ рдЕрддреНрдпрдВрдд рджреБрд░реНрд▓рдн рд╣реИ (рдкреНрд░рддрд┐ 100, 000 рдЯреАрдХреЗ рдХреЗ рдЗрдВрдЬреЗрдХреНрд╢рди рдХреЗ 2 рдорд╛рдорд▓реЗ) рдХрд┐ рдПрдирд╛рдлрд┐рд▓реЗрдХреНрдЯрд┐рдХ рдЭрдЯрдХрд╛ рд╣реЛрддрд╛ рд╣реИ, рдЬреЛ рдЕрдХреНрд╕рд░ рдмрдбрд╝реЗ рдмрдЪреНрдЪреЛрдВ рдореЗрдВ рд╡реИрдХреНрд╕реАрди рдХреЗ рдмрд╛рд░-рдмрд╛рд░ рдкреНрд░рд╢рд╛рд╕рди рдХреЗ рдмрд╛рдж рддреБрд░рдВрдд рдпрд╛ 20-30 рдорд┐рдирдЯ рдХреЗ рднреАрддрд░ рджреЗрдЦрд╛ рдЬрд╛рддрд╛ рд╣реИред рдмрдЪреНрдЪреЛрдВ рдХреЗ рд▓рд┐рдП рдкреНрд░рд╛рд░рдВрднрд┐рдХ рдЕрд╡рд╕реНрдерд╛рдЯреАрдХрд╛рдХрд░рдг рдХреЗ рдПрдХ рд╕рдкреНрддрд╛рд╣ рдмрд╛рдж рдЕрдзрд┐рдХ рд╡рд┐рд╢рд┐рд╖реНрдЯ рдкрддрди рд╣реЛрддрд╛ рд╣реИред

рдбреАрдЯреАрдкреА рд╡реИрдХреНрд╕реАрди рдХреЗ рдкрд░реНрдЯреБрд╕рд┐рд╕ рдШрдЯрдХ рдХрд╛ рдкрд░рд┐рд╕рдВрдЪрд╛рд░реА рдПрдВрдЯреАрдмреЙрдбреА рдХреЗ рдирд┐рд░реНрдорд╛рдг рдкрд░ рдПрдХ рд╕реНрдкрд╖реНрдЯ рдкреЙрд▓реАрдХреНрд▓реЛрдирд▓ рдкреНрд░рднрд╛рд╡ рд╣реЛрддрд╛ рд╣реИ, рдЬрд┐рд╕рдореЗрдВ рдЖрдИрдЬреАрдИ рд╡рд░реНрдЧ рдХреЗ рд░реАрдЧрд┐рди рдПрдВрдЯреАрдмреЙрдбреА рд╢рд╛рдорд┐рд▓ рд╣реИрдВред рдЗрд╕ рд╕рдВрдмрдВрдз рдореЗрдВ, рдЯреАрдХреЗ рдбреАрдкреАрдЯреА рд╡реИрдХреНрд╕реАрди рд╕реЗ рдЕрд╕рдВрдмрдВрдзрд┐рдд рдПрд▓рд░реНрдЬреА рдХреЗ рд▓рд┐рдП рдПрд▓рд░реНрдЬреА рдкреНрд░рддрд┐рдХреНрд░рд┐рдпрд╛рдУрдВ рдореЗрдВ рдпреЛрдЧрджрд╛рди рдХрд░ рд╕рдХрддреЗ рд╣реИрдВ, рдЙрджрд╛рд╣рд░рдг рдХреЗ рд▓рд┐рдП, рдкрд░рд╛рдЧ, рдШрд░ рдХреА рдзреВрд▓, рдЖрджрд┐ред

рддрдВрддреНрд░рд┐рдХрд╛ рддрдВрддреНрд░ рдХреЗ рдШрд╛рд╡реЛрдВ рдореЗрдВ рд╕реЗ, рдРрдВрдарди рдкреНрд░рддрд┐рдХреНрд░рд┐рдпрд╛рдПрдВ рд╕рдмрд╕реЗ рд╡рд┐рд╢рд┐рд╖реНрдЯ рд╣реИрдВ (рдкреНрд░рддрд┐ 10,000 рдЗрдВрдЬреЗрдХреНрд╢рди рдореЗрдВ 8-14 рдорд╛рдорд▓реЗ)ред рдмрдЪреНрдЪреЛрдВ рдореЗрдВ рдЯреАрдХрд╛рдХрд░рдг рдХреЗ рдмрд╛рдж рдкрд╣рд▓реЗ рджреЛ рджрд┐рдиреЛрдВ рдореЗрдВ рдореБрдЦреНрдп рд░реВрдк рд╕реЗ рдЬрдЯрд┐рд▓рддрд╛рдПрдБ рджрд┐рдЦрд╛рдИ рджреЗрддреА рд╣реИрдВ рдмрдЪрдкрди... рдЬреНрд╡рд░ рдХреЗ рджреМрд░реЗ рдЕрдХреНрд╕рд░ рдЕрддрд┐рддрд╛рдк рдХреА рдкреГрд╖реНрдарднреВрдорд┐ рдХреЗ рдЦрд┐рд▓рд╛рдл рд╣реЛрддреЗ рд╣реИрдВ, рдХрдИ рд╕реЗрдХрдВрдб рд╕реЗ рд▓реЗрдХрд░ рдХрдИ рдорд┐рдирдЯ рддрдХ, рдЖрдорддреМрд░ рдкрд░ рдПрдХрд▓, рдХрднреА-рдХрднреА рдЪреЗрддрдирд╛ рдХреЗ рдиреБрдХрд╕рд╛рди рдХреЗ рд╕рд╛рдеред рдЕрдзрд┐рдХ рдореЗрдВ рд▓реЗрдЯ рдбреЗрдЯреНрд╕(3-5 рджрд┐рди рдФрд░ рдмрд╛рдж рдореЗрдВ) рдЬреНрд╡рд░ рдХреЗ рджреМрд░реЗ рд╡рд┐рдХрд╕рд┐рдд рд╣реЛ рд╕рдХрддреЗ рд╣реИрдВ, рдЬреЛ рдмрдбрд╝реЗ рдмрд╣реБрд░реВрдкрддрд╛ рдХреА рд╡рд┐рд╢реЗрд╖рддрд╛ рд╣реИ рдФрд░ рдЕрдзрд┐рдХ рдмрд╛рд░ рддрдм рд╣реЛрддрд╛ рд╣реИ рдЬрдм рдмрдЪреНрдЪрд╛ рд╕реЛ рдЬрд╛рддрд╛ рд╣реИ рдФрд░ рдЬрд╛рдЧ рдЬрд╛рддрд╛ рд╣реИред

рдЬреАрд╡рди рдХреЗ рдкрд╣рд▓реЗ рдЫрд╣ рдорд╣реАрдиреЛрдВ рдХреЗ рдмрдЪреНрдЪреЛрдВ рдореЗрдВ, рдЯреАрдХрд╛рдХрд░рдг рдХреЗ рдХреБрдЫ рдШрдВрдЯреЛрдВ рдмрд╛рдж, рд╢реНрд░рд┐рд▓ рд╕реНрдХреНрд░реАрдорд┐рдВрдЧ рд╕рд┐рдВрдбреНрд░реЛрдо рджрд┐рдЦрд╛рдИ рджреЗ рд╕рдХрддрд╛ рд╣реИред рдЙрдЪреНрдЪ рд╕реНрд╡рд░ рдореЗрдВ рдЪреАрдЦрдирд╛ 1 рд╕реЗ 10 рдШрдВрдЯреЗ рддрдХ рд░рд╣рддрд╛ рд╣реИ рдФрд░ рдЕрдзрд┐рдХ рдЧрдВрднреАрд░ рдиреНрдпреВрд░реЛрд▓реЙрдЬрд┐рдХрд▓ рдЬрдЯрд┐рд▓рддрд╛рдУрдВ рдХрд╛ рдЕрдЧреНрд░рджреВрдд рд╣реЛ рд╕рдХрддрд╛ рд╣реИред рдпрд╣ рдорд╛рдирд╛ рдЬрд╛рддрд╛ рд╣реИ рдХрд┐ рд░реЛрдирд╛ рд╕реАрдПрд╕рдПрдл рдЧрдбрд╝рдмрдбрд╝реА рдХреЗ рдХрд╛рд░рдг рд╣реЛрдиреЗ рд╡рд╛рд▓реЗ рдЧрдВрднреАрд░ рд╕рд┐рд░рджрд░реНрдж рд╕реЗ рдЬреБрдбрд╝рд╛ рд╣реИред

рдЯреАрдХрд╛рдХрд░рдг рдХреЗ рдмрд╛рдж рдХреЗ 3-8 рджрд┐рдиреЛрдВ рдХреЗ рдмрд╛рдж рдЯреАрдХрд╛рдХрд░рдг рдХреЗ рдмрд╛рдж рдПрдиреНрд╕реЗрдлрд▓рд╛рдЗрдЯрд┐рд╕ рджреБрд░реНрд▓рдн рдорд╛рдорд▓реЛрдВ рдореЗрдВ рд╡рд┐рдХрд╕рд┐рдд рд╣реЛрддрд╛ рд╣реИ (рдкреНрд░рддрд┐ 250,000-500,000 рдЗрдВрдЬреЗрдХреНрд╢рди рдореЗрдВ 1 рдорд╛рдорд▓рд╛), рдЬрдмрдХрд┐ рдЖрдХреНрд╖реЗрдк рдордирд╛рдпрд╛ рдЬрд╛рддрд╛ рд╣реИ, рдЕрдХреНрд╕рд░ рджреЛрд╣рд░рд╛рдпрд╛ рдЬрд╛рддрд╛ рд╣реИ, рд╣рд╛рдЗрдкрд░рдХрд┐рдиреЗрд╕рд┐рд╕, рдЪрд░рдо рд╕реАрдорд╛рдУрдВ рдХрд╛ рдкреИрд░реЗрд╕рд┐рд╕ред рджреБрд░реНрднрд╛рдЧреНрдп рд╕реЗ, рдбреАрдЯреАрдкреА рд╡реИрдХреНрд╕реАрди рд╕реЗ рдиреНрдпреВрд░реЛрд▓реЙрдЬрд┐рдХрд▓ рдЬрдЯрд┐рд▓рддрд╛рдУрдВ рдХреЛ рдкреВрд░реА рддрд░рд╣ рд╕реЗ рд░реЛрдХрдирд╛ рдЕрднреА рддрдХ рд╕рдВрднрд╡ рдирд╣реАрдВ рд╣реИ, рд▓реЗрдХрд┐рди рдпрд╣ рдзреНрдпрд╛рди рджрд┐рдпрд╛ рдЬрд╛рдирд╛ рдЪрд╛рд╣рд┐рдП рдХрд┐ рдкрд░реНрдЯреБрд╕рд┐рд╕ рд╕реЗ рдорд╕реНрддрд┐рд╖реНрдХ рдХреНрд╖рддрд┐ рдХрд╛ рдЬреЛрдЦрд┐рдо рдЯреАрдХреЗ рдХреА рддреБрд▓рдирд╛ рдореЗрдВ 3000 рдЧреБрдирд╛ рдЕрдзрд┐рдХ рд╣реИред

рдЖрдкрдХреЛ рдХрдм рдЪрд┐рдВрддрд╛ рдирд╣реАрдВ рдХрд░рдиреА рдЪрд╛рд╣рд┐рдП?

рдбреАрдкреАрдЯреА рдЯреАрдХрд╛рдХрд░рдг рдХреЗ рдмрд╛рдж рд╡рд┐рд╢рд┐рд╖реНрдЯ рджреБрд╖реНрдкреНрд░рднрд╛рд╡:

  1. рдмрдЪреНрдЪрд╛ рд▓рдВрдЧрдбрд╝рд╛ рд╣реИ (рдкреИрд░ рдореЗрдВ рджрд░реНрдж рд╣реЛрддрд╛ рд╣реИ):
  2. рдбреАрдкреАрдЯреА рдЯреАрдХрд╛рдХрд░рдг рдХреЗ рдмрд╛рдж, рдЗрдВрдЬреЗрдХреНрд╢рди рд╕реНрдерд▓ рдкрд░ рд╕реВрдЬрди рдХрд╛ рдЧрдарди;
  3. рдкреИрд░ рд╕реВрдЬ рдЧрдпрд╛ рд╣реИ;
  4. рдПрдХ рднрдбрд╝рдХрд╛рдК рдкреНрд░рдХреНрд░рд┐рдпрд╛ рд╣реЛрддреА рд╣реИ;
  5. рдЬреБрдХрд╛рдо рдиреЗ рдЬрдХрдбрд╝рд╛;
  6. рдбреАрдкреАрдЯреА рдХреЗ рдмрд╛рдж, рдЖрдкрдХреЛ рдЦрд╛рдВрд╕реА рдпрд╛ рдХрд╛рд▓реА рдЦрд╛рдВрд╕реА рднреА рд╣реЛрддреА рд╣реИ;
  7. рдЯреАрдХрд╛рдХрд░рдг рдХреЗ рдмрд╛рдж рдПрдХ рдлреЛрдбрд╝рд╛ рдерд╛;
  8. рдмрдЪреНрдЪрд╛ рдЪрд┐рдХрдирдкреЙрдХреНрд╕ рд╕реЗ рдмреАрдорд╛рд░ рд╣реЛ рдЧрдпрд╛ред
  9. рдбреАрдкреАрдЯреА рдЯреАрдХрд╛рдХрд░рдг рдХреЗ рдмрд╛рдж рдПрдХ рдмрдЪреНрдЪрд╛ рдмрд╣реБрдд рдореВрдбреА рдФрд░ рдиреАрдВрдж рд╕реЗ рднрд░рд╛ рд╣реЛ рд╕рдХрддрд╛ рд╣реИред

рдЖрдЗрдП рдЗрд╕реЗ рдХреНрд░рдо рд╕реЗ рд╕рдордЭреЗрдВ . рдмрдЪреНрдЪрд╛ рд╡рд╛рд╕реНрддрд╡ рдореЗрдВ рдкреИрд░ рдкрд░ рд▓рдВрдЧрдбрд╝рд╛ рд╕рдХрддрд╛ рд╣реИ , рдХреНрдпреЛрдВрдХрд┐ рдЗрдВрдЬреЗрдХреНрд╢рди рд╕реНрдерд▓ рдкрд░ рд╕реВрдЬрди рдмрдирдиреЗ рдХреЗ рдмрд╛рдж, рдкреИрд░ рдереЛрдбрд╝рд╛ рд╕реВрдЬ рд╕рдХрддрд╛ рд╣реИред рд╣рд▓реНрдХреА рд╕реВрдЬрди рдкреНрд░рдХреНрд░рд┐рдпрд╛ рднреА рд╣реЛ рд╕рдХрддреА рд╣реИред

рд╕рд░реНрджреА рд╡рд╛рд╕реНрддрд╡ рдореЗрдВ рдбреАрдкреАрдЯреА рдХреЗ рдХрд╛рд░рдг рд╣реЛ рд╕рдХрддреА рд╣реИ, рдХреНрдпреЛрдВрдХрд┐ рдЗрд╕рдХреЗ рдкрд░рд┐рдЪрдп рдХреЗ рдмрд╛рдж, рдкреНрд░рддрд┐рд░рдХреНрд╖рд╛ рдкреНрд░рдгрд╛рд▓реА рдХрдордЬреЛрд░ рд╣реЛ рд╕рдХрддреА рд╣реИ рдФрд░ рдмрдЪреНрдЪреЗ рдХреЛ рд╕рд░реНрджреА рд▓рдЧрдиреЗ рдХреА рд╕рдВрднрд╛рд╡рдирд╛ рдЕрдзрд┐рдХ рд╣реЛрддреА рд╣реИред рдЗрд╕реАрд▓рд┐рдП рдпрджрд┐ рдЖрдкрдХрд╛ рдмрдЪреНрдЪрд╛ рдбреАрдкреАрдЯреА рдХреЗ рдЯреАрдХреЗ рд▓рдЧрдиреЗ рдХреЗ рдмрд╛рдж рдмреАрдорд╛рд░ рд╣реЛ рдЬрд╛рддрд╛ рд╣реИ, рддреЛ рдпрд╣ рдЖрджрд░реНрд╢ рдореЗрдВ рдлрд┐рдЯ рдмреИрдарддрд╛ рд╣реИред

рдЪреВрдВрдХрд┐ рдЯреАрдХреЗ 100% рдмреАрдорд╛рд░реА рд╕реЗ рд░рдХреНрд╖рд╛ рдирд╣реАрдВ рдХрд░ рд╕рдХрддреЗ (рд▓реЗрдХрд┐рди рдХреЗрд╡рд▓ 95 .)-98%), рддреЛ рдЯреАрдХрд╛рдХрд░рдг рдХреЗ рдмрд╛рдж рдХрд╛рд▓реА рдЦрд╛рдВрд╕реА рдХреА рдЙрдкрд╕реНрдерд┐рддрд┐ рд╡рд┐рдХрд╕рд┐рдд рд░реЛрдЧ рдХреЗ рдкреНрд░рддрд┐ рдПрдВрдЯреАрдмреЙрдбреА рдХреА рдЕрдиреБрдкрд╕реНрдерд┐рддрд┐ рдХрд╛ рд╕рдВрдХреЗрдд рджреЗ рд╕рдХрддреА рд╣реИред

рдЖрдЬрдХрд▓, рдбреАрдкреАрдЯреА рдЯреАрдХрд╛рдХрд░рдг рдХреЗ рдмрд╛рдж рдлреЛрдбрд╝реЗ рдХреЗ рдЧрдарди рдХреЛ рдкреВрд░рд╛ рдХрд░рдирд╛ рд▓рдЧрднрдЧ рдЕрд╕рдВрднрд╡ рд╣реИред рдЕрдм рдпрд╣ рдмрдЪреНрдЪреЗ рдХреЛ рдЬрд╛рдВрдШ рдореЗрдВ, рдореБрдЦреНрдпрддрдГ рдЙрд╕рдХреЗ рд╕рд╛рдордиреЗ рдХреЗ рднрд╛рдЧ рдореЗрдВ рдХрд┐рдпрд╛ рдЬрд╛рддрд╛ рд╣реИред рдЗрд╕рд▓рд┐рдП рдХрдИ рдорд╛рддрд╛рдУрдВ рдХреА рд╢рд┐рдХрд╛рдпрдд рд╣реЛрддреА рд╣реИ рдХрд┐ рдбреАрдкреАрдЯреА рдХреЗ рдмрд╛рдж рдЙрдирдХреЗ рдкреИрд░ рдореЗрдВ рд╕реВрдЬрди рдЖ рдЬрд╛рддреА рд╣реИред рд▓реЗрдХрд┐рди рдлреЛрдбрд╝рд╛ рдирд╣реАрдВ рдЙрдард╛, рдЬрд┐рд╕рдХрд╛ рд╕рд╛рдордирд╛ рдкрд┐рдЫрд▓реА рдкреАрдврд╝рд┐рдпреЛрдВ рдХреЛ рдЕрдХреНрд╕рд░ рдХрд░рдирд╛ рдкрдбрд╝рддрд╛ рдерд╛!

рдбреАрдкреАрдЯреА рдХреЗ рдмрд╛рдж рдлреЛрдбрд╝рд╛ рдмрд╣реБрдд рдЖрдо рдерд╛ рдЬрдм рдЧреНрд▓реВрдЯрд╕ рдореИрдХреНрд╕рд┐рдорд╕ рдХреЛ рдЯреАрдХрд╛ рд▓рдЧрд╛рдпрд╛ рдЧрдпрд╛ рдерд╛ , рдЬрд╣рд╛рдВ рдмрдЪреНрдЪреЗ рдореЗрдВ рдмрд╣реБрдд рдЕрдзрд┐рдХ рд╡рд╕рд╛ рдЬрдорд╛ рд╣реЛрддреА рд╣реИред рдкрд╛рдВрдЪрд╡реЗрдВ рдмрд┐рдВрджреБ рддрдХ рдЧрд┐рд░рдиреЗ рдХреА рд╕реБрд╡рд┐рдзрд╛ рдХреЗ рд▓рд┐рдП рдмрдЪреНрдЪреЗ рдХреЗ рд▓рд┐рдП рдЙрдирдХреА рдЙрдкрд╕реНрдерд┐рддрд┐ рдмрд╕ рдЖрд╡рд╢реНрдпрдХ рд╣реИред рдЗрди рдлреИрдЯреА рдЬрдорд╛рдУрдВ рдореЗрдВ, рдкрджрд╛рд░реНрде рдкреВрд░реА рддрд░рд╣ рд╕реЗ рднрдВрдЧ рдирд╣реАрдВ рд╣реЛ рд╕рдХрддрд╛ рд╣реИ рдФрд░ рдмрд╕ рд╕реНрдерд┐рд░ рд╣реЛ рдЬрд╛рддрд╛ рд╣реИ, рдЬрд┐рд╕рд╕реЗ рд╕рдВрднрд╛рд╡рд┐рдд рдлреЛрдбрд╝рд╛ рд╣реЛ рдЬрд╛рддрд╛ рд╣реИ рдФрд░ рдХреЛрдИ рд▓рд╛рдн рдирд╣реАрдВ рд╣реЛрддрд╛ рд╣реИред

рдбреАрдкреАрдЯреА рдХреЗ рдмрд╛рдж рдЪрд┐рдХрдирдкреЙрдХреНрд╕ рдЯреАрдХрд╛рдХрд░рдг рдХреЗ рдХрд╛рд░рдг рдирд╣реАрдВ рд╣реЛ рд╕рдХрддрд╛ рд╣реИ рд▓реЗрдХрд┐рди рдЕрдЧрд░ рдЗрдВрдЬреЗрдХреНрд╢рди рдХреЗ рдмрд╛рдж рдмрдЪреНрдЪрд╛ рдмреАрдорд╛рд░ рд╣реЛ рдЬрд╛рддрд╛ рд╣реИ, рддреЛ рдЖрдкрдХреЛ рдирд┐рд╢реНрдЪрд┐рдд рд░реВрдк рд╕реЗ рдФрд░ рдЬрд▓реНрдж рд╕реЗ рдЬрд▓реНрдж рдбреЙрдХреНрдЯрд░ рд╕реЗ рд╕рд▓рд╛рд╣ рд▓реЗрдиреА рдЪрд╛рд╣рд┐рдПред рдХреНрдпреЛрдВрдХрд┐ рдбреАрдкреАрдЯреА рдЯреАрдХрд╛рдХрд░рдг рдХреЗ рдмрд╛рдж рд░реЛрдЧ рдкреНрд░рддрд┐рд░реЛрдзрдХ рдХреНрд╖рдорддрд╛ рдХрд╛ рдХрдордЬреЛрд░ рд╣реЛрдирд╛ рдЪрд┐рдХрдирдкреЙрдХреНрд╕ рд░реЛрдЧ рдХрд╛ рдПрдХ рдЧрдВрднреАрд░ рдХреЛрд░реНрд╕ рджреЗ рд╕рдХрддрд╛ рд╣реИред

рд╣рд╛рд▓рд╛рдБрдХрд┐, рдЖрдкрдХреЛ рддреБрд░рдВрдд рдЕрдкрдиреЗ рдмрд╛рд▓ рд░реЛрдЧ рд╡рд┐рд╢реЗрд╖рдЬреНрдЮ рд╕реЗ рд╕рдВрдкрд░реНрдХ рдХрд░рдирд╛ рдЪрд╛рд╣рд┐рдП рдпрджрд┐:

┬╖ 1 рдмрдЪреНрдЪреЗ рдХреЗ рд╢рд░реАрд░ рдХрд╛ рддрд╛рдкрдорд╛рди 39 рдбрд┐рдЧреНрд░реА рд╕реЗрд▓реНрд╕рд┐рдпрд╕ рдХреЗ рдХрд░реАрдм рд╣реИ;

┬╖ 2 рдЗрдВрдЬреЗрдХреНрд╢рди рд╕рд╛рдЗрдЯ рдЧрдВрднреАрд░ рд░реВрдк рд╕реЗ рд╕реВрдЬ рдЧрдИ рд╣реИ (рдкрд░рд┐рдзрд┐ рдореЗрдВ 8-10 рд╕реЗрдореА рд╕реЗ рдЕрдзрд┐рдХ);

┬╖ 3 рдмрдЪреНрдЪрд╛ 3 рдШрдВрдЯреЗ рд╕реЗ рдЕрдзрд┐рдХ рд╕рдордп рддрдХ рдЬреЛрд░ рд╕реЗ рдФрд░ рд▓рдЧрд╛рддрд╛рд░ рд░реЛрддрд╛ рд╣реИ (рдирд┐рд░реНрдЬрд▓реАрдХрд░рдг рдХрд╛ рдЦрддрд░рд╛ рд╣реЛрддрд╛ рд╣реИ)ред

рдбреАрдкреАрдЯреА рдЯреАрдХрд╛рдХрд░рдг рдХреЗ рдмрд╛рдж рддрд╛рдкрдорд╛рдиредрдЖрдорддреМрд░ рдкрд░, рджреБрдирд┐рдпрд╛ рднрд░ рдХреЗ рдмрд╛рд▓ рд░реЛрдЧ рд╡рд┐рд╢реЗрд╖рдЬреНрдЮ рдЗрд╕ рдмрд╛рдд рдкрд░ рдЬреЛрд░ рджреЗрддреЗ рд╣реИрдВ рдХрд┐ рдПрдХ рд╢рд┐рд╢реБ рдореЗрдВ рдмрдврд╝рд╛ рд╣реБрдЖ рддрд╛рдкрдорд╛рди, 38 рдбрд┐рдЧреНрд░реА рд╕реЗрд▓реНрд╕рд┐рдпрд╕ рд╕реЗ рдЕрдзрд┐рдХ рдирд╣реАрдВ, рдиреАрдЪреЗ рдирд╣реАрдВ рд▓рд╛рдпрд╛ рдЬрд╛рдирд╛ рдЪрд╛рд╣рд┐рдПред рд╣рд╛рд▓рд╛рдВрдХрд┐, рдЯреАрдХрд╛рдХрд░рдг рдХреА рд╢рд░реНрддреЛрдВ рдореЗрдВ рдпрд╣ рдирд┐рдпрдо "рд░рджреНрдж" рдХрд┐рдпрд╛ рдЬрд╛рддрд╛ рд╣реИ - рдпрджрд┐ рдЯреАрдХрд╛рдХрд░рдг рдХреЗ рдмрд╛рдж рдбреАрдкреАрдЯреА рддрд╛рдкрдорд╛рдирдмрдврд╝рдирд╛ рд╢реБрд░реВ рд╣реЛ рдЧрдпрд╛, рддреБрд░рдВрдд рдмрдЪреНрдЪреЗ рдХреЛ рдПрдХ рдЬреНрд╡рд░рдирд╛рд╢рдХ рдПрдЬреЗрдВрдЯ рджреЗрдВред

рдЗрдВрдЬреЗрдХреНрд╢рди рд╕реНрдерд▓ рдкрд░ рдЧрд╛рдВрда, рд╕реВрдЬрди рдФрд░ рд╕реВрдЬрдиредрдпрджрд┐ рд╕реНрд╡рд╛рд╕реНрдереНрдп рдХрд╛рд░реНрдпрдХрд░реНрддрд╛ рдЕрдиреБрднрд╡реА рдФрд░ "рдЖрд╕рд╛рди" рд╣реИ, рддреЛ рдЗрдВрдЬреЗрдХреНрд╢рди рдХреЗ рдмрд╛рдж рдХреЛрдИ рдмрдбрд╝реА рдЧрд╛рдВрда рдФрд░ рд╕реВрдЬрди рдирд╣реАрдВ рд╣реЛрдЧреАред рд▓реЗрдХрд┐рди рдРрд╕рд╛ рднреА рд╣реЛрддрд╛ рд╣реИ рдХрд┐ рджрд╡рд╛ рдЬрд╛рдВрдШ рдХреА рдорд╛рдВрд╕рдкреЗрд╢рд┐рдпреЛрдВ рдХреЗ рдКрддрдХреЛрдВ рдореЗрдВ рдирд╣реАрдВ, рдмрд▓реНрдХрд┐ рдЪрдордбрд╝реЗ рдХреЗ рдиреАрдЪреЗ рдХреЗ рд╡рд╕рд╛рдпреБрдХреНрдд рдКрддрдХ рдореЗрдВ рдорд┐рд▓рддреА рд╣реИред рдЗрд╕ рдорд╛рдорд▓реЗ рдореЗрдВ, рд╕рдмрд╕реЗ рдЕрдзрд┐рдХ рдмрд╛рд░, рдПрдХ рдордЬрдмреВрдд рдЕрд╡рдзрд┐ рдФрд░ рд╕реВрдЬрди рдХрд╛ рдЧрдарди рд╣реЛрддрд╛ рд╣реИред рдЕрдкрдиреЗ рдЪрд┐рдХрд┐рддреНрд╕рдХ рдХреЛ рдЗрдВрдЬреЗрдХреНрд╢рди рд╕рд╛рдЗрдЯ рджрд┐рдЦрд╛рдПрдВ - рд╡рд╣ рд╕реБрд░рдХреНрд╖рд┐рдд рджрд╡рд╛рдПрдВ рд▓рд┐рдЦреЗрдВрдЧреЗ рдЬреЛ рд░рдХреНрдд рдкрд░рд┐рд╕рдВрдЪрд░рдг рдХреЛ рдмрдврд╝рд╛рдПрдЧреА рдФрд░ рд╕реВрдЬрди рдХреЛ рдХрдо рдХрд░реЗрдЧреАред

рд╣рд╛рд▓рд╛рдВрдХрд┐, рдорд╛рдореВрд▓реА рд╕реВрдЬрди рдХреЗ рд╕рд╛рде рдЗрдВрдЬреЗрдХреНрд╢рди рд╕реНрдерд▓ рдкрд░ рдЧрдВрднреАрд░ рд╕реВрдЬрди рдФрд░ рд╕реВрдЬрди рдХреЛ рднреНрд░рдорд┐рдд рди рдХрд░реЗрдВред рд╕реНрд╡рд╛рднрд╛рд╡рд┐рдХ рд░реВрдк рд╕реЗ, рдЬрдм рджрд╡рд╛ рдХреЛ рдорд╛рдВрд╕рдкреЗрд╢рд┐рдпреЛрдВ рдореЗрдВ рдЗрдВрдЬреЗрдХреНрдЯ рдХрд┐рдпрд╛ рдЬрд╛рддрд╛ рд╣реИ, рдЬрд┐рд╕рдореЗрдВ рдХрдордЬреЛрд░ рд╕рдВрдХреНрд░рд╛рдордХ рдХреЛрд╢рд┐рдХрд╛рдПрдВ рд╣реЛрддреА рд╣реИрдВ, рддреЛ рдЗрдВрдЬреЗрдХреНрд╢рди рд╕реНрдерд▓ рдкрд░ рд╣рд▓реНрдХреА рд╕реНрдерд╛рдиреАрдп рд╕реВрдЬрди рдХреА рдПрдХ рд╕рд╛рдорд╛рдиреНрдп рдкреНрд░рдХреНрд░рд┐рдпрд╛ рд╣реЛрддреА рд╣реИред рдпрд╣ 1-2 рд╕рдкреНрддрд╛рд╣ рдХреЗ рдмрд╛рдж рдмрдЪреНрдЪреЗ рдХреЗ рд╕реНрд╡рд╛рд╕реНрдереНрдп рдХреЛ рдХреЛрдИ рдиреБрдХрд╕рд╛рди рдкрд╣реБрдВрдЪрд╛рдП рдмрд┐рдирд╛ рдЕрдкрдиреЗ рдЖрдк рдЪрд▓реА рдЬрд╛рдПрдЧреАред

рдЗрдВрдЬреЗрдХреНрд╢рди рд╕реНрдерд▓ рдкрд░ рддреНрд╡рдЪрд╛ рдХрд╛ рд▓рд╛рд▓ рд╣реЛрдирд╛ рдФрд░ рдЦреБрдЬрд▓реА рд╣реЛрдирд╛редрд╣рд▓реНрдХреА рд▓рд╛рд▓рд┐рдорд╛ (рдЗрдВрдЬреЗрдХреНрд╢рди рд╕реНрдерд▓ рдкрд░ рд▓рдЧрднрдЧ 2-4 рд╕реЗрдореА рдХреА рддреНрд░рд┐рдЬреНрдпрд╛) рднреА рд╕рд╛рдорд╛рдиреНрдп рд╣реИ, рдЬреЛ рдЯреАрдХреЗ рдХреЗ рдкреНрд░рд╢рд╛рд╕рди рдХреЗ рдХрд╛рд░рдг рд╣реЛрдиреЗ рд╡рд╛рд▓реА рд╣рд▓реНрдХреА рд╕реНрдерд╛рдиреАрдп рд╕реВрдЬрди рд╕реЗ рдХрд╛рдлреА рд╕реНрдкрд╖реНрдЯ рд╣реИред рдпрджрд┐, рдЗрдВрдЬреЗрдХреНрд╢рди рд╕реНрдерд▓ рдкрд░ рд▓рд╛рд▓рд┐рдорд╛ рдХреЗ рдЕрд▓рд╛рд╡рд╛, рдЖрдк рдХреЛрдИ рдЕрдиреНрдп "рдкрд░реЗрд╢рд╛рдиреА" рдирд╣реАрдВ рджреЗрдЦрддреЗ рд╣реИрдВ - рдХреБрдЫ рднреА рди рдХрд░реЗрдВред рд▓рдЧрднрдЧ 8-10 рджрд┐рдиреЛрдВ рдХреЗ рднреАрддрд░ рд▓рд╛рд▓реА рдзреАрд░реЗ-рдзреАрд░реЗ рдЧрд╛рдпрдм рд╣реЛ рдЬрд╛рдПрдЧреАред

рдмрдЪреНрдЪреЗ рдХреЗ рд▓рд┐рдП рдЯреАрдХреЗ рдХреЛ рд╕рд╣рди рдХрд░рдирд╛ рдХрд┐рддрдирд╛ рдХрдард┐рди рд╣реИ, рдЗрд╕ рдкрд░ рдирд┐рд░реНрднрд░ рдХрд░рддреЗ рд╣реБрдП, рд▓рдХреНрд╖рдгреЛрдВ рдХреЛ рдХрдо рдХрд░рдиреЗ рдХреЗ рд▓рд┐рдП рдХрдИ рд╡рд┐рдХрд▓реНрдк рд╣реИрдВ:

  • рдЯреАрдХрд╛рдХрд░рдг рдХреЗ рдмрд╛рдж рдорд╛рд▓рд┐рд╢ рдХрд░реЗрдВ;
  • рдПрдХ рд╕рдВрдкреАрдбрд╝рд┐рдд рд▓рд╛рдЧреВ рдХрд░реЗрдВ;
  • рд╡рд┐рд╢реЗрд╖ рдорд▓рд╣рдо рдХрд╛ рдкреНрд░рдпреЛрдЧ рдХрд░реЗрдВред

рдЗрд╕ рддрд░рд╣ рдХреА рдорджрдж рдХреЗ рд▓рд┐рдП рдирд┐рд╢реНрдЪрд┐рдд рд░реВрдк рд╕реЗ рдбреЙрдХреНрдЯрд░ рдХреЗ рдкрд░рд╛рдорд░реНрд╢ рдХреА рдЖрд╡рд╢реНрдпрдХрддрд╛ рд╣реЛрддреА рд╣реИ, рдХреНрдпреЛрдВрдХрд┐ рдпрд╣ рд╕рдм рдкреНрд░рдХрдЯ рд╣реЛрдиреЗ рд╡рд╛рд▓реА рдкреНрд░рддрд┐рдХреНрд░рд┐рдпрд╛ рдХреА рдЧрдВрднреАрд░рддрд╛ рдФрд░ рдЙрд╕рдХреЗ рдкреНрд░рдХрд╛рд░ рдкрд░ рдирд┐рд░реНрднрд░ рдХрд░рддрд╛ рд╣реИред рд▓реЗрдХрд┐рди, рдЬреЛ рдмрд┐рд▓реНрдХреБрд▓ рдирд╣реАрдВ рдХрд┐рдпрд╛ рдЬрд╛ рд╕рдХрддрд╛ рд╣реИ рд╡рд╣ рд╣реИ рдЕрд▓реНрдХреЛрд╣рд▓ рдХрдВрдкреНрд░реЗрд╕ рд▓рдЧрд╛рдирд╛ред

рдкрд░рд┐рдгрд╛рдореЛрдВ

  • рдЬреНрдпрд╛рджрд╛рддрд░ рдорд╛рдорд▓реЛрдВ рдореЗрдВ рдбреАрдЯреАрдкреА рдЯреАрдХрд╛рдХрд░рдг рдЬрд░реВрд░реА рд╣реИ, рд▓реЗрдХрд┐рди рд╣рд░ рдорд╛рддрд╛-рдкрд┐рддрд╛ рдЗрд╕реЗ рдордирд╛ рдХрд░ рд╕рдХрддреЗ рд╣реИрдВред
  • рдбреАрдкреАрдЯреА рд╕рд╣рди рдХрд░рдиреЗ рдХреЗ рд▓рд┐рдП рд╕рдмрд╕реЗ рдХрдард┐рди рдЯреАрдХрд╛ рд╣реИред
  • рд╕рднреА рдЯреАрдХреЛрдВ рд╕реЗ рдкрд╣рд▓реЗ, рдмрдЪреНрдЪреЗ рдХреЛ рдмрд┐рд▓реНрдХреБрд▓ рд╕реНрд╡рд╕реНрде рдФрд░ рд╡рд┐рднрд┐рдиреНрди рдПрд▓рд░реНрдЬреА рд╕реЗ рдореБрдХреНрдд рд╣реЛрдирд╛ рдЪрд╛рд╣рд┐рдПред
  • рдбреАрдкреАрдЯреА рдХреЗ рдмрд╛рдж рд╣реЛрдиреЗ рд╡рд╛рд▓реЗ рджреБрд╖реНрдкреНрд░рднрд╛рд╡ рдФрд░ рдЬрдЯрд┐рд▓рддрд╛рдПрдВ рд╕реНрдерд╛рдиреАрдп рдФрд░ рд╕рд╛рдорд╛рдиреНрдп рд╣реИрдВред рд╕рдмрд╕реЗ рдХрдард┐рди рдЬрдЯрд┐рд▓рддрд╛рдПрдВ рдПрд▓рд░реНрдЬреА рдкреНрд░рддрд┐рдХреНрд░рд┐рдпрд╛рдУрдВ рдХреЗ рд░реВрдк рдореЗрдВ рд╣реЛрддреА рд╣реИрдВред
  • рдЖрдк рдорд╛рд▓рд┐рд╢, рд▓реЛрд╢рди рд▓рдЧрд╛рдиреЗ рдпрд╛ рдХрд┐рд╕реА рд╡рд┐рд╢реЗрд╖ рдорд▓рд╣рдо рд╕реЗ рдЕрднрд┐рд╖реЗрдХ рдХрд░рдиреЗ рдХреЗ рдмрд╛рдж рдЯреАрдХрд╛рдХрд░рдг рдХреЗ рдмрд╛рдж рджрд┐рдЦрд╛рдИ рджреЗрдиреЗ рд╡рд╛рд▓реЗ рд▓рдХреНрд╖рдгреЛрдВ рдХреЛ рджреВрд░ рдХрд░рдиреЗ рдореЗрдВ рдЕрдкрдиреЗ рдмрдЪреНрдЪреЗ рдХреА рдорджрдж рдХрд░ рд╕рдХрддреА рд╣реИрдВред рд▓реЗрдХрд┐рди рдРрд╕реА рдХреНрд░рд┐рдпрд╛рдПрдВ рдХреЗрд╡рд▓ рдбреЙрдХреНрдЯрд░ рдХреА рд╕рд┐рдлрд╛рд░рд┐рд╢ рдкрд░ рдФрд░ рдЙрдирдХреА рджреЗрдЦрд░реЗрдЦ рдореЗрдВ рд╣реА рдХреА рдЬрд╛рдиреА рдЪрд╛рд╣рд┐рдПред
  • рдкрд╣рд▓реЗ рддреАрди рдбреАрдЯреАрдкреА рдЯреАрдХрд╛рдХрд░рдг рдкрд╣рд▓реЗ рд╡рд░реНрд╖ рдХреЗ рд▓рд┐рдП рдкреНрд░рддрд┐рд░рдХреНрд╖рд╛ рд╕рдорд░реНрдерди рдкреНрд░рджрд╛рди рдХрд░рддреЗ рд╣реИрдВред рдбреЗрдврд╝ рд╕рд╛рд▓ рдХреА рдЙрдореНрд░ рдореЗрдВ рдЖрдЧреЗ рдЯреАрдХрд╛рдХрд░рдг рдЗрди рдмреАрдорд╛рд░рд┐рдпреЛрдВ (рдХрд╛рд▓реА рдЦрд╛рдВрд╕реА, рдбрд┐рдкреНрдереАрд░рд┐рдпрд╛ рдФрд░ рдЯреЗрдЯрдирд╕) рдХреЗ рдкреНрд░рддрд┐ рдкреНрд░рддрд┐рд░реЛрдзрдХ рдХреНрд╖рдорддрд╛ рдХреЛ 6-8 рд╕рд╛рд▓ рддрдХ рд╕реБрдирд┐рд╢реНрдЪрд┐рдд рдХрд░рддрд╛ рд╣реИред

рдорд╛рд░реНрдЪ 20, 2012 10:40 рдЕрдкрд░рд╛рд╣реНрди

рд▓рдбрд╝рдХрд┐рдпреЛрдВ рдиреЗ рдХрд▓, рдореИрдВрдиреЗ рдПрдХ рдкреЛрд╕реНрдЯ рд▓рд┐рдЦрд╛ рдерд╛ рдХрд┐ рдЙрдиреНрд╣реЗрдВ рдбреАрдкреАрдЯреА рдХреИрд╕реЗ рдорд┐рд▓рд╛, рдЙрдиреНрд╣реЛрдВрдиреЗ рдбреАрдкреАрдЯреА рдХреЗ рдмрд╛рд░реЗ рдореЗрдВ рдЬрд╛рдирдХрд╛рд░реА рдХреЛ рдЬрдВрдЧрд▓ рдореЗрдВ рдлреЗрдВрдХ рджрд┐рдпрд╛, рдкрдврд╝рд╛ рд╢рд╛рдпрдж рдХрд┐рд╕реА рдХреЛ рдбреАрдкреАрдЯреА рд╡реИрдХреНрд╕реАрди рдХреА рдЖрд╡рд╢реНрдпрдХрддрд╛ рд╣реЛрдЧреА рдпрд╛ рдбреАрдкреАрдЯреА рдмрдЪреНрдЪреЛрдВ рдХреЗ рд╡рд┐рдХрд╛рд╕ рдХреЛ рд░реЛрдХрдирд╛ - рдЯреАрдХреЛрдВ рдХрд╛ рд╕рдмрд╕реЗ рднрдпрд╛рдирдХред рдбрд┐рдкреНрдереАрд░рд┐рдпрд╛, рдкрд░реНрдЯреБрд╕рд┐рд╕ рдФрд░ рдЯреЗрдЯрдирд╕ рдХреЛрд╢рд┐рдХрд╛рдУрдВ рдХрд╛ рдПрдХ рд╡рд┐рд╕реНрдлреЛрдЯрдХ рдорд┐рд╢реНрд░рдгред рдФрд░ рдЗрд╕ рдЕрджреНрднреБрдд рдЯреАрдХреЗ рдХреЛ рддреАрди рдорд╣реАрдиреЗ рдХреА рдЙрдореНрд░ рд╕реЗ рд╢рд┐рд╢реБ рдХреЗ рд╢рд░реАрд░ рдореЗрдВ рдЪрд╛рд░ рдмрд╛рд░ рдЗрдВрдЬреЗрдХреНрдЯ рдХрд┐рдпрд╛ рдЬрд╛рддрд╛ рд╣реИред рдпрд╣ рдПрдХ рдмрд╣реБрдд рд╣реА рджрд░реНрджрдирд╛рдХ рдЯреАрдХрд╛рдХрд░рдг рд╣реИ, рдФрд░ рдХреБрдЫ рдмрдЪреНрдЪреЗ рд▓рдВрдмреЗ, рдирд┐рд░рдВрддрд░ рд░реЛрдиреЗ рдХреЗ рд╕рд╛рде рдЗрд╕ рдкрд░ рдкреНрд░рддрд┐рдХреНрд░рд┐рдпрд╛ рдХрд░рддреЗ рд╣реИрдВред рдбреАрдкреАрдЯреА рдореЗрдВ рд╕рдмрд╕реЗ рдЕрдзрд┐рдХ рдЬрдЯрд┐рд▓рддрд╛рдПрдВ рд╣реЛрддреА рд╣реИрдВ рдФрд░ рдмрдЪреНрдЪреЗ рдХреЗ рд╢рд░реАрд░ рдореЗрдВ рдПрд▓рд░реНрдЬреА рдХреА рдкреНрд░рддрд┐рдХреНрд░рд┐рдпрд╛ рдХреЗ рдЬреЛрдЦрд┐рдо рдХрд╛ рдкреНрд░рддрд┐рд╢рдд рдЕрдзрд┐рдХ рд╣реЛрддрд╛ рд╣реИред рдЗрд╕ рдЯреАрдХреЗ рдХреА рдЕрдВрддрд░рд╛рддреНрдорд╛ рдкрд░ - рдХрдИ рдмрдЪреНрдЪреЛрдВ рдХреА рдореМрдд, рдореБрдХрджрдореЗ, рдХрдИ рдмрд╛рд░ рдпреВрд░реЛрдкреАрдп рджреЗрд╢реЛрдВ рдореЗрдВ рдЗрд╕реЗ рдкреНрд░рддрд┐рдмрдВрдзрд┐рдд рдХрд┐рдпрд╛ рдЧрдпрд╛ рдерд╛, рд▓реЗрдХрд┐рди рд░реВрд╕ рдореЗрдВ рдирд╣реАрдВред рдЬрд╛рдкрд╛рди рдФрд░ рдпреВрд░реЛрдк рдиреЗ рдбреАрдкреАрдЯреА рдХреЛ рдЫреЛрдбрд╝ рджрд┐рдпрд╛ рд╣реИ рд╕рддреНрддрд░ рдХреЗ рджрд╢рдХ рдХреА рд╢реБрд░реБрдЖрдд рддрдХ, рдЬрд╛рдкрд╛рди рдореЗрдВ 37 рдмрдЪреНрдЪреЛрдВ рдХреА рдбреАрдкреАрдЯреА рд╕реЗ рдореГрддреНрдпреБ рд╣реЛ рдЧрдИ рдереАред рдЬрд╛рдкрд╛рдирд┐рдпреЛрдВ рдиреЗ рдЕрдкрдиреЗ рдмрдЪреНрдЪреЛрдВ рдХреЛ рдпрд╣ рдЯреАрдХрд╛ рджреЗрдирд╛ рдмрдВрдж рдХрд░ рджрд┐рдпрд╛, рдлрд┐рд░ рдЗрд╕реЗ рд╢реИрд╢рд╡рд╛рд╡рд╕реНрдерд╛ рд╕реЗ 2 рд╡рд░реНрд╖ рдХреА рдЖрдпреБ рдореЗрдВ рд╕реНрдерд╛рдирд╛рдВрддрд░рд┐рдд рдХрд░ рджрд┐рдпрд╛ред рдирддреАрдЬрддрди, рдмрд╛рд▓ рдореГрддреНрдпреБ рджрд░ рдХреЗ рдорд╛рдорд▓реЗ рдореЗрдВ рдЬрд╛рдкрд╛рди рджреБрдирд┐рдпрд╛ рдореЗрдВ 17 рд╡реЗрдВ рд╕реНрдерд╛рди рд╕реЗ рдирд╛рдЯрдХреАрдп рд░реВрдк рд╕реЗ рдЕрдВрддрд┐рдо рд╕реНрдерд╛рди рдкрд░ рд╕реНрдерд╛рдирд╛рдВрддрд░рд┐рдд рд╣реЛ рдЧрдпрд╛ рдерд╛ред 1980 рдХреЗ рджрд╢рдХ рдореЗрдВ, рдЙрдиреНрд╣реЛрдВрдиреЗ рдПрдХ рдирдП рдЕрдХреЛрд╢рд┐рдХреАрдп рдЯреАрдХреЗ рдХреЗ рд╕рд╛рде рдкрд░реНрдЯреБрд╕рд┐рд╕ рдХреЗ рдЦрд┐рд▓рд╛рдл рдЯреАрдХрд╛рдХрд░рдг рд╢реБрд░реВ рдХрд┐рдпрд╛, рдЬрд┐рд╕рдХреЗ рдХрд╛рд░рдг рдЕрдЧрд▓реЗ 10-12 рд╡рд░реНрд╖реЛрдВ рдореЗрдВ рдЕрдЪрд╛рдирдХ рд╢рд┐рд╢реБ рдореГрддреНрдпреБ рд╕рд┐рдВрдбреНрд░реЛрдо рдореЗрдВ рдЪрд╛рд░ рдЧреБрдирд╛ рд╡реГрджреНрдзрд┐ рд╣реБрдИред рдЗрд╕реА рддрд░рд╣ рдХреА рд╕реНрдерд┐рддрд┐ рдЗрдВрдЧреНрд▓реИрдВрдб, рдЬрд░реНрдордиреА, рд╣реЙрд▓реИрдВрдб рдореЗрдВ рд╣реБрдИред рдкрд░реНрдЯреБрд╕рд┐рд╕ рдХреЗ рдЯреАрдХрд╛рдХрд░рдг рдиреЗ рджрд░реНрдЬрдиреЛрдВ рдмрдЪреНрдЪреЛрдВ рдХреЛ рдорд╛рд░ рдбрд╛рд▓рд╛ рдФрд░ рд╡рд┐рдХрд▓рд╛рдВрдЧ рдХрд░ рджрд┐рдпрд╛, рдЬрд┐рд╕рдХреЗ рдмрд╛рдж рдЖрдмрд╛рджреА рдиреЗ рдЗрд╕ рдЯреАрдХрд╛рдХрд░рдг рд╕реЗ рдЗрдирдХрд╛рд░ рдХрд░рдирд╛ рд╢реБрд░реВ рдХрд░ рджрд┐рдпрд╛ред рдЯреАрдХрд╛рдХрд░рдг рдХрд╡рд░реЗрдЬ рдореЗрдВ рдХрдореА рдХреЗ рд╕рд╛рде, рдЕрд╕реНрдкрддрд╛рд▓реЛрдВ рдХреА рдпрд╛рддреНрд░рд╛рдУрдВ рдХреА рд╕рдВрдЦреНрдпрд╛ рдореЗрдВ рддреЗрдЬреА рд╕реЗ рдХрдореА рдЖрдИ рд╣реИ, рдФрд░ рдЬрд╣рд╛рдВ рдЙрдиреНрд╣реЛрдВрдиреЗ рдЕрднреА рднреА рдЯреАрдХрд╛рдХрд░рдг рд╕реЗ рдЗрдирдХрд╛рд░ рдирд╣реАрдВ рдХрд┐рдпрд╛ рд╣реИ, рд╡рд╣рд╛рдВ рдмреАрдорд╛рд░рд┐рдпреЛрдВ рдХреА рд╕рдВрдЦреНрдпрд╛ рдореЗрдВ рд╡реГрджреНрдзрд┐ рджреЗрдЦреА рдЧрдИ рд╣реИ, рдпрд╛рдиреА рд╡реИрдХреНрд╕реАрди рдорд╣рд╛рдорд╛рд░реА рд╕реЗ рдирд╣реАрдВ рдмрдЪрд╛ рд╣реИред рдЗрд╕рдХрд╛ рдХреНрдпрд╛ рдорддрд▓рдм рд╣реИ? рддрдереНрдп рдпрд╣ рд╣реИ рдХрд┐ рдбреАрдкреАрдЯреА рдЯреАрдХрд╛ рдШрд╛рддрдХ рд░реВрдк рд╕реЗ рд╣рд╛рдирд┐рдХрд╛рд░рдХ рд╣реИ, рдФрд░ рд╕рдмрд╕реЗ рдЕрдЪреНрдЫрд╛, рдмрд╕ рдмреЗрдХрд╛рд░ рд╣реИ, рдФрд░ рдХрд┐рд╕реА рдХрд╛рд░рдг рд╕реЗ рдЯреАрдХрд╛рдХрд░рдг рдХреИрд▓реЗрдВрдбрд░ рдкрд░ рд░рд╣рддрд╛ рд╣реИ рдЬреЛ рдХреЗрд╡рд▓ рдЙрд╕рдХреЗ рд▓рд┐рдП рдлрд╛рдпрджреЗрдордВрдж рд╣реИ, рди рдХрд┐ рд▓реЛрдЧреЛрдВ рдХреЗ рд╣рд┐рдд рдореЗрдВред рдЗрд╕ рдЬрд╣рд░реАрд▓реЗ рдбреАрдкреАрдЯреА рд╡реИрдХреНрд╕реАрди рдХреЛ рд╡реИрдХреНрд╕реАрди рднреА рдирд╣реАрдВ рдХрд╣рд╛ рдЬрд╛рддрд╛ рд╣реИ, рдмрд▓реНрдХрд┐ рдПрдХ рд░рд╛рд╕рд╛рдпрдирд┐рдХ рдФрд░ рдЬреИрд╡рд┐рдХ рд╕рдореВрд╣ рд╣реЛрддрд╛ рд╣реИ, рдЬрд┐рд╕рдореЗрдВ рдХрдИ рд░рд╛рд╕рд╛рдпрдирд┐рдХ рдШрдЯрдХ рд╣реЛрддреЗ рд╣реИрдВ рдЬреЛ рдорд╛рдирд╡ рд╕реНрд╡рд╛рд╕реНрдереНрдп рдХреЗ рд▓рд┐рдП рд╣рд╛рдирд┐рдХрд╛рд░рдХ рд╣реЛрддреЗ рд╣реИрдВ, рдЬрд┐рд╕рд╕реЗ рддрдВрддреНрд░рд┐рдХрд╛ рддрдВрддреНрд░ рдореЗрдВ рдЕрдкрд░рд┐рд╡рд░реНрддрдиреАрдп рдкрд░рд┐рд╡рд░реНрддрди рд╣реЛрддреЗ рд╣реИрдВ, рдЧреБрд░реНрджреЗ рдФрд░ рдорд╕реНрддрд┐рд╖реНрдХ рдХреА рдХреЛрд╢рд┐рдХрд╛рдУрдВ рдХреЛ рдкреНрд░рднрд╛рд╡рд┐рдд рдХрд░рддреЗ рд╣реИрдВ, рдЬрд┐рд╕рд╕реЗ рдХреИрдВрд╕рд░ рд╣реЛ рдЬрд╛рддрд╛ рд╣реИред рдкреЗрдЯред рдпреЗ рд╕рднреА рдШрдЯрдХ рдбреАрдкреАрдЯреА рд╡реИрдХреНрд╕реАрди рдХреЛ рд╕рдмрд╕реЗ рдЦрддрд░рдирд╛рдХ рд╡реИрдХреНрд╕реАрди рдмрдирд╛рддреЗ рд╣реИрдВ рдЬрд┐рд╕рд╕реЗ рдмрдЪреНрдЪреЛрдВ рдореЗрдВ рдСрдЯрд┐рдЬреНрдо рдФрд░ рд▓рдХрд╡рд╛ рд╣реЛрддрд╛ рд╣реИред рдмрд╣реБрдд рд╕реЗ рд▓реЛрдЧ рдЗрд╕рдХреЗ рдмрд╛рд░реЗ рдореЗрдВ рдирд╣реАрдВ рдЬрд╛рдирддреЗ рд╣реИрдВ, рдФрд░ рдЦрддрд░реЗ рдХреЗ рдмрд╛рд░реЗ рдореЗрдВ рддрдм рддрдХ рдирд╣реАрдВ рдЬрд╛рдирддреЗ рдЬрдм рддрдХ рдХрд┐ рд╡реЗ рдЦреБрдж рдкрд░реЗрд╢рд╛рдиреА рдХрд╛ рд╕рд╛рдордирд╛ рди рдХрд░реЗрдВред рд╕рдВрдкреВрд░реНрдг рдкрд░реНрдЯреБрд╕рд┐рд╕ рдХреЛрд╢рд┐рдХрд╛рдУрдВ рдХреЗ рдЕрд▓рд╛рд╡рд╛, рдпрд╣ рджрд╡рд╛ рдмрд╣реБрдд рдЦрддрд░рдирд╛рдХ рд╣реИ, рдПрдХ рдХрд╛рд░реНрдмрдирд┐рдХ рдкрд╛рд░рд╛ рдХреАрдЯрдирд╛рд╢рдХ рдЬрд┐рд╕реЗ рдореЗрд░рдерд┐рдУрд▓реЗрдЯ рдпрд╛ рдерд┐рдпреЛрдорд░реНрд╕рд▓ рдХрд╣рд╛ рдЬрд╛рддрд╛ рд╣реИ, рдЬреЛ рдПрдХ рд╕рдВрд░рдХреНрд╖рдХ рдХреЗ рд░реВрдк рдореЗрдВ рдкреНрд░рдпреЛрдЧ рдХрд┐рдпрд╛ рдЬрд╛рддрд╛ рд╣реИ, рдФрд░ рдлреЙрд░реНрдорд▓рд╛рдбреЗрд╣рд╛рдЗрдб - рдпреЗ рд╕рднреА рдЬрд╣рд░ рдЯреАрдХреЗ рдХреА рдПрдХ рдЦреБрд░рд╛рдХ рдореЗрдВ рдЬрд╣рд░ рдХреЗ рд▓рд┐рдП рдкрд░реНрдпрд╛рдкреНрдд рдорд╛рддреНрд░рд╛ рдореЗрдВ рдореМрдЬреВрдж рд╣реЛрддреЗ рд╣реИрдВред рддрди рдЫреЛрдЯрд╛ рдЖрджрдореА! рд╣рдорд╛рд░реЗ рджреЗрд╢ рдореЗрдВ рдореЗрд░реНрдерд┐рдпреЛрд▓реЗрдЯ рдХреЛ рдПрдХ рджрд╡рд╛ рдирд╣реАрдВ рдорд╛рдирд╛ рдЬрд╛рддрд╛ рд╣реИ, рдЗрд╕рдХрд╛ рд╡рд╛рд╕реНрддрд╡ рдореЗрдВ рдкрд░реАрдХреНрд╖рдг рдирд╣реАрдВ рдХрд┐рдпрд╛ рдЧрдпрд╛ рд╣реИ, рдЗрд╕реЗ рдПрдХ рд╡реИрдХреНрд╕реАрди рдореЗрдВ рдЗрд╕реНрддреЗрдорд╛рд▓ рдХрд░рдиреЗ рдХреА рдЕрдиреБрдорддрд┐ рджреА рдЧрдИ рдереА, рдХреЗрд╡рд▓ рдкрд╛рдВрдЪ рдЧрд┐рдиреА рд╕реВрдЕрд░реЛрдВ рдкрд░ рдкрд░реАрдХреНрд╖рдг рдХреЗ рдкрд░рд┐рдгрд╛рдореЛрдВ рдХреЗ рдЖрдзрд╛рд░ рдкрд░, рдЬрд┐рдиреНрд╣реЗрдВ рдПрдХ рд╕рдордп рдореЗрдВ рдПрдХ рдЦреБрд░рд╛рдХ рджреА рдЧрдИ рдереАред рдЯреАрдХрд╛рдХрд░рдг рдХреЗ рджреМрд░рд╛рди рдмрдЪреНрдЪреЗ рдХреЛ рдкрд╛рдВрдЪ рдЧреБрдирд╛ рдЕрдзрд┐рдХ рдЦреБрд░рд╛рдХ рджреА рдЬрд╛рддреА рд╣реИ! Merthiolate рд╢рд░реАрд░ рд╕реЗ рдЙрддреНрд╕рд░реНрдЬрд┐рдд рдирд╣реАрдВ рд╣реЛрддрд╛ рд╣реИ, рддрдВрддреНрд░рд┐рдХрд╛ рдКрддрдХ рдореЗрдВ рдЬрдорд╛ рд╣реЛ рдЬрд╛рддрд╛ рд╣реИ, рдФрд░ рдПрд▓реНрдпреВрдореАрдирд┐рдпрдо рд╣рд╛рдЗрдбреНрд░реЙрдХреНрд╕рд╛рдЗрдб рдХреЗ рд╕рдВрдпреЛрдЬрди рдореЗрдВ рдЗрд╕рдХреА рд╡рд┐рд╖рд╛рдХреНрддрддрд╛ рджрд╕ рдЧреБрдирд╛ рдмрдврд╝ рдЬрд╛рддреА рд╣реИ! рдпрд╣ рдЕрдиреБрдорд╛рди рд▓рдЧрд╛рдирд╛ рдЖрд╕рд╛рди рд╣реИ рдХрд┐ рдПрд▓реНрдпреБрдорд┐рдирд┐рдпрдо рд╣рд╛рдЗрдбреНрд░реЙрдХреНрд╕рд╛рдЗрдб рднреА рдбреАрдкреАрдЯреА рдХреА рдЦреБрд░рд╛рдХ рдореЗрдВ рдирд┐рд╣рд┐рдд рд╣реИред рдореЗрд░реНрдерд┐рдпреЛрд▓реЗрдЯ рдПрдХ рддрдХрдиреАрдХреА рдХреАрдЯрдирд╛рд╢рдХ рд╣реИ, рдЬрд┐рд╕реЗ рдпреВрд░реЛрдк рди рдХреЗрд╡рд▓ рдПрдХ рджрд╡рд╛ рдорд╛рдирддрд╛ рд╣реИ, рдмрд▓реНрдХрд┐ рдЕрдкрдиреЗ рдХреНрд╖реЗрддреНрд░ рдореЗрдВ рдЗрд╕ рдЬрд╣рд░ рдХрд╛ рдЙрддреНрдкрд╛рджрди рдХрд░рдиреЗ рд╕реЗ рднреА рдЗрдирдХрд╛рд░ рдХрд░рддрд╛ рд╣реИред рдФрд░ рд╣рдорд╛рд░реЗ рджреЗрд╢ рдореЗрдВ, рдпрд╣ рдПрдХ рд╡реИрдХреНрд╕реАрди рдореЗрдВ рд╕реБрд░рдХреНрд╖рд┐рдд рд░реВрдк рд╕реЗ рдЙрдкрдпреЛрдЧ рдХрд┐рдпрд╛ рдЬрд╛рддрд╛ рд╣реИ, рдФрд░ рд╣рдорд╛рд░рд╛ рд╕реНрд╡рд╛рд╕реНрдереНрдп рдордВрддреНрд░рд╛рд▓рдп рдЗрд╕ рджрд╡рд╛ рдХреЗ рдЦрддрд░реЗ рдкрд░ рд╢реЛрдз рднреА рдирд╣реАрдВ рдХрд░рдиреЗ рдЬрд╛ рд░рд╣рд╛ рд╣реИ! рд▓рд╛рдн рдпрд╛ рдЬреЛрдЦрд┐рдо? рд╡рд┐рд╢реНрд╡ рд╕реНрд╡рд╛рд╕реНрдереНрдп рд╕рдВрдЧрдарди рджреНрд╡рд╛рд░рд╛ рдорд╛рдиреНрдпрддрд╛ рдкреНрд░рд╛рдкреНрдд рдЖрдВрдХрдбрд╝реЛрдВ рдХреЗ рдЕрдиреБрд╕рд╛рд░, рдбреАрдкреАрдЯреА рдЯреАрдХрд╛рдХрд░рдг рд▓рдЧрд╛рддрд╛рд░ рдорд╕реНрддрд┐рд╖реНрдХ рдХреНрд╖рддрд┐, рд╡рд┐рднрд┐рдиреНрди рдиреНрдпреВрд░реЛрд▓реЙрдЬрд┐рдХрд▓ рджреМрд░реЗ, рдореГрддреНрдпреБ рддрдХ (рдкреНрд░рддрд┐ рдорд┐рд▓рд┐рдпрди рдЬрдирд╕рдВрдЦреНрдпрд╛ рдкрд░ 5 рдореГрддреНрдпреБ) рдХрд╛ рдХрд╛рд░рдг рдмрдирддрд╛ рд╣реИред 70 рдХреЗ рджрд╢рдХ рдореЗрдВ, рд╕реНрд╡реАрдбрд┐рд╢ рд╡реИрдЬреНрдЮрд╛рдирд┐рдХреЛрдВ рдиреЗ рдкреВрд░реЗ рд╕реЗрд▓ рдбреАрдЯреАрдкреА рд╡реИрдХреНрд╕реАрди рдФрд░ рдПрдиреНрд╕реЗрдлреЗрд▓реЛрдкреИрдереАрдЬ (рджреМрд░реЗ) рдХреА рд╢реБрд░реВрдЖрдд рдХреЗ рд╕рд╛рде рд╕реАрдзрд╛ рд╕рдВрдмрдВрдз рд╕рд╛рдмрд┐рдд рдХрд┐рдпрд╛ред рд╡реИрдЬреНрдЮрд╛рдирд┐рдХреЛрдВ рдиреЗ рдлреИрд╕рд▓рд╛ рдХрд┐рдпрд╛ рд╣реИ рдХрд┐ рдЯреАрдХрд╛рдХрд░рдг рдХреЗ рд▓рд╛рдн рдЬреЛрдЦрд┐рдо рдХреЗ рд▓рд╛рдпрдХ рдирд╣реАрдВ рд╣реИрдВред рдХрдИ рджреЗрд╢реЛрдВ рдореЗрдВ рдЙрдкрдпреЛрдЧ рдкрд░ рдкреНрд░рддрд┐рдмрдВрдз рдХреЗ рдмрд╛рд╡рдЬреВрдж, рд╕рдВрдпреБрдХреНрдд рд░рд╛рдЬреНрдп рдЕрдореЗрд░рд┐рдХрд╛ рддреАрд╕рд░реА рджреБрдирд┐рдпрд╛ рдХреЗ рджреЗрд╢реЛрдВ рдореЗрдВ рдбреАрдЯреАрдкреА рдпреБрдХреНрдд рдбреАрдЯреАрдкреА рдХрд╛ рдЙрддреНрдкрд╛рджрди рдФрд░ рдмрд┐рдХреНрд░реА рдЬрд╛рд░реА рд░рдЦрддрд╛ рд╣реИ, рдЬрдмрдХрд┐ рдЕрдореЗрд░рд┐рдХрд┐рдпреЛрдВ рдиреЗ рдШрд░реЗрд▓реВ рд╕реНрддрд░ рдкрд░ рдЯреАрдХреЗ рдХреЗ рдЗрд╕ рд░реВрдк рдХреЛ рдЫреЛрдбрд╝ рджрд┐рдпрд╛ рд╣реИред рдФрд░ рдкрд░реЗрд╢рд╛рдиреА рдпрд╣ рд╣реИ рдХрд┐ рдХреЛрдИ рднреА рдкрд╣рд▓реЗ рд╕реЗ рдХрднреА рдирд╣реАрдВ рдХрд╣ рд╕рдХрддрд╛ рдХрд┐ рдХреНрдпрд╛ рдЗрд╕ рдЯреАрдХрд╛рдХрд░рдг рд╕реЗ рдЗрд╕ рд╡рд┐рд╢реЗрд╖ рдмрдЪреНрдЪреЗ рдореЗрдВ рдХреБрдЫ рдЬрдЯрд┐рд▓рддрд╛ рд╣реЛрдЧреА, рдпрд╛ рд╕рдм рдХреБрдЫ рдареАрдХ рд╣реЛ рдЬрд╛рдПрдЧрд╛ред рдбреЙрдХреНрдЯрд░ рдЖрд╢реНрд╡рд╕реНрдд рдХрд░рддреЗ рд╣реИрдВ - рдпрд╣ рдПрдХ рд╕реБрд░рдХреНрд╖рд┐рдд рдЯреАрдХрд╛рдХрд░рдг рд╣реИ, рдЬрдЯрд┐рд▓рддрд╛рдПрдВ рдЕрддреНрдпрдВрдд рджреБрд░реНрд▓рдн рд╣реИрдВ, рдФрд░ рдЕрдХреНрд╕рд░ рдпрд╣ рд╕рдм рдЯреАрдХрд╛рдХрд░рдг рд╕реЗ рдкрд╣рд▓реЗ рдпрд╛ рдмрд╛рдж рдореЗрдВ рдЪрд░реНрдЪрд╛ рдирд╣реАрдВ рдХреА рдЬрд╛рддреА рд╣реИ, рдХреЗрд╡рд▓ рдЕрдЧрд░ рдмрдЪреНрдЪреЗ рдХреЗ рд╕рд╛рде рдХреЛрдИ рджреБрд░реНрдШрдЯрдирд╛ рд╣реЛрддреА рд╣реИред рд▓реЗрдХрд┐рди рдЗрд╕ рдорд╛рдорд▓реЗ рдореЗрдВ рднреА, рдЖрдкрдХреЛ рдмрддрд╛рдпрд╛ рдЬрд╛рдПрдЧрд╛ рдХрд┐ рдЯреАрдХрд╛рдХрд░рдг рдХрд╛ рдЗрд╕рд╕реЗ рдХреЛрдИ рд▓реЗрдирд╛-рджреЗрдирд╛ рдирд╣реАрдВ рд╣реИ, рдФрд░ рдпрд╣ рд╕рд╛рдмрд┐рдд рдХрд░рдирд╛ рдореБрд╢реНрдХрд┐рд▓ рд╣реЛрдЧрд╛ рдХрд┐ рдкреНрд░рдХрдЯ рд░реЛрдЧ рдХрд┐рдП рдЧрдП рдЯреАрдХрд╛рдХрд░рдг рд╕реЗ рд╕рдВрдмрдВрдзрд┐рдд рд╣реИрдВред рдЗрд╕ рдЯреАрдХреЗ рд╕реЗ рдХреНрдпрд╛ рдЬрдЯрд┐рд▓рддрд╛рдПрдБ рд╣реЛ рд╕рдХрддреА рд╣реИрдВ, рдЗрд╕ рдкрд░ рдзреНрдпрд╛рди рджреЗрдВ: рддреНрд╡рдЪрд╛ рдкрд░ рд╡рд┐рд╢рд╛рд▓ рдкреНрдпреБрд▓реБрд▓реЗрдВрдЯ рдЯреНрдпреВрдорд░ рдЬрд┐рдиреНрд╣реЗрдВ рдЦреЛрд▓рдирд╛ рдкрдбрд╝рддрд╛ рд╣реИ, рдХреЗрдВрджреНрд░реАрдп рддрдВрддреНрд░рд┐рдХрд╛ рддрдВрддреНрд░ рдХреЛ рдиреБрдХрд╕рд╛рди, рдЬреЛрдбрд╝реЛрдВ, рдЬрдард░рд╛рдВрддреНрд░ рд╕рдВрдмрдВрдзреА рдорд╛рд░реНрдЧ, рд╣реГрджрдп, рд╡рд┐рднрд┐рдиреНрди рдПрд▓рд░реНрдЬреА рдкреНрд░рддрд┐рдХреНрд░рд┐рдпрд╛рдПрдВ, рдЕрд╕реНрдердорд╛, рдордзреБрдореЗрд╣, рдЕрд╡реНрдпрдХреНрдд рд░реЛрдЧреЛрдВ рдХрд╛ рдЬрд╛рдЧрд░рдг - рддрдкреЗрджрд┐рдХ , рд╣реЗрдкреЗрдЯрд╛рдЗрдЯрд┐рд╕; рддреАрд╡реНрд░рдЧреНрд░рд╛рд╣рд┐рддрд╛ рдЖрдШрд╛рдд, рдЕрдЪрд╛рдирдХ рдореМрдд... рдЕрдиреНрдп рдмреАрдорд╛рд░рд┐рдпреЛрдВ рдХреА рдорд╣рд╛рдорд╛рд░реА рдХреЗ рджреМрд░рд╛рди рдЯреАрдХрд╛рдХрд░рдг рд╣реЛ рд╕рдХрддрд╛ рд╣реИ рдШрд╛рддрдХ! рддреЛ рдХреНрдпрд╛ рдпрд╣ рдЗрд╕рдХреЗ рд▓рд╛рдпрдХ рд╣реИ, рдкрд╣рд▓реЗ рд╕реЗ рдкрд░реНрдЯреБрд╕рд┐рд╕ рдорд╣рд╛рдорд╛рд░реА рдХреЗ рдбрд░ рд╕реЗ, рдмрдЪреНрдЪреЗ рдХреЗ рд╢рд░реАрд░ рдореЗрдВ рд░реЛрдЧрдЬрдирдХ рдХреЛрд╢рд┐рдХрд╛рдУрдВ рдФрд░ рд╡рд┐рд╖рд╛рдХреНрдд рдкрджрд╛рд░реНрдереЛрдВ рдХреА рдРрд╕реА рдЦрддрд░рдирд╛рдХ рдЦреБрд░рд╛рдХ рдХреА рд╢реБрд░реВрдЖрдд рдХреЗ рд▓рд┐рдП рд╕рд╣рдордд рд╣реЛрдирд╛, рдЬрд┐рд╕рд╕реЗ рдпрд╣ рдЬреЛрдЦрд┐рдо рдмрд╣реБрдд рдмрдврд╝ рдЬрд╛рддрд╛ рд╣реИ рдХрд┐ рдмрдЪреНрдЪрд╛ рддрдм рдЕрдХреНрд╖рдо рд╣реЛ рдЬрд╛рдПрдЧрд╛ рдпрд╛ рдЗрд╕рд╕реЗ рднреА рдмрджрддрд░ рдорд░рдирд╛? рдпрд╛ рд╢рд╛рдпрдж рдпрд╣ рдмрдЪреНрдЪреЗ рдХреЗ рд╕реНрд╡рд╛рд╕реНрдереНрдп рдХреЛ рдмрдирд╛рдП рд░рдЦрдиреЗ рдФрд░ рдЙрд╕рдХреА рдкреНрд░рддрд┐рд░рдХреНрд╖рд╛ рдХреЛ рдордЬрдмреВрдд рдХрд░рдиреЗ рдХреЗ рд╡реИрдХрд▓реНрдкрд┐рдХ рддрд░реАрдХреЛрдВ рдХреЛ рдЦреЛрдЬрдиреЗ рдХреЗ рд▓рд╛рдпрдХ рд╣реИ? рдорд╛рддрд╛-рдкрд┐рддрд╛ рдХреЛ рдЯреАрдХрд╛рдХрд░рдг рд╕реЗ рдЗрдирдХрд╛рд░ рдХрд░рдиреЗ рдпрд╛ рд╕рд╣рдордд рд╣реЛрдиреЗ рдХрд╛ рдЕрдзрд┐рдХрд╛рд░ рд╣реИ, рд▓реЗрдХрд┐рди рдХрд┐рд╕реА рднреА рдорд╛рдорд▓реЗ рдореЗрдВ, рдкрд╣рд▓реЗ рд╡рд┐рд╢реНрд╡рд╕рдиреАрдп рд╕реНрд░реЛрддреЛрдВ рд╕реЗ рдкреВрд░реА рдЬрд╛рдирдХрд╛рд░реА рдкреНрд░рд╛рдкреНрдд рдХрд░рдиреЗ рдХреЗ рд▓рд╛рдпрдХ рд╣реИ, рдЬреЛ рд╕реМрднрд╛рдЧреНрдп рд╕реЗ, рдЖрдЬ рд╕рднреА рдХреЗ рд▓рд┐рдП рдкрд╣рд▓реЗ рд╕реЗ рд╣реА рдЙрдкрд▓рдмреНрдз рд╣реИред рдФрд░ рдЕрдм рд╕рд╡рд╛рд▓ рдпрд╣ рд╣реИ рдХрд┐ рдРрд╕рд╛ рдХреИрд╕реЗ рди рдХрд░реЗрдВ? рдЖрдЦрд┐рд░ рдЬрд░реВрд░реА рд╣реИ!!! рдмреЗрд╣рддрд░ рд╣реИ рдЦреБрдж рдХрд╛ рдмреАрдорд╛ рдХрд░рд╛ рд▓реЗрдВ!!! рд╢рд╣рд░ рдореЗрдВ рдЕрдм рд╣рдорд╛рд░реЗ рдмрдЪреНрдЪреЗ рд╣реИрдВ рдЬреЛ рдЦрд╛рдВрд╕реА рд╕реЗ рдмреАрдорд╛рд░ рд╣реЛ рдЧрдП рд╣реИрдВ, рдЙрдиреНрд╣реЛрдВрдиреЗ рдЗрд╕ рдмреАрдорд╛рд░реА рдХреЗ рдмрд╛рд░реЗ рдореЗрдВ рдкрд╣рд▓реЗ рдХрднреА рдирд╣реАрдВ рд╕реБрдирд╛ рд╣реИ, рд▓реЗрдХрд┐рди рдЕрдм рд╡реЗ рдЦреБрдж рдХреЛ рдкреНрд░рдХрдЯ рдХрд░рдирд╛ рд╢реБрд░реВ рдХрд░ рдЪреБрдХреЗ рд╣реИрдВ, рдореИрдВ 3 рдмрдЪреНрдЪреЛрдВ рдХреА рдорд╛рдВ рд╣реВрдВ, рдореЗрд░реА рджреЛ рдмреЗрдЯрд┐рдпреЛрдВ рдХреЛ рдбреАрдкреАрдЯреА рдЯреАрдЯреАрдЯреА рдорд┐рд▓рд╛ рд╣реИ, рдореИрдВрдиреЗ рдЕрднреА рдЧрдВрднреАрд░ рдЯреАрдХрд╛рдХрд░рдг рд╕реЗ рдЗрдирдХрд╛рд░ рди рдХрд░реЗрдВред рдХреНрдпрд╛ рдпрд╣ рдЗрд╕рдХреЗ рд▓рд╛рдпрдХ рд╣реИ рдпрд╛ рдирд╣реАрдВ рдХрд░рдирд╛ рд╣реИ? ??

рдбреАрдЯреАрдкреА рдЯреАрдХрд╛рдХрд░рдг рдХреЗ рд╕рдмрд╕реЗ рдЕрдкреНрд░рд┐рдп рдкрд░рд┐рдгрд╛рдо рдирд╣реАрдВ рд╣реЛ рд╕рдХрддреЗ рд╣реИрдВред рдЗрд╕реЗ рджреЗрдЦрддреЗ рд╣реБрдП, рдХрдИ рдорд╛рддрд╛-рдкрд┐рддрд╛ рдЗрд╕рд╕реЗ рд╕рд╛рд╡рдзрд╛рди рд╣реИрдВ, рдЗрд╕ рддрдереНрдп рдХреЗ рдмрд╛рд╡рдЬреВрдж рдХрд┐ рдпрд╣ рд╢рд░реАрд░ рдХреЛ рд╡рд╛рдпрд░рд╕ рд╕реЗ рдмрдЪрд╛рдиреЗ рдХреЗ рд▓рд┐рдП рдбрд┐рдЬрд╝рд╛рдЗрди рдХрд┐рдпрд╛ рдЧрдпрд╛ рд╣реИ рдЬреЛ рдорд╣рд╛рдорд╛рд░реА рдХрд╛ рдХрд╛рд░рдг рдмрди рд╕рдХрддрд╛ рд╣реИред

рддреЛ рдЯреАрдХрд╛рдХрд░рдг рдХреЗ рд▓рд┐рдП рдмрдЪреНрдЪреЗ рдХреЗ рд╕рд╛рде рдЬрд╛рддреЗ рд╕рдордп рдХреНрдпрд╛ рддреИрдпрд╛рд░ рдХрд┐рдпрд╛ рдЬрд╛рдирд╛ рдЪрд╛рд╣рд┐рдП, рдФрд░ рдЗрд╕ рд╣реЗрд░рдлреЗрд░ рдХреЗ рд▓рд┐рдП рд╕рд╣реА рддреИрдпрд╛рд░реА рдХреНрдпрд╛ рд╣реЛрдиреА рдЪрд╛рд╣рд┐рдПред рдорд╛рддрд╛-рдкрд┐рддрд╛ рдХреЛ рдЯреАрдХреЗ рдХреА рдЙрдкрдпреБрдХреНрддрддрд╛ рдФрд░ рдЗрд╕рдХреЗ рд╕рдВрднрд╛рд╡рд┐рдд рд╡рд┐рдХрд▓реНрдк рдХреЗ рдмрд╛рд░реЗ рдореЗрдВ рд╕реВрдЪрд┐рдд рдХрд┐рдпрд╛ рдЬрд╛рдирд╛ рдЪрд╛рд╣рд┐рдПред

рдкреНрд░рддреНрдпреЗрдХ рдЯреАрдХрд╛рдХрд░рдг, рдЗрд╕рдХреЗ рдЯреАрдХрд╛рдХрд░рдг рдХрд╛ рдЙрджреНрджреЗрд╢реНрдп рдмрдЪреНрдЪреЗ рдХреЗ рд╢рд░реАрд░ рдореЗрдВ рд╕реБрд░рдХреНрд╖рд╛рддреНрдордХ рдПрдВрдЯреАрдмреЙрдбреА рдХреЗ рдЙрддреНрдкрд╛рджрди рдХреЛ рдмрдврд╝рд╛рд╡рд╛ рджреЗрдирд╛ рд╣реИред рдбреАрдкреАрдЯреА рдХреЛ рд╕рдмрд╕реЗ рдХрдард┐рди рдЯреАрдХрд╛рдХрд░рдгреЛрдВ рдореЗрдВ рд╕реЗ рдПрдХ рдорд╛рдирд╛ рдЬрд╛ рд╕рдХрддрд╛ рд╣реИ, рдХреНрдпреЛрдВрдХрд┐ рдЗрдВрдЬреЗрдХреНрд╢рди рдХреЗ рджреМрд░рд╛рди рдЗрдВрдЬреЗрдХреНрд╢рди рд╡рд╛рд▓реЗ рд╕реАрд░рдо рдореЗрдВ рддреАрди рдШрдЯрдХ рд╣реЛрддреЗ рд╣реИрдВред

рддреАрди-рдШрдЯрдХ рдЯреАрдХреЗ (рдХрд╛рд▓реА рдЦрд╛рдВрд╕реА, рдЯреЗрдЯрдирд╕, рдбрд┐рдкреНрдереАрд░рд┐рдпрд╛) рдореЗрдВ, рд╡рд╣ рдШрдЯрдХ рдЬреЛ рд╢рд░реАрд░ рдореЗрдВ рдПрдХ рдордЬрдмреВрдд рдкреНрд░рддрд┐рдХреНрд░рд┐рдпрд╛ рдкреИрджрд╛ рдХрд░ рд╕рдХрддрд╛ рд╣реИ, рд╡рд╣ рд╣реИ рдХрд╛рд▓реА рдЦрд╛рдВрд╕реАред рдЗрд╕реЗ рдЗрд╕ рддрдереНрдп рд╕реЗ рд╕рдордЭрд╛рдпрд╛ рдЬрд╛ рд╕рдХрддрд╛ рд╣реИ рдХрд┐ рдпрд╣ рд╡рд╣ рдШрдЯрдХ рд╣реИ рдЬреЛ рдПрдХ рдЬреАрд╡рд┐рдд рдЬреАрд╡рд╛рдгреБ рдХреЗ рдЖрдзрд╛рд░ рдкрд░ рдмрдирд╛рдпрд╛ рдЧрдпрд╛ рд╣реИ рдЬреЛ рдкреНрд░рд╛рд░рдВрднрд┐рдХ рдкреНрд░рд╕рдВрд╕реНрдХрд░рдг (рдЖрдВрд╢рд┐рдХ) рд╕реЗ рдЧреБрдЬрд░рд╛ рд╣реИред

рдХрд╛рд▓реА рдЦрд╛рдВрд╕реА рдХрд╛ рдмрдЪреНрдЪреЗ рдХреЗ рд╢рд░реАрд░ рдкрд░ рдХрд╛рдлреА рдкреНрд░рднрд╛рд╡ рдкрдбрд╝рддрд╛ рд╣реИред рдЙрджрд╛рд╣рд░рдг рдХреЗ рд▓рд┐рдП, рдпрд╣ рдмрдЪреНрдЪреЗ рдХреЗ рдорд╕реНрддрд┐рд╖реНрдХ рдкрд░ рдирдХрд╛рд░рд╛рддреНрдордХ рдкреНрд░рднрд╛рд╡ рдбрд╛рд▓ рд╕рдХрддрд╛ рд╣реИ рдФрд░ рдЙрд╕рдХреА рдЕрдкрд░рд┐рдкрдХреНрд╡рддрд╛ рдХреЗ рдХрд╛рд░рдг рдЙрд╕рдХреЗ рддрдВрддреНрд░рд┐рдХрд╛ рддрдВрддреНрд░ рдХреЗ рдХрд╛рдордХрд╛рдЬ рдХреЛ рдмрд╛рдзрд┐рдд рдХрд░ рд╕рдХрддрд╛ рд╣реИред рд╡реИрдХреНрд╕реАрди рдмрдирд╛рдиреЗ рд╡рд╛рд▓реЗ рдмрд╛рдХреА рдШрдЯрдХреЛрдВ рдХреЗ рд▓рд┐рдП, рд╡реЗ рд╢рд╛рдпрдж рд╣реА рдХрднреА рд╢рд░реАрд░ рдореЗрдВ рдирдХрд╛рд░рд╛рддреНрдордХ рдкреНрд░рддрд┐рдХреНрд░рд┐рдпрд╛рдУрдВ рдХреА рдЙрдкрд╕реНрдерд┐рддрд┐ рдореЗрдВ рдпреЛрдЧрджрд╛рди рдХрд░рддреЗ рд╣реИрдВред рдЖрдЦрд┐рд░рдХрд╛рд░, рдЙрдирдХрд╛ рдЙрдкрдпреЛрдЧ рдкреВрд░реА рддрд░рд╣ рд╕реЗ рд╕реЛрдЦрдиреЗ рд╡рд╛рд▓реЗ рдкрджрд╛рд░реНрде рдХреЗ рд░реВрдк рдореЗрдВ рдХрд┐рдпрд╛ рдЬрд╛рддрд╛ рд╣реИ рдЬрд┐рд╕реЗ рд╢рд░реАрд░ рдЕрдЪреНрдЫреА рддрд░рд╣ рд╕реЗ рдЕрд╡рд╢реЛрд╖рд┐рдд рдХрд░рддрд╛ рд╣реИред

рдРрд╕реЗ рд╕рдордп рд╣реЛрддреЗ рд╣реИрдВ рдЬрдм рдПрдХ рдмрдЪреНрдЪреЗ рдХреЛ рддреАрди рдШрдЯрдХреЛрдВ рд╡рд╛рд▓реЗ рдЯреАрдХреЗ рдХреЗ рд▓рд┐рдП рдорддрднреЗрдж рд╣реЛрддреЗ рд╣реИрдВред рдЗрдирдореЗрдВ рддреАрд╡реНрд░ рд╢реНрд╡рд╕рди рд╕рдВрдХреНрд░рдордг, рдкреБрд░рд╛рдиреА рдмреАрдорд╛рд░рд┐рдпрд╛рдВ, рдЬрдЯрд┐рд▓рддрд╛рдПрдВ рдпрд╛ рдкрд┐рдЫрд▓реЗ рдЯреАрдХреЗ рдХреА рдордЬрдмреВрдд рдкреНрд░рддрд┐рдХреНрд░рд┐рдпрд╛ рд╢рд╛рдорд┐рд▓ рд╣реИ, рдмрдЪреНрдЪреЗ рдХреЛ рддрдВрддреНрд░рд┐рдХрд╛ рддрдВрддреНрд░ рдХреА рдмреАрдорд╛рд░реА рд╣реИред

рдРрд╕реЗ рдорд╛рдорд▓реЛрдВ рдореЗрдВ, рджреЛ-рдШрдЯрдХ рд╕реАрд░рдо (рдПрдбреАрдПрд╕) рдХрд╛ рдЯреАрдХрд╛рдХрд░рдг рдпрд╛ рдЙрдкрдпреЛрдЧ рдирд╣реАрдВ рдХрд░рдирд╛ рдмреЗрд╣рддрд░ рд╣реИ, рд▓реЗрдХрд┐рди рдХреЗрд╡рд▓ рдПрдХ рдбреЙрдХреНрдЯрд░ рджреНрд╡рд╛рд░рд╛ рдЯреБрдХрдбрд╝реЛрдВ рдХреА рдкреВрд░реА рдЬрд╛рдВрдЪ рдХреЗ рдмрд╛рдж, рдЖрд╡рд╢реНрдпрдХ рдкрд░реАрдХреНрд╖рдг рдХрд░рдирд╛ред

рдХреНрдпрд╛ рдкрд░рд┐рдгрд╛рдо рд╕рд╛рдорд╛рдиреНрдп рдорд╛рдиреЗ рдЬрд╛рддреЗ рд╣реИрдВ

рдбреАрдкреАрдЯреА рдХрд╛ рдЯреАрдХрд╛ рд▓рдЧрд╡рд╛рдиреЗ рдХреЗ рдмрд╛рдж, рдорд╛рддрд╛-рдкрд┐рддрд╛ рдХреЛ рдЗрд╕ рддрдереНрдп рдХреЗ рд▓рд┐рдП рддреИрдпрд╛рд░ рд░рд╣рдирд╛ рдЪрд╛рд╣рд┐рдП рдХрд┐ рдЙрдиреНрд╣реЗрдВ рдХреБрдЫ рдкрд░рд┐рдгрд╛рдореЛрдВ рдХрд╛ рд╕рд╛рдордирд╛ рдХрд░рдирд╛ рдкрдбрд╝ рд╕рдХрддрд╛ рд╣реИред рд╕рднреА рдмрдЪреНрдЪреЗ рдЕрдкрдиреА рдЕрднрд┐рд╡реНрдпрдХреНрддрд┐ рдХреЗ рд╕рдВрдкрд░реНрдХ рдореЗрдВ рдирд╣реАрдВ рдЖрддреЗ рд╣реИрдВ, рд▓реЗрдХрд┐рди рдЬреНрдпрд╛рджрд╛рддрд░ рдорд╛рдорд▓реЛрдВ рдореЗрдВ рдХрдордЬреЛрд░ рдкреНрд░рддрд┐рд░рдХреНрд╖рд╛ рд╡рд╛рд▓реЗред рдЕрдиреНрдп рдорд╛рдорд▓реЛрдВ рдореЗрдВ, рдмрдЪреНрдЪреЗ рдХрд╛ рд╡реНрдпрд╡рд╣рд╛рд░ рдФрд░ рд╕реНрдерд┐рддрд┐ рдЕрдкрд░рд┐рд╡рд░реНрддрд┐рдд рд░рд╣рддреА рд╣реИред рдЯреАрдХрд╛рдХрд░рдг рдХреЗ рдмрд╛рдж рдЬреЛ рдкрд░рд┐рдгрд╛рдо рд╣реЛ рд╕рдХрддреЗ рд╣реИрдВ, рдФрд░ рдЖрджрд░реНрд╢ рд╣реИрдВ, рдЙрдирдореЗрдВ рд╢рд╛рдорд┐рд▓ рд╣реИрдВ:

  • рдЙрд╕ рдЬрдЧрд╣ рдкрд░ рд▓рд╛рд▓реА рдХрд╛ рджрд┐рдЦрдирд╛ рдЬрд╣рд╛рдВ рдЯреАрдХрд╛ рджрд┐рдпрд╛ рдЧрдпрд╛ рдерд╛ред рдпрджрд┐ рдпрд╣ рд╕реНрдерд╛рди рдЧрд░реНрдо рдирд╣реАрдВ рд╣реИ, рдЗрд╕рдХрд╛ рд╡реНрдпрд╛рд╕ 10 рд╕реЗрдВрдЯреАрдореАрдЯрд░ рд╕реЗ рдЕрдзрд┐рдХ рдирд╣реАрдВ рд╣реИ, рддреЛ рдЖрдкрдХреЛ рдШрдмрд░рд╛рдирд╛ рдирд╣реАрдВ рдЪрд╛рд╣рд┐рдПред рдпрд╣ рд╕реБрдирд┐рд╢реНрдЪрд┐рдд рдХрд░рдирд╛ рдорд╣рддреНрд╡рдкреВрд░реНрдг рд╣реИ рдХрд┐ рдпрд╣ рдЖрдХрд╛рд░ рдореЗрдВ рд╡реГрджреНрдзрд┐ рдирд╣реАрдВ рдХрд░рддрд╛ рд╣реИ рдФрд░ рдЗрд╕рдХрд╛ рдЖрдХрд╛рд░ рдирд╣реАрдВ рдмрджрд▓рддрд╛ рд╣реИ, рдХреНрдпреЛрдВрдХрд┐ рдпрд╣ рд╕реНрд╡рдпрдВ рдХреЛ рдПрдбреАрдорд╛, рджрдорди рдХреЗ рд░реВрдк рдореЗрдВ рдкреНрд░рдХрдЯ рдХрд░ рд╕рдХрддрд╛ рд╣реИ, рдФрд░ рдЗрд╕реЗ рдЕрдм рд╢рд░реАрд░ рдХреА рд╕рд╛рдорд╛рдиреНрдп рдкреНрд░рддрд┐рдХреНрд░рд┐рдпрд╛ рдирд╣реАрдВ рдХрд╣рд╛ рдЬрд╛ рд╕рдХрддрд╛ рд╣реИред

рднрдбрд╝рдХрд╛рдК рдкреНрд░рдХреНрд░рд┐рдпрд╛ рдХреЗ рдХрд╛рд░рдг, рдмрдЪреНрдЪреЗ рдХреЛ рджрд░реНрджрдирд╛рдХ рд╕рдВрд╡реЗрджрдирд╛рдУрдВ рдХрд╛ рдЕрдиреБрднрд╡ рд╣реЛ рд╕рдХрддрд╛ рд╣реИ рдЬреЛ рдордиреЛрджрд╢рд╛ рдФрд░ рдЕрд╢рд╛рдВрддрд┐, рдиреАрдВрдж рдХреА рдЧрдбрд╝рдмрдбрд╝реА, рднреВрдЦ рдХреЛ рдЙрддреНрддреЗрдЬрд┐рдд рдХрд░рддрд╛ рд╣реИред рдЖрдк рдПрдХ рджрд┐рди рдкрд╣рд▓реЗ рдбреЙрдХреНрдЯрд░ рдХреЗ рд╕рд╛рде рд╕рд╣рдордд рджрд░реНрдж рдирд┐рд╡рд╛рд░рдХ рджрд╡рд╛рдУрдВ рдХрд╛ рдЙрдкрдпреЛрдЧ рдХрд░рдХреЗ рдмрдЪреНрдЪреЗ рдХреА рдорджрдж рдХрд░ рд╕рдХрддреЗ рд╣реИрдВред

  • рд╢рд░реАрд░ рдХреЗ рддрд╛рдкрдорд╛рди рдореЗрдВ рд╡реГрджреНрдзрд┐ред рдпрд╣ рдЗрд╕ рддрдереНрдп рдХреЗ рдХрд╛рд░рдг рд╣реЛрддрд╛ рд╣реИ рдХрд┐ рд╢рд░реАрд░ рдПрдВрдЯреАрдмреЙрдбреА рдХреЗ рдЙрддреНрдкрд╛рджрди рдХреЗ рд▓рд┐рдП рд▓рдбрд╝ рд░рд╣рд╛ рд╣реИред рдпрджрд┐ рддрд╛рдкрдорд╛рди 38 рдбрд┐рдЧреНрд░реА рд╕реЗ рдЕрдзрд┐рдХ рд╣реЛ рддреЛ рдЙрд╕реЗ рдиреАрдЪреЗ рд▓рд╛рдпрд╛ рдЬрд╛рдирд╛ рдЪрд╛рд╣рд┐рдП, рдЬрд┐рд╕рд╕реЗ рдмрдЪреНрдЪреЗ рдХреА рд╕рд╛рдорд╛рдиреНрдп рд╕реНрдерд┐рддрд┐ рд╕реБрдирд┐рд╢реНрдЪрд┐рдд рд╣реЛрддреА рд╣реИ, рдФрд░ рд╢рд░реАрд░ рдореЗрдВ рд╕реБрд░рдХреНрд╖рд╛рддреНрдордХ рдПрдВрдЯреАрдмреЙрдбреА рдмрдирд╛рдиреЗ рдХреА рдкреНрд░рдХреНрд░рд┐рдпрд╛ рдареАрдХ рд╕реЗ рд╣реЛрддреА рд╣реИред
  • рдПрдХ рдмрдЪреНрдЪреЗ рдХреА рдЦрд╛рдБрд╕реА, рдмрд╣рддреА рдирд╛рдХ рдХреА рдЙрдкрд╕реНрдерд┐рддрд┐ рдПрдХ рдФрд░ рд╕рд╛рдорд╛рдиреНрдп рдкрд░рд┐рдгрд╛рдо рд╣реИ рдЬреЛ рдбреАрдкреАрдЯреА рдХреЗ рдмрд╛рдж рд╣реЛ рд╕рдХрддрд╛ рд╣реИред рд▓рдХреНрд╖рдг рд╕реАрд░рдо рдХреЗ рдШрдЯрдХреЛрдВ рдореЗрдВ рд╕реЗ рдПрдХ рдХреЗ рдХрд╛рд░рдг рд╣реЛ рд╕рдХрддреЗ рд╣реИрдВ, рдХрд╛рд▓реА рдЦрд╛рдВрд╕реАред

рдпрджрд┐ crumbs рдХреЗ рдЙрдкрд░реЛрдХреНрдд рдкрд░рд┐рдгрд╛рдо рд╣реИрдВ, рддреЛ рдЖрдк рджрд░реНрдж рдХреЛ рдХрдо рдХрд░рдХреЗ рдЙрд╕рдХреА рдорджрдж рдХрд░ рд╕рдХрддреЗ рд╣реИрдВред рдЗрд╕реЗ рд╕реВрдЬрди рд╡рд╛рд▓реЗ рддреНрд╡рдЪрд╛ рдХреНрд╖реЗрддреНрд░ рдХреА рд╣рд▓реНрдХреА рдорд╛рд▓рд┐рд╢ рдХрд░рдиреЗ рдХреА рдЕрдиреБрдорддрд┐ рд╣реИ, рдПрдХ рд╕реЗрдХ рдпрд╛ рдЙрддреНрдкрд╛рдж (рдорд░рд╣рдо, рдХреНрд░реАрдо) рдХрд╛ рдЙрдкрдпреЛрдЧ рдХрд░реЗрдВ рдЬреЛ рдЕрд╕реБрд╡рд┐рдзрд╛ рдХреЛ рдХрдо рдХрд░реЗрдЧрд╛ред рдЬреНрд╡рд░рдирд╛рд╢рдХ рджрд╡рд╛рдУрдВ рдХреЛ рднреА рд╕рдордЭрджрд╛рд░реА рд╕реЗ рддреИрдпрд╛рд░ рдХрд┐рдпрд╛ рдЬрд╛рдирд╛ рдЪрд╛рд╣рд┐рдП, рдЖрдЬ рдЙрдирдореЗрдВ рд╕реЗ рдЕрдзрд┐рдХрд╛рдВрд╢ рди рдХреЗрд╡рд▓ рд╢рд░реАрд░ рдХреЗ рддрд╛рдкрдорд╛рди рдХреЛ рдХрдо рдХрд░рдиреЗ рдХреА рдЕрдиреБрдорддрд┐ рджреЗрддреЗ рд╣реИрдВ, рдмрд▓реНрдХрд┐ рдЙрдЪрд┐рдд рдПрдирд╛рд▓реНрдЬреЗрд╕рд┐рдХ рдкреНрд░рднрд╛рд╡ рднреА рдкреНрд░рджрд╛рди рдХрд░рддреЗ рд╣реИрдВред

рдЧрдВрднреАрд░ рдкрд░рд┐рдгрд╛рдо

рдбреАрдкреАрдЯреА рдЯреАрдХрд╛рдХрд░рдг рдХреЗ рдкрд░рд┐рдгрд╛рдо рд╢рд┐рд╢реБ рдХреЗ рд╕реНрд╡рд╛рд╕реНрдереНрдп рдХреЗ рд▓рд┐рдП рдЦрддрд░рдирд╛рдХ рд╣реЛ рд╕рдХрддреЗ рд╣реИрдВред рдорд╛рддрд╛-рдкрд┐рддрд╛ рдЬреЛ рдЗрд╕реЗ рдЕрдкрдиреЗ рдмрдЪреНрдЪреЗ рдХреЛ рдмрдирд╛рдиреЗ рдХреА рдпреЛрдЬрдирд╛ рдмрдирд╛ рд░рд╣реЗ рд╣реИрдВ, рдЙрдиреНрд╣реЗрдВ рдЗрд╕рдХреЗ рдмрд╛рд░реЗ рдореЗрдВ рдкрддрд╛ рд╣реЛрдирд╛ рдЪрд╛рд╣рд┐рдПред рдЧрдВрднреАрд░ рдкрд░рд┐рдгрд╛рдореЛрдВ рдореЗрдВ рд╢рд╛рдорд┐рд▓ рд╣реИрдВ:

  • рдРрдВрдардиред рдЗрд╕рдХреА рдЙрдкрд╕реНрдерд┐рддрд┐ рдЗрд╕ рддрдереНрдп рдХреЗ рдХрд╛рд░рдг рд╣реИ рдХрд┐ рд╕реАрд░рдо рд▓рдЧрд╛рдиреЗ рдХреЗ рдмрд╛рдж рдмрдЪреНрдЪреЗ рдХреЗ рд╢рд░реАрд░ рдХрд╛ рддрд╛рдкрдорд╛рди рдмрдврд╝ рдЬрд╛рддрд╛ рд╣реИред рдЗрд╕реА рд╕рдордп, рд╢рд░реАрд░ рдореЗрдВ рдПрдХ рдиреНрдпреВрд░реЛрд▓реЙрдЬрд┐рдХрд▓ рдкреНрд░рдХреГрддрд┐ рдХреЗ рд╡рд┐рдЪрд▓рди рд╣реЛрддреЗ рд╣реИрдВред рдЗрд╕рд▓рд┐рдП, рдЗрд╕рдореЗрдВ 38 рдбрд┐рдЧреНрд░реА рддрдХ рдХреА рд╡реГрджреНрдзрд┐ рднреА рдЗрд╕ рд╕реНрдерд┐рддрд┐ рдХреЛ рднрдбрд╝рдХрд╛ рд╕рдХрддреА рд╣реИ рдФрд░ рд╕реЗрд░реЗрдмреНрд░рд▓ рд╣рд╛рдЗрдкреЛрдХреНрд╕рд┐рдпрд╛ рдХреЛ рдЬрдиреНрдо рджреЗ рд╕рдХрддреА рд╣реИред
  • рдПрд▓рд░реНрдЬреАред рдЗрд╕рдХреА рдЕрднрд┐рд╡реНрдпрдХреНрддрд┐ рдХреА рдбрд┐рдЧреНрд░реА рдХреЗ рдЖрдзрд╛рд░ рдкрд░, рддреНрд╡рдЪрд╛ рдкрд░ рдЪрдХрддреНрддреЗ рдЕрд▓реНрдкрдХрд╛рд▓рд┐рдХ рдкреНрд░рдХреГрддрд┐ рдХреЗ рд╣реЛ рд╕рдХрддреЗ рд╣реИрдВ, рдФрд░ рдХреБрдЫ рджрд┐рдиреЛрдВ рдХреЗ рдмрд╛рдж, рд╕реАрд░рдо рдХреЗ рдкреНрд░рд╢рд╛рд╕рди рдХреЗ рдмрд╛рдж рдЧрд╛рдпрдм рд╣реЛ рдЬрд╛рддреЗ рд╣реИрдВ, рдпрд╛ рд╡реЗ рдЕрдзрд┐рдХ рдЧрдВрднреАрд░ рдЬрдЯрд┐рд▓рддрд╛рдУрдВ рдореЗрдВ рдкрд░рд┐рд╡рд░реНрддрд┐рдд рд╣реЛ рд╕рдХрддреЗ рд╣реИрдВред рд╣рдо рдПрдЯреЛрдкрд┐рдХ рдЬрд┐рд▓реНрдж рдХреА рд╕реВрдЬрди, рдбрд╛рдпрдереЗрд╕рд┐рд╕ рдХреЗ рдмрд╛рд░реЗ рдореЗрдВ рдмрд╛рдд рдХрд░ рд░рд╣реЗ рд╣реИрдВред рдпрд╣ рд╕рдм рдмрдЪреНрдЪреЛрдВ рдХреЛ рдЕрд╕реБрд╡рд┐рдзрд╛ рджреЗрддрд╛ рд╣реИ рдФрд░ рд╡рд┐рд╢реЗрд╖ рдЙрдкрдЪрд╛рд░ рдХреА рдЖрд╡рд╢реНрдпрдХрддрд╛ рд╣реЛрддреА рд╣реИред
  • рдЖрдВрддрд░рд┐рдХ рдЕрдВрдЧреЛрдВ рдХреА рд╕реВрдЬрди, рдЬреЛ рдПрд▓рд░реНрдЬреА рдХреА рдкреНрд░рддрд┐рдХреНрд░рд┐рдпрд╛ рдХреА рдЙрдкрд╕реНрдерд┐рддрд┐ рд╕реЗ рдЬреБрдбрд╝реА рд╣реЛрддреА рд╣реИред рдЗрд╕ рд╕рдореВрд╣ рдХреЗ рдЧрдВрднреАрд░ рдкрд░рд┐рдгрд╛рдореЛрдВ рдореЗрдВ рдПрдирд╛рдлрд┐рд▓реЗрдХреНрдЯрд┐рдХ рд╕рджрдореЗ рдХреЛ рднреА рдиреЛрдЯ рдХрд┐рдпрд╛ рдЧрдпрд╛ рдерд╛ред
  • рд╕реЗрд░реЗрдмреНрд░рд▓ рдПрдиреНрд╕реЗрдлрд▓рд╛рдЗрдЯрд┐рд╕ рдПрдХ рдЧрдВрднреАрд░ рдЬрдЯрд┐рд▓рддрд╛ рд╣реИ рдЬреЛ рдкреВрд░реА рддрд░рд╣ рд╕реЗ рдмрдирд╛ рд╕рдХрддреА рд╣реИ рд╕реНрд╡рд╕реНрде рдмрдЪреНрдЪрд╛рд╡рд┐рдХрд▓рд╛рдВрдЧ рд╡реНрдпрдХреНрддрд┐ (рдбрд╛рдЙрдирд┐рдЬреНрдо)ред рдпрд╣ рдЗрд╕ рддрдереНрдп рдХреЗ рдХрд╛рд░рдг рд╣реЛ рд╕рдХрддрд╛ рд╣реИ рдХрд┐ рдЯреАрдХреЗ рдХреЗ рдШрдЯрдХ - рдХрд╛рд▓реА рдЦрд╛рдВрд╕реА, рдореЗрдирд┐рдиреНрдЬреЗрд╕ рдкрд░ рдирдХрд╛рд░рд╛рддреНрдордХ рдкреНрд░рднрд╛рд╡ рдбрд╛рд▓рддреА рд╣реИред


рдмрдЪреНрдЪреЗ рдХреЗ рд╢рд░реАрд░ рдореЗрдВ рддреАрди-рдШрдЯрдХ рдЯреАрдХреЗ рдХреА рд╢реБрд░реВрдЖрдд рдХреЗ рдмрд╛рдж рдкрд╣рд▓реЗ рддреАрди рджрд┐рдиреЛрдВ рдореЗрдВ, рдЙрдкрд░реНрдпреБрдХреНрдд рдкрд░рд┐рдгрд╛рдореЛрдВ рдХреА рд╕рдВрднрд╛рд╡рдирд╛ рд╣реИред рдЗрд╕ рд╕рдВрдмрдВрдз рдореЗрдВ, рд╣рд░ рд╕рд╛рд▓ рдЕрдзрд┐рдХ рд╕реЗ рдЕрдзрд┐рдХ рдорд╛рддрд╛-рдкрд┐рддрд╛ рдЗрд╕реЗ рдЫреЛрдбрд╝рдиреЗ рдХрд╛ рдлреИрд╕рд▓рд╛ рдХрд░рддреЗ рд╣реИрдВ, рдпрд╣ рдирд╣реАрдВ рд╕рдордЭрддреЗ рдХрд┐ рд╡реЗ рдЕрдкрдиреЗ рдмрдЪреНрдЪреЗ рдХреЛ рдХрд┐рд╕ рдЬреЛрдЦрд┐рдо рдореЗрдВ рдбрд╛рд▓ рд░рд╣реЗ рд╣реИрдВред

рдЖрдЦрд┐рд░рдХрд╛рд░, рдКрдкрд░ рдЬреЛ рдХреБрдЫ рднреА рдЙрд▓реНрд▓реЗрдЦ рдХрд┐рдпрд╛ рдЧрдпрд╛ рдерд╛, рд╡рд╣ рдХрд╛рд▓реА рдЦрд╛рдВрд╕реА, рдЯреЗрдЯрдирд╕ рдФрд░ рдбрд┐рдкреНрдереАрд░рд┐рдпрд╛ рдХреЗ рдШрдЯрдХреЛрдВ рджреНрд╡рд╛рд░рд╛ рд╢рд░реАрд░ рдкрд░ рд╣реЛрдиреЗ рд╡рд╛рд▓реА рдкреНрд░рддрд┐рдХреНрд░рд┐рдпрд╛ рдХрд╛ рдПрдХ рдЖрдВрд╢рд┐рдХ рдЕрднрд┐рд╡реНрдпрдХреНрддрд┐ рд╣реИред рдФрд░ рд╡рд╛рдпрд░рд╕ рдЦреБрдж рдмрдЪреНрдЪреЗ рдХреЗ рд╢рд░реАрд░ рдХреЛ рдХреИрд╕реЗ рдкреНрд░рднрд╛рд╡рд┐рдд рдХрд░ рд╕рдХрддрд╛ рд╣реИ рдФрд░ рдпрд╣ рд╕реЛрдЪрдирд╛ рдбрд░рд╛рд╡рдирд╛ рд╣реИред рдЗрд╕рд▓рд┐рдП, рдЯреАрдХрд╛рдХрд░рдг рдХрд┐рдпрд╛ рдЬрд╛рдирд╛ рдЖрд╡рд╢реНрдпрдХ рд╣реИ, рд▓реЗрдХрд┐рди рдЗрд╕реЗ рд╕рд╣реА рдврдВрдЧ рд╕реЗ рдХрд┐рдпрд╛ рдЬрд╛рдирд╛ рдЪрд╛рд╣рд┐рдПред

рдЙрдЪрд┐рдд рддреИрдпрд╛рд░реА

рдПрдХ рдмрдЪреНрдЪреЗ рдХреЛ рдЯреАрдХрд╛ рд▓рдЧрд╛рдиреЗ рдХрд╛ рдирд┐рд░реНрдгрдп рд▓реЗрдиреЗ рдХреЗ рдмрд╛рдж, рдорд╛рддрд╛-рдкрд┐рддрд╛ рдХреЛ рдпрд╣ рд╕рдордЭрдирд╛ рдЪрд╛рд╣рд┐рдП рдХрд┐ рд╢рд░реАрд░ рдореЗрдВ рд╕реАрд░рдо рдХреА рд╢реБрд░реВрдЖрдд рдХреЗ рдмрд╛рдж рд╕рд╣реА рддреИрдпрд╛рд░реА рдФрд░ рдХреНрд░рд┐рдпрд╛рдУрдВ рдХрд╛ рдкрд╛рд▓рди рдХрд░рдирд╛ рдПрдХ рдмрдбрд╝реА рднреВрдорд┐рдХрд╛ рдирд┐рднрд╛рддрд╛ рд╣реИред рдФрд░ рдЕрдЧрд░, рдЯреАрдХрд╛рдХрд░рдг рдХреЗ рдмрд╛рдж, рдмрдЪреНрдЪреЗ рдХреЛ рд╕рдВрд╡реЗрджрдирд╛рд╣рд╛рд░реА, рдЬреНрд╡рд░рдирд╛рд╢рдХ рджреЗрдиреЗ рдХреЗ рд▓рд┐рдП рдкрд░реНрдпрд╛рдкреНрдд рд╣реИ, рддреЛ рддреИрдпрд╛рд░реА рдкреНрд░рдХреНрд░рд┐рдпрд╛ рдореЗрдВ рдЕрдзрд┐рдХ рд╕рдордп рдФрд░ рдкреНрд░рдпрд╛рд╕ рд▓рдЧреЗрдЧрд╛ред

рд▓реЗрдХрд┐рди рд╕рд╛рде рд╣реА, рдпрд╣ рд╡рд┐рд╢реНрд╡рд╛рд╕ рдкреНрд░рджрд╛рди рдХрд░реЗрдЧрд╛ рдХрд┐ рдмрдЪреНрдЪрд╛ рд╕реНрд╡рд╕реНрде рд╣реИ рдФрд░ рдирдХрд╛рд░рд╛рддреНрдордХ рдкрд░рд┐рдгрд╛рдо, рднрд▓реЗ рд╣реА рд╡реЗ рдкреНрд░рдХрдЯ рд╣реЛрдВ, рдХрдо рд╕реЗ рдХрдо рд╣реЛрдВрдЧреЗред рдЖрдк рдирд┐рдореНрдирд▓рд┐рдЦрд┐рдд рдХреЗ рдмрд╛рдж рдЯреАрдХрд╛ рд▓рдЧрд╡рд╛ рд╕рдХрддреЗ рд╣реИрдВ:

  • рдбреЙрдХреНрдЯрд░ рджреНрд╡рд╛рд░рд╛ рдмрдЪреНрдЪреЗ рдХреА рдЬрд╛рдВрдЪ рдХреА рдЧрдИ рдФрд░ рд░рдХреНрдд рдкрд░реАрдХреНрд╖рдг рдХреЗ рд▓рд┐рдП рдЕрдкреЙрдЗрдВрдЯрдореЗрдВрдЯ рджрд┐рдпрд╛ рдЧрдпрд╛, рдЬреЛ рдкрд╛рд╕ рд╣реЛ рдЧрдпрд╛ред
  • рд╡рд┐рд╢реЗрд╖рдЬреНрдЮреЛрдВ рдХреЗ рд╕рд╛рде рдПрдХ рдкреНрд░рд╛рд░рдВрднрд┐рдХ рдкрд░рд╛рдорд░реНрд╢ рдХрд╛ рдкрд╛рд▓рди рдХрд┐рдпрд╛ рдЧрдпрд╛, рдЬрд┐рдиреНрд╣реЛрдВрдиреЗ рдорд╛рддрд╛-рдкрд┐рддрд╛ рдХреЛ рдЗрд╕ рдЯреАрдХрд╛рдХрд░рдг рдХреА рдЖрд╡рд╢реНрдпрдХрддрд╛ рдХреЗ рдмрд╛рд░реЗ рдореЗрдВ рдкреВрд░реА рддрд░рд╣ рд╕реЗ рд╕рдордЭрд╛рдпрд╛, рдЗрд╕рдХреЗ рдХреНрдпрд╛ рдкрд░рд┐рдгрд╛рдо рд╣реЛ рд╕рдХрддреЗ рд╣реИрдВ рдФрд░ рдпрджрд┐ рд╡реЗ рджрд┐рдЦрд╛рдИ рджреЗрддреЗ рд╣реИрдВ рддреЛ рдХреНрдпрд╛ рдХрд░реЗрдВред рдЗрд╕ рдкрд░рд╛рдорд░реНрд╢ рдХреЗ рдмрд╛рдж, рдЙрдирдХреЗ рдкрд╛рд╕ рдХреЛрдИ рдкреНрд░рд╢реНрди рдирд╣реАрдВ рд╣реЛрдирд╛ рдЪрд╛рд╣рд┐рдПред
  • рдбреЙрдХреНрдЯрд░ рд╕реЗ рд╕рд▓рд╛рд╣ рдорд╢рд╡рд┐рд░рд╛ рдХрд┐рдпрд╛ рдЧрдпрд╛ред рдЗрд╕ рдШрдЯрдирд╛ рдореЗрдВ рдХрд┐ рдкрд┐рдЫрд▓реЗ рдЯреАрдХреЗ рд╕реЗ рдмрдЪреНрдЪреЗ рдореЗрдВ рдЬрдЯрд┐рд▓рддрд╛рдПрдБ рдкреИрджрд╛ рд╣реБрдИрдВ, рдЗрд╕реЗ рдЪреБрдк рдирд╣реАрдВ рд░рд╣рдирд╛ рдЪрд╛рд╣рд┐рдПред рдЗрд╕ рдорд╛рдорд▓реЗ рдореЗрдВ, рдбреЙрдХреНрдЯрд░ рдбреАрдкреАрдЯреА рдХреЗ рд╡рд┐рдХрд▓реНрдк рдХрд╛ рдЪрдпрди рдХрд░реЗрдВрдЧреЗ рдФрд░ рд╕рд▓рд╛рд╣ рджреЗрдВрдЧреЗред

рдбреАрдЯреАрдкреА рд╕реЗ рдХреБрдЫ рджрд┐рди рдкрд╣рд▓реЗ рдФрд░ рдЙрд╕рдХреЗ рдмрд╛рдж рдмрдЪреНрдЪреЗ рдХреЗ рд╕рд╛рде рдЙрди рдЬрдЧрд╣реЛрдВ рдкрд░ рдЯрд╣рд▓реЗрдВ рдЬрд╣рд╛рдВ рдмрдбрд╝реА рд╕рдВрдЦреНрдпрд╛ рдореЗрдВ рд▓реЛрдЧ рд░рд╣рддреЗ рд╣реИрдВ, рдЗрд╕реЗ рдХрдо рд╕реЗ рдХрдо рдХрд░рдиреЗ рдХреА рд╕рд┐рдлрд╛рд░рд┐рд╢ рдХреА рдЬрд╛рддреА рд╣реИред рдпрд╣ рд╡рд╛рдпрд░рд╕ рдХреЛ рд╢рд░реАрд░ рдореЗрдВ рдкреНрд░рд╡реЗрд╢ рдХрд░рдиреЗ рд╕реЗ рд░реЛрдХреЗрдЧрд╛ рдЬреЛ рдкреНрд░рддрд┐рд░рдХреНрд╖рд╛ рдкреНрд░рдгрд╛рд▓реА рдХреЛ рдХрдордЬреЛрд░ рдХрд░ рд╕рдХрддреЗ рд╣реИрдВ рдФрд░ рдЯреАрдХрд╛рдХрд░рдг рдХреЗ рдмрд╛рдж рдкрд░рд┐рдгрд╛рдо рджреЗ рд╕рдХрддреЗ рд╣реИрдВред рд╡рд┐рд╢реЗрд╖ рдзреНрдпрд╛рди crumbs рдХреЗ рдкреЛрд╖рдг рдХреЗ рд▓рд┐рдП рджрд┐рдпрд╛ рдЬрд╛рдирд╛ рдЪрд╛рд╣рд┐рдПред рдЯреАрдХрд╛рдХрд░рдг рдХреЗ рдмрд╛рдж рдЕрдЧрд▓реЗ рд╕рдкреНрддрд╛рд╣ рдореЗрдВ, рдЙрд╕рдХреЗ рдЖрд╣рд╛рд░ рдореЗрдВ рдирдП рдЦрд╛рджреНрдп рдкрджрд╛рд░реНрдереЛрдВ рдХреЛ рд╢рд╛рдорд┐рд▓ рдХрд░рдиреЗ рдХреА рд╕рд▓рд╛рд╣ рдирд╣реАрдВ рджреА рдЬрд╛рддреА рд╣реИред


рдЯреБрдХрдбрд╝реЛрдВ рдХреА рд╕реНрдерд┐рддрд┐ рдХреЗ рд▓рд┐рдП, рдЙрд╕рдХреЗ рд╢рд░реАрд░ рдореЗрдВ рд╕реАрд░рдо рдХреА рд╢реБрд░реВрдЖрдд рдХреЗ 3 рджрд┐рдиреЛрдВ рдХреЗ рднреАрддрд░, рдЗрд╕реЗ рд╕рд╛рдорд╛рдиреНрдп рд╣реЛрдирд╛ рдЪрд╛рд╣рд┐рдП - рд╢рд░реАрд░ рдХрд╛ рддрд╛рдкрдорд╛рди рд╕рд╛рдорд╛рдиреНрдп рд╣реЛ рдЬрд╛рдПрдЧрд╛, рд╕реВрдЬрди, рд▓рд╛рд▓рд┐рдорд╛ рдХрдо рд╣реЛ рдЬрд╛рдПрдЧреА, рд▓рдВрдЧрдбрд╝рд╛рдкрди рдЧрд╛рдпрдм рд╣реЛ рдЬрд╛рдПрдЧрд╛, рдЕрдЧрд░ рдпрд╣ рдерд╛ред рдЦрд░рдмреВрдЬреЗ рдХреА рд╕реНрдерд┐рддрд┐ рдореЗрдВ рднреА рд╕реБрдзрд╛рд░ рд╣реЛрдЧрд╛ред

рд╡рд╣ рд╣рдВрд╕рдореБрдЦ, рдореБрд╕реНрдХреБрд░рд╛рддреЗ рд╣реБрдП, рд╢рд╛рдВрдд рд╣реЛ рдЬрд╛рдПрдЧрд╛, рд╢рд╛рд╕рди рд╕рд╛рдорд╛рдиреНрдп рд╣реЛ рдЬрд╛рдПрдЧрд╛, рдХреНрдпреЛрдВрдХрд┐ рдЙрд╕реЗ рдЕрдм рдХрд┐рд╕реА рдЪреАрдЬ рдХреА рдЪрд┐рдВрддрд╛ рдирд╣реАрдВ рд╣реИред рдЗрд╕рдХрд╛ рдорддрд▓рдм рд╣реИ рдХрд┐ рд╕реАрд░рдо рдореЗрдВ рдореМрдЬреВрдж рдШрдЯрдХреЛрдВ рдХреЛ рд╢рд░реАрд░ рджреНрд╡рд╛рд░рд╛ рдЖрддреНрдорд╕рд╛рдд рдХрд░ рд▓рд┐рдпрд╛ рдЧрдпрд╛ рд╣реИред

рдмрдЪреНрдЪреЗ рдХреЛ рдЯреАрдХреЗ рдХреА рдЬрд░реВрд░рдд рд╣реИ рдпрд╛ рдирд╣реАрдВ, рдЗрд╕рдХрд╛ рдлреИрд╕рд▓рд╛ рдорд╛рддрд╛-рдкрд┐рддрд╛ рдкрд░ рдирд┐рд░реНрднрд░ рдХрд░рддрд╛ рд╣реИред рд▓реЗрдХрд┐рди рдЙрдиреНрд╣реЗрдВ рдпрд╣ рд╕рдордЭрдирд╛ рдЪрд╛рд╣рд┐рдП рдХрд┐ рдЯреАрдХрд╛рдХрд░рдг рдЕрдиреБрд╕реВрдЪреА рдХрд╛ рдЖрд╡рд┐рд╖реНрдХрд╛рд░ рд╡реНрдпрд░реНрде рдореЗрдВ рдирд╣реАрдВ рдХрд┐рдпрд╛ рдЧрдпрд╛ рдерд╛ред рдЖрдЦрд┐рд░рдХрд╛рд░, рдмрдЪреНрдЪреЛрдВ рдХрд╛ рд╕реНрд╡рд╛рд╕реНрдереНрдп рдЙрдирдХреА рд╕рдордпрдмрджреНрдзрддрд╛ рдкрд░ рдирд┐рд░реНрднрд░ рдХрд░рддрд╛ рд╣реИред

рдпрджрд┐ рдирд┐: рд╢реБрд▓реНрдХ рдЯреАрдХрд╛рдХрд░рдг рдореЗрдВ рд╡рд┐рд╢реНрд╡рд╛рд╕, рдЬреЛ рдЕрдХреНрд╕рд░ рдмрдЪреНрдЪреЛрдВ рдХреЗ рдХреНрд▓реАрдирд┐рдХ рдФрд░ рдХрд┐рдВрдбрд░рдЧрд╛рд░реНрдЯрди рдореЗрдВ рджрд┐рдЦрд╛рдИ рджреЗрддрд╛ рд╣реИ, рдЖрддреНрдорд╡рд┐рд╢реНрд╡рд╛рд╕ рдХреЛ рдкреНрд░реЗрд░рд┐рдд рдирд╣реАрдВ рдХрд░рддрд╛ рд╣реИ, рддреЛ рд╡реИрдХреНрд╕реАрди рдХреЛ рдлрд╛рд░реНрдореЗрд╕реА рдореЗрдВ рд╕рднреА рдкреНрд░рд╛рд╕рдВрдЧрд┐рдХ рджрд╕реНрддрд╛рд╡реЗрдЬреЛрдВ рдХреЗ рд╕рд╛рде рдЦрд░реАрджрд╛ рдЬрд╛ рд╕рдХрддрд╛ рд╣реИред рдЬрдм рдЖрдк рдореБрдлреНрдд рдЯреАрдХреЗ рдХрд╛ рдЙрдкрдпреЛрдЧ рдХрд░рдиреЗ рдХреА рдпреЛрдЬрдирд╛ рдмрдирд╛рддреЗ рд╣реИрдВ рддреЛ рдЖрдкрдХреЛ рджрд╕реНрддрд╛рд╡реЗрдЬрд╝реАрдХрд░рдг рдХреА рдЙрдкрд▓рдмреНрдзрддрд╛ рдореЗрдВ рднреА рд░реБрдЪрд┐ рд╣реЛрдиреА рдЪрд╛рд╣рд┐рдПред

рдорд╛рддрд╛-рдкрд┐рддрд╛ рдХреА рд╕рдореАрдХреНрд╖рд╛рдУрдВ рдХреЗ рдЕрдиреБрд╕рд╛рд░, рдбреАрдкреАрдЯреА рдХреЗ рдкрд░рд┐рдгрд╛рдо рдмрд╣реБрдд рд╡рд┐рд╡рд┐рдз рд╣реЛ рд╕рдХрддреЗ рд╣реИрдВ: рд╣рд▓реНрдХреЗ рд╕реЗ рд▓реЗрдХрд░ рдЕрддреНрдпрдВрдд рдЧрдВрднреАрд░ рддрдХред рдЙрди рдорд╛рдорд▓реЛрдВ рдХреЗ рдмрд╛рд░реЗ рдореЗрдВ рдЖрдкрдХреЛ рдХреНрдпрд╛ рдЬрд╛рдирдиреЗ рдХреА рдЬрд░реВрд░рдд рд╣реИ рдЬрдм рдЯреАрдХрд╛рдХрд░рдг рдореЗрдВ рджреЗрд░реА рд╣реЛ рд░рд╣реА рд╣реИ рддрд╛рдХрд┐ рдЙрдирд╕реЗ рдмрдЪрд╛ рдЬрд╛ рд╕рдХреЗ?

рдбреАрдкреАрдЯреА рдХреЗ рдкрд░рд┐рдгрд╛рдо

рдПрдХ рдЫреЛрдЯреЗ рдмрдЪреНрдЪреЗ рдореЗрдВ, рдбреЙрдХреНрдЯрд░ рдбрд╛рдпрдереЗрд╕рд┐рд╕ рдХреА рдЕрднрд┐рд╡реНрдпрдХреНрддрд┐рдпреЛрдВ рдХреЛ рдиреЛрдЯрд┐рд╕ рдирд╣реАрдВ рдХрд░ рд╕рдХрддрд╛ рд╣реИред рдФрд░ рдХрдИ рдорд╛рддрд╛-рдкрд┐рддрд╛ рдпрд╣ рдирд╣реАрдВ рдЬрд╛рдирддреЗ рд╣реИрдВ рдХрд┐ рдЗрд╕ рдорд╛рдорд▓реЗ рдореЗрдВ рдЯреАрдХрд╛ рд▓рдЧрд╡рд╛рдирд╛ рдЗрд╕рдХреЗ рд▓рд╛рдпрдХ рдирд╣реАрдВ рд╣реИред рдХреНрдпреЛрдВрдХрд┐ рдбреАрдкреАрдЯреА рдЯреАрдХрд╛рдХрд░рдг рд╕рдмрд╕реЗ рдЕрдЪреНрдЫреЗ рдкрд░рд┐рдгрд╛рдо рдирд╣реАрдВ рджреЗ рд╕рдХрддрд╛ рд╣реИред

рдЖрдВрддрд░рд┐рдХ рдЕрдВрдЧ

рдЧреБрд░реНрджреЗ рд╡рд┐рд╢реЗрд╖ рд░реВрдк рд╕реЗ рдкреНрд░рднрд╛рд╡рд┐рдд рд╣реЛрддреЗ рд╣реИрдВред рдпрд╣ рдЗрд╕ рддрдереНрдп рд╕реЗ рдкреНрд░рдХрдЯ рд╣реЛрддрд╛ рд╣реИ рдХрд┐ рдмрдЪреНрдЪреЗ рдХрд╛ рд╢рд░реАрд░ рдЕрдирд╛рд╡рд╢реНрдпрдХ рддрд░рд▓ рдкрджрд╛рд░реНрде рдирд╣реАрдВ рдирд┐рдХрд╛рд▓ рд╕рдХрддрд╛ рд╣реИ, рдФрд░ рдпрд╣ рдЬрдорд╛ рд╣реЛ рдЬрд╛рддрд╛ рд╣реИ рдЕрд▓рдЧ рдХрдкрдбрд╝реЗ, рдЬреЛ рдПрдбрд┐рдорд╛ рдХреЗ рд╡рд┐рдХрд╛рд╕ рдореЗрдВ рдпреЛрдЧрджрд╛рди рджреЗрддрд╛ рд╣реИред

рддрд░рд▓ рд▓рдЧрднрдЧ рд╕рднреА рдЕрдВрдЧреЛрдВ рдХреА рджреАрд╡рд╛рд░реЛрдВ рдореЗрдВ рд╣реЛ рд╕рдХрддрд╛ рд╣реИред рд╕реНрд╡рд╛рднрд╛рд╡рд┐рдХ рд░реВрдк рд╕реЗ, рдЗрд╕ рддрд░рд╣ рдХреЗ рдкрд░рд┐рдгрд╛рдо рдХреЛ рдирдЧреНрди рдЖрдВрдЦреЛрдВ рд╕реЗ рдиреЛрдЯрд┐рд╕ рдХрд░рдирд╛ рдореБрд╢реНрдХрд┐рд▓ рд╣реИред рдХрдИ рдорд╛рдорд▓реЛрдВ рдореЗрдВ, рдРрд╕реА рд╕реВрдЬрди рдкреБрд░рд╛рдиреА рдкрд╛рдЗрд▓реЛрдирдлреНрд░рд╛рдЗрдЯрд┐рд╕ рдХреЗ рд╡рд┐рдХрд╛рд╕ рдХрд╛ рдХрд╛рд░рдг рдмрди рдЬрд╛рддреА рд╣реИред

рдХреБрдЫ рдорд╛рдорд▓реЛрдВ рдореЗрдВ, рдбреАрдкреАрдЯреА рдЗрд╕ рддрдереНрдп рдХреЛ рдЬрдиреНрдо рджреЗ рд╕рдХрддрд╛ рд╣реИ рдХрд┐ рдмрдЪреНрдЪрд╛ рди рдХреЗрд╡рд▓ рдЕрдкрдиреЗ рдкреИрд░реЛрдВ рдкрд░ рдЦрдбрд╝рд╛ рд╣реЛ рд╕рдХрддрд╛ рд╣реИ, рдмрд▓реНрдХрд┐ рдЪрд▓рдиреЗ рдореЗрдВ рднреА рдЕрд╕рдорд░реНрде рд╣реИред рд╡реИрдХреНрд╕реАрди рдХреА рдЬрдЯрд┐рд▓ рд╕рдВрд░рдЪрдирд╛ рдФрд░ рдЗрд╕рдХреА рдЙрдЪреНрдЪ рдкреНрд░рддрд┐рдХреНрд░рд┐рдпрд╛рд╢реАрд▓рддрд╛ рдРрд╕реЗ рдкрд░рд┐рдгрд╛рдо рджреЗ рд╕рдХрддреА рд╣реИред

рд╣рд╛рд▓рд╛рдВрдХрд┐, рдЕрдХреНрд╕рд░ рдХреБрдЫ рджрд┐рдиреЛрдВ рдХреЗ рдмрд╛рдж, рдЬрдм рдЗрдВрдЬреЗрдХреНрд╢рди рд╕рд╛рдЗрдЯ рдкрд░ рджрд░реНрдж рдЧрд╛рдпрдм рд╣реЛ рдЬрд╛рддрд╛ рд╣реИ, рддреЛ рдЪрд▓рдиреЗ рдФрд░ рдкреИрд░ рдкрд░ рд╕рдорд░реНрдерди рдХрд░рдиреЗ рдХреА рдХреНрд╖рдорддрд╛ рдмрд╣рд╛рд▓ рд╣реЛ рдЬрд╛рддреА рд╣реИред

рд▓реЗрдХрд┐рди рджреВрд╕рд░рд╛, рдЗрддрдирд╛ рдЕрдиреБрдХреВрд▓ рдирд╣реАрдВ рдкрд░рд┐рдгрд╛рдо рднреА рд╕рдВрднрд╡ рд╣реИред рдбреАрдкреАрдЯреА рдЯреАрдХрд╛рдХрд░рдг рдХреЗ рдмрд╛рдж рдЗрдВрдЬреЗрдХреНрд╢рди рд╕реНрдерд▓ рдкрд░ рдПрдХ рд╕реАрд▓ рдмрди рд╕рдХрддреА рд╣реИред рдпрд╣ рдЬрд▓реНрджреА рд╕реЗ рдорд╣рддреНрд╡рдкреВрд░реНрдг рдФрд░ рдЖрдХрд╛рд░ рдореЗрдВ рдмрдбрд╝рд╛ рд╣реЛ рдЬрд╛рддрд╛ рд╣реИред

рдереЛрдбрд╝реЗ рд╕рдордп рдореЗрдВ, рджрдорди рд╣реЛ рд╕рдХрддрд╛ рд╣реИ, рдФрд░ рдпрд╣ рдкрд╣рд▓реЗ рд╕реЗ рд╣реА рд╕рд░реНрдЬрди рдХреА рдХреНрд╖рдорддрд╛ рд╣реИред рдЬрд┐рд╕ рдХреНрд╖реЗрддреНрд░ рдореЗрдВ рдорд╡рд╛рдж рдЬрдорд╛ рд╣реЛрддрд╛ рд╣реИ рдЙрд╕реЗ рд╕рд╛рдл рдХрд┐рдпрд╛ рдЬрд╛рддрд╛ рд╣реИ, рдФрд░ рдлрд┐рд░ рдПрдХ рдирд╛рд▓реА рд░рдЦреА рдЬрд╛рддреА рд╣реИред

рд╕реНрд╡рд╛рднрд╛рд╡рд┐рдХ рд░реВрдк рд╕реЗ, рдпрд╣ рдмрдЪреНрдЪреЗ рдХреЗ рд▓рд┐рдП рдмрд╣реБрдд рджрд░реНрджрдирд╛рдХ рд╣реИ, рдЗрд╕рдХреЗ рд╕рд╛рде рднреВрдЦ рдФрд░ рдиреАрдВрдж рдореЗрдВ рдЧрд┐рд░рд╛рд╡рдЯ рд╣реЛ рд╕рдХрддреА рд╣реИред рдПрдХ рд╡рд░реНрд╖ рд╕реЗ рдХрдо рдЙрдореНрд░ рдХреЗ рдмрдЪреНрдЪреЛрдВ рдХреЗ рд▓рд┐рдП, рдбреАрдкреАрдЯреА рдЯреАрдХрд╛рдХрд░рдг рдХреЗ рдмрд╛рдж рдХреЗ рдкрд░рд┐рдгрд╛рдо рдмрд╣реБрдд рднрдпрд╛рдирдХ рд╣реЛ рд╕рдХрддреЗ рд╣реИрдВред рдЖрдЦрд╝рд┐рд░рдХрд╛рд░ рдЫреЛрдЯрд╛ рдмрдЪреНрдЪрд╛рдпрд╣ рдирд╣реАрдВ рдмрддрд╛ рд╕рдХрддрд╛ рдХрд┐ рдХреНрдпрд╛ рдФрд░ рдХрд╣рд╛рдБ рджрд░реНрдж рд╣реЛрддрд╛ рд╣реИред

рдореБрдЦреНрдп рдмрд╛рдд рдЗрд╕ рдкрд▓ рдХреЛ рдпрд╛рдж рдирд╣реАрдВ рдХрд░рдирд╛ рд╣реИред рдФрд░ рдЬреИрд╕реЗ рд╣реА рджрдорди рдХреЗ рдЧрдарди рдХрд╛ рд╕рдВрджреЗрд╣ рд╣реЛрддрд╛ рд╣реИ, рдбреЙрдХреНрдЯрд░ рд╕реЗ рдкрд░рд╛рдорд░реНрд╢ рд▓реЗрдВред рд╡рд┐рд░реЛрдзреА рднрдбрд╝рдХрд╛рдК рдорд▓рд╣рдо рдХреЗ рд╕рд╛рде рд╕рд╛рдордирд╛ рдХрд░рдирд╛ рд╕рдВрднрд╡ рд╣реЛ рд╕рдХрддрд╛ рд╣реИ рдЬреЛ рд╕реАрд▓ рдХреЛ рднрдВрдЧ рдХрд░рдиреЗ рдореЗрдВ рдорджрдж рдХрд░реЗрдЧрд╛ред рдЗрд╕рд╕реЗ рд╕рд░реНрдЬрд░реА рд╕реЗ рдмрдЪрдиреЗ рдореЗрдВ рдорджрдж рдорд┐рд▓реЗрдЧреАред

рдЯреАрдХрд╛рдХрд░рдг рдХреЗ рдмрд╛рдж рдЗрдВрд╕реЗрдлреЗрд▓рд╛рдЗрдЯрд┐рд╕

рдХреБрдЫ рдмрдЪреНрдЪреЛрдВ рдореЗрдВ, рдЯреАрдХрд╛рдХрд░рдг рдХреЗ рдмрд╛рдж, рдРрд╕реА рдкреНрд░рддрд┐рдХреНрд░рд┐рдпрд╛ рд╢реБрд░реВ рд╣реЛрддреА рд╣реИ, рдЬреЛ рдПрдХ рдордЬрдмреВрдд рд╣рд┐рд╕реНрдЯреЗрд░рд┐рдХрд▓ рд░реЛрдирд╛ рдХреЗ рд╕рд╛рде рд╣реЛрддреА рд╣реИ, рдЬреЛ рд▓рдВрдмреЗ рд╕рдордп рддрдХ рдЬрд╛рд░реА рд░рд╣ рд╕рдХрддреА рд╣реИред рдХрднреА-рдХрднреА рдпреЗ рд▓рдХреНрд╖рдг рд╢реБрд░реВ рд╣реЛрддреЗ рд╣реА рдЧрд╛рдпрдм рд╣реЛ рдЬрд╛рддреЗ рд╣реИрдВред

рдбреЙрдХреНрдЯрд░ рдЯреАрдХрд╛рдХрд░рдг рд╕реНрдерд▓ рдкрд░ рджрд░реНрдж рдФрд░ рдЕрддреНрдпрдзрд┐рдХ рд╕рдВрд╡реЗрджрдирд╢реАрд▓рддрд╛ рдХреЗ рд▓рд┐рдП рд╣рд░ рдЪреАрдЬ рдХрд╛ рд╢реНрд░реЗрдп рджреЗрддреЗ рд╣реИрдВред рд▓реЗрдХрд┐рди рдПрдХ рдмрдЪреНрдЪреЗ рдХрд╛ рд▓рдВрдмреЗ рд╕рдордп рддрдХ рд░реЛрдирд╛ рдЯреАрдХрд╛рдХрд░рдг рдХреЗ рдмрд╛рдж рдХреЗ рдПрдиреНрд╕реЗрдлрд▓рд╛рдЗрдЯрд┐рд╕ рдЬреИрд╕реЗ рдЧрдВрднреАрд░ рдкрд░рд┐рдгрд╛рдо рдХрд╛ рд╕рдВрдХреЗрдд рджреЗ рд╕рдХрддрд╛ рд╣реИред

рдЯреАрдХрд╛рдХрд░рдг рдХреЗ рдмрд╛рдж рдПрдиреНрд╕реЗрдлрд▓рд╛рдЗрдЯрд┐рд╕ рдорд╕реНрддрд┐рд╖реНрдХ рд╕рдВрд░рдЪрдирд╛рдУрдВ рдХреА рдЧрддрд┐рд╡рд┐рдзрд┐ рдореЗрдВ рдкрд░рд┐рд╡рд░реНрддрди рдХрд╛ рдХрд╛рд░рдг рдмрди рд╕рдХрддрд╛ рд╣реИ, рдпрд╣рд╛рдВ рддрдХ тАЛтАЛрдХрд┐ рдЖрддреНрдордХреЗрдВрджреНрд░рд┐рдд рднреА рд╣реЛ рд╕рдХрддрд╛ рд╣реИред рдЗрд╕ рдорд╛рдорд▓реЗ рдореЗрдВ, рдмрдЪреНрдЪрд╛ рднрд╛рд╡рдирд╛рдУрдВ рдХреЛ рдЦрд░рд╛рдм рддрд░реАрдХреЗ рд╕реЗ рд╡реНрдпрдХреНрдд рдХрд░рддрд╛ рд╣реИ, рдкреНрд░рд┐рдпрдЬрдиреЛрдВ рдХреЛ рдкрд╣рдЪрд╛рдирдирд╛ рдмрдВрдж рдХрд░ рджреЗрддрд╛ рд╣реИред рд╕рдВрдЪрд╛рд░ рдХрд░рдиреЗ рдХреА рдХреНрд╖рдорддрд╛ рдХреЗ рд▓рд┐рдП рдЬрд┐рдореНрдореЗрджрд╛рд░ рдХрд╛рд░реНрдп рдмрд┐рдЧрдбрд╝рд╛ рд╣реБрдЖ рд╣реИред рднрд╛рд╖рдг рдХреЗрдВрджреНрд░ рднреА рдкреНрд░рднрд╛рд╡рд┐рдд рд╣реЛ рд╕рдХрддрд╛ рд╣реИ, рдФрд░ рдпрд╣ рд╕реАрдЦрдиреЗ рдХреА рдХреНрд╖рдорддрд╛ рдХреЛ рдкреНрд░рднрд╛рд╡рд┐рдд рдХрд░рддрд╛ рд╣реИред

рддреНрд╡рдЪрд╛ рдХреА рдкреНрд░рддрд┐рдХреНрд░рд┐рдпрд╛рдПрдВ

рдПрдХ рдФрд░ рдЧрдВрднреАрд░ рдкрд░рд┐рдгрд╛рдо рдЬреЛ рдбреАрдкреАрдЯреА рдХрд╛ рдХрд╛рд░рдг рдмрди рд╕рдХрддрд╛ рд╣реИ рд╡рд╣ рдПрдЯреЛрдкрд┐рдХ рдЬрд┐рд▓реНрдж рдХреА рд╕реВрдЬрди рдпрд╛ рдиреНрдпреВрд░реЛрдбрд░реНрдорд╛реЗрдЯрд╛рдЗрдЯрд┐рд╕ рдХреА рдШрдЯрдирд╛ рд╕реЗ рдЬреБрдбрд╝рд╛ рд╣реИред

рдЗрди рддреНрд╡рдЪрд╛ рд╕реНрдерд┐рддрд┐рдпреЛрдВ рдХреЗ рд▓рд┐рдП рд╕реЗ рдЕрдзрд┐рдХ рдХреА рдЖрд╡рд╢реНрдпрдХрддрд╛ рд╣реЛрддреА рд╣реИ рджрд╡рд╛ рд╕реЗ рдЗрд▓рд╛рдЬ, рд▓реЗрдХрд┐рди рдЙрди рдЦрд╛рджреНрдп рдкрджрд╛рд░реНрдереЛрдВ рдХрд╛ рднреА рдкрддрд╛ рд▓рдЧрд╛рдирд╛ рдЬреЛ рдПрд▓рд░реНрдЬреА рдХреА рдкреНрд░рддрд┐рдХреНрд░рд┐рдпрд╛ рдХреЛ рднрдбрд╝рдХрд╛ рд╕рдХрддреЗ рд╣реИрдВред рд╕реВрдЬрди рд╡рд╛рд▓реА рддреНрд╡рдЪрд╛ рдХрд╛ рдЗрд▓рд╛рдЬ рд╡рд┐рд╢реЗрд╖ рд╕реМрдВрджрд░реНрдп рдкреНрд░рд╕рд╛рдзрдиреЛрдВ рд╕реЗ рдХрд┐рдпрд╛ рдЬрд╛рдирд╛ рдЪрд╛рд╣рд┐рдПред

"" рдХрд╛ рдХреНрдпрд╛ рдЕрд░реНрде рд╣реИ рдФрд░ рдЗрд╕реЗ рдХреИрд╕реЗ рдХрд░рдирд╛ рд╣реИ?

рд╡рд┐рд╡рд░рдг рдПрдХ рдЕрд▓рдЧ рдкреГрд╖реНрда рдкрд░ред рдХреНрдпрд╛ рдЖрдк рдЬрд╛рдирддреЗ рд╣реИрдВ рдирд╡рдЬрд╛рдд рд╢рд┐рд╢реБ рдХреЛ рдХреИрд╕реЗ рдирд╣рд▓рд╛рдПрдВ? рдЕрдЧрд░ рдирд╣реАрдВ, рддреЛ рд╣рдордиреЗ рдЖрдкрдХреЗ рд▓рд┐рдП рдПрдХ рдЖрд░реНрдЯрд┐рдХрд▓ рддреИрдпрд╛рд░ рдХрд┐рдпрд╛ рд╣реИред

рдкреГрд╖реНрда рдкрд░ рджреА рдЧрдИ рддрд╛рд▓рд┐рдХрд╛ рдореЗрдВ рджрд┐рдЦрд╛рдпрд╛ рдЬрд╛рдПрдЧрд╛ рдХрд┐ 6 рдорд╣реАрдиреЗ рдореЗрдВ рдХреМрди рд╕реЗ рдкреВрд░рдХ рдЦрд╛рджреНрдп рдкрджрд╛рд░реНрде рдкреЗрд╢ рдХрд┐рдП рдЬрд╛рдиреЗ рдЪрд╛рд╣рд┐рдПред

рдорд╛рддрд╛-рдкрд┐рддрд╛ рдХреЗ рд▓рд┐рдП, рдпрд╣ рд╕рднреА рдЕрддрд┐рд░рд┐рдХреНрдд рд╡рд┐рддреНрддреАрдп рдЦрд░реНрдЪ рд╣реИрдВ, рдФрд░ рдмрдЪреНрдЪреЗ рдХреЗ рд▓рд┐рдП, рдЦреБрдЬрд▓реА рдФрд░ рдЫреАрд▓рдиреЗ рд╕реЗ рдкреАрдбрд╝рд╛, рдЬреЛ рдЬрд┐рд▓реНрдж рдХреА рд╕реВрдЬрди рд╕реНрд╡рдпрдВ рдкреНрд░рдХрдЯ рд╣реЛрддреА рд╣реИред

рдбреАрдкреАрдЯреА рдХреЗ рдмрд╛рдж рдХреЗ рдкрд░рд┐рдгрд╛рдо рдмрдЪреНрдЪреЗ рдХреЗ рд╕реНрд╡рд╛рд╕реНрдереНрдп рдХреЛ рдЧрдВрднреАрд░ рд░реВрдк рд╕реЗ рдиреБрдХрд╕рд╛рди рдкрд╣реБрдВрдЪрд╛ рд╕рдХрддреЗ рд╣реИрдВред рд╣рд╛рд▓рд╛рдВрдХрд┐, рдЖрдкрдХреЛ рдпрд╣ рд╕рдордЭрдиреЗ рдХреА рдЬрд░реВрд░рдд рд╣реИ рдХрд┐ рдХрд╛рд▓реА рдЦрд╛рдВрд╕реА, рдбрд┐рдкреНрдереАрд░рд┐рдпрд╛ рдФрд░ рдЯреЗрдЯрдирд╕ рдХреЗ рдорд╛рдорд▓реЗ рдореЗрдВ, рдкрд░рд┐рдгрд╛рдо рдХреА рд╕рдВрднрд╛рд╡рдирд╛ рдХрдИ рдЧреБрдирд╛ рдмрдврд╝ рдЬрд╛рддреА рд╣реИред

рдФрд░ рд╢рд░реАрд░ рдХреА рдЧрдВрднреАрд░ рдкреНрд░рддрд┐рдХреНрд░рд┐рдпрд╛рдПрдВ рдмрд╣реБрдд рдЕрдзрд┐рдХ рд╕рд╛рдорд╛рдиреНрдп рд╣реИрдВред

рдмреЗрд╢рдХ, рдЯреАрдХрд╛рдХрд░рдг рдЖрд╡рд╢реНрдпрдХ рд╣реИ, рд▓реЗрдХрд┐рди рдЗрд╕реЗ рдРрд╕реЗ рд╕рдордп рдореЗрдВ рдХрд┐рдпрд╛ рдЬрд╛рдирд╛ рдЪрд╛рд╣рд┐рдП рдЬрдм рдмрдЪреНрдЪрд╛ рдмрд┐рд▓реНрдХреБрд▓ рд╕реНрд╡рд╕реНрде рд╣реЛред

рдХреЛрдИ рднреА рдкреНрд░рддрд┐рдХреНрд░рд┐рдпрд╛ рдЬреЛ рдорд╛рддрд╛-рдкрд┐рддрд╛ рдореЗрдВ рдереЛрдбрд╝реА рд╕реА рднреА рд╢рдВрдХрд╛ рдкреИрджрд╛ рдХрд░рддреА рд╣реИ, рдЙрд╕реЗ рддреБрд░рдВрдд рдмрд╛рд▓ рд░реЛрдЧ рд╡рд┐рд╢реЗрд╖рдЬреНрдЮ рдХреЛ рдмрддрд╛рдирд╛ рдЪрд╛рд╣рд┐рдПред рдпрджрд┐ рдЯреАрдХрд╛рдХрд░рдг рдХреЗ рдмрд╛рдж рдХреБрдЫ рдЧрд▓рдд рд╣реЛ рдЬрд╛рддрд╛ рд╣реИ, рддреЛ рдЖрдкрдХреЛ рддреБрд░рдВрдд рдХрд┐рд╕реА рд╡рд┐рд╢реЗрд╖рдЬреНрдЮ рд╕реЗ рд╕рдВрдкрд░реНрдХ рдХрд░рдирд╛ рдЪрд╛рд╣рд┐рдПред
рдХреНрдпрд╛ рдЖрдкрдХреЛ рд▓реЗрдЦ рдкрд╕рдВрдж рдЖрдпрд╛?рдЖрд░рдПрд╕рдПрд╕ рдХреЗ рдорд╛рдзреНрдпрдо рд╕реЗ рд╕рд╛рдЗрдЯ рдЕрдкрдбреЗрдЯ рдХреА рд╕рджрд╕реНрдпрддрд╛ рд▓реЗрдВ, рдпрд╛ рдмрдиреЗ рд░рд╣реЗрдВ

рдШреВрд╕рдПрдХ рдЗрдореНрдпреБрдиреЛрдмрд╛рдпреЛрд▓реЙрдЬрд┐рдХрд▓ рджрд╡рд╛ рд╣реИ рдЬрд┐рд╕реЗ рдХреБрдЫ рд╕рдВрднрд╛рд╡рд┐рдд рдЦрддрд░рдирд╛рдХ рд╕рдВрдХреНрд░рд╛рдордХ рд░реЛрдЧреЛрдВ рдХреЗ рд▓рд┐рдП рд▓рдЧрд╛рддрд╛рд░ рдкреНрд░рддрд┐рд░рдХреНрд╖рд╛ рдмрдирд╛рдиреЗ рдХреЗ рд▓рд┐рдП рд╢рд░реАрд░ рдореЗрдВ рдкреЗрд╢ рдХрд┐рдпрд╛ рдЬрд╛рддрд╛ рд╣реИред рдпрд╣ рдЙрдирдХреЗ рдЧреБрдгреЛрдВ рдФрд░ рдЙрджреНрджреЗрд╢реНрдп рдХреЗ рдХрд╛рд░рдг рд╣реИ рдХрд┐ рдЯреАрдХрд╛рдХрд░рдг рд╢рд░реАрд░ рд╕реЗ рдХреБрдЫ рдкреНрд░рддрд┐рдХреНрд░рд┐рдпрд╛рдПрдВ рдкреИрджрд╛ рдХрд░рдиреЗ рдореЗрдВ рд╕рдХреНрд╖рдо рд╣реИрдВред рдРрд╕реА рдкреНрд░рддрд┐рдХреНрд░рд┐рдпрд╛рдУрдВ рдХрд╛ рдкреВрд░рд╛ рд╕реЗрдЯ рджреЛ рд╢реНрд░реЗрдгрд┐рдпреЛрдВ рдореЗрдВ рдмрд╛рдВрдЯрд╛ рдЧрдпрд╛ рд╣реИ:
1. рдЯреАрдХрд╛рдХрд░рдг рдХреЗ рдмрд╛рдж рдкреНрд░рддрд┐рдХреНрд░рд┐рдпрд╛ (рдкреАрд╡реАрдЖрд░)ред
2. рдЯреАрдХрд╛рдХрд░рдг рдХреЗ рдмрд╛рдж рдХреА рдЬрдЯрд┐рд▓рддрд╛рдПрдВ (рдкреАрд╡реАрдУ)ред

рдЯреАрдХрд╛рдХрд░рдг рдХреЗ рдмрд╛рдж рдХреА рдкреНрд░рддрд┐рдХреНрд░рд┐рдпрд╛рдПрдВрдмрдЪреНрдЪреЗ рдХреА рд╕реНрдерд┐рддрд┐ рдореЗрдВ рд╡рд┐рднрд┐рдиреНрди рдкрд░рд┐рд╡рд░реНрддрдиреЛрдВ рдХрд╛ рдкреНрд░рддрд┐рдирд┐рдзрд┐рддреНрд╡ рдХрд░рддреЗ рд╣реИрдВ рдЬреЛ рдкрд░рд┐рдЪрдп рдХреЗ рдмрд╛рдж рд╡рд┐рдХрд╕рд┐рдд рд╣реЛрддреЗ рд╣реИрдВ рдЯреАрдХреЗ, рдФрд░ рдереЛрдбрд╝реЗ рд╕рдордп рдХреЗ рднреАрддрд░ рдЕрдкрдиреЗ рдЖрдк рдЧреБрдЬрд░ рдЬрд╛рддреЗ рд╣реИрдВред рд╢рд░реАрд░ рдореЗрдВ рдкрд░рд┐рд╡рд░реНрддрди рдЬреЛ рдЯреАрдХрд╛рдХрд░рдг рдХреЗ рдмрд╛рдж рдХреА рдкреНрд░рддрд┐рдХреНрд░рд┐рдпрд╛рдУрдВ рдХреЗ рд░реВрдк рдореЗрдВ рдпреЛрдЧреНрдп рд╣реИрдВ, рдЕрд╕реНрдерд┐рд░ рд╣реИрдВ, рд╡рд┐рд╢реБрджреНрдз рд░реВрдк рд╕реЗ рдХрд╛рд░реНрдпрд╛рддреНрдордХ рд╣реИрдВ, рдХреЛрдИ рдЦрддрд░рд╛ рдкреИрджрд╛ рдирд╣реАрдВ рдХрд░рддреЗ рд╣реИрдВ рдФрд░ рд╕реНрдерд╛рдпреА рд╕реНрд╡рд╛рд╕реНрдереНрдп рдХреНрд╖рддрд┐ рдХрд╛ рдХрд╛рд░рдг рдирд╣реАрдВ рдмрдирддреЗ рд╣реИрдВред

рдЯреАрдХрд╛рдХрд░рдг рдХреЗ рдмрд╛рдж рдХреА рдЬрдЯрд┐рд▓рддрд╛рдПрдВрдорд╛рдирд╡ рд╢рд░реАрд░ рдореЗрдВ рд▓рдЧрд╛рддрд╛рд░ рд╣реЛрдиреЗ рд╡рд╛рд▓реЗ рдкрд░рд┐рд╡рд░реНрддрди рд╣реИрдВ рдЬреЛ рдЯреАрдХреЗ рдХреА рд╢реБрд░реВрдЖрдд рдХреЗ рдмрд╛рдж рд╣реБрдП рд╣реИрдВред рдЗрд╕ рдорд╛рдорд▓реЗ рдореЗрдВ, рдЙрд▓реНрд▓рдВрдШрди рджреАрд░реНрдШрдХрд╛рд▓рд┐рдХ рд╣реИрдВ, рд╢рд╛рд░реАрд░рд┐рдХ рдорд╛рдирджрдВрдб рд╕реЗ рдХрд╛рдлреА рдЖрдЧреЗ рдЬрд╛рддреЗ рд╣реИрдВ рдФрд░ рдорд╛рдирд╡ рд╕реНрд╡рд╛рд╕реНрдереНрдп рдХреЗ рд╡рд┐рднрд┐рдиреНрди рд╡рд┐рдХрд╛рд░реЛрдВ рдХреЛ рдЬрдиреНрдо рджреЗрддреЗ рд╣реИрдВред рдЖрдЗрдП рдЕрдзрд┐рдХ рд╡рд┐рд╕реНрддрд╛рд░ рд╕реЗ рдЯреАрдХрд╛рдХрд░рдг рдХреА рд╕рдВрднрд╛рд╡рд┐рдд рдЬрдЯрд┐рд▓рддрд╛рдУрдВ рдкрд░ рд╡рд┐рдЪрд╛рд░ рдХрд░реЗрдВред

рдЯреАрдХрд╛рдХрд░рдг рдХреЗ рдмрд╛рдж рдХреА рдЬрдЯрд┐рд▓рддрд╛рдПрдВ рд╡рд┐рд╖рд╛рдХреНрдд (рдЕрд╕рд╛рдорд╛рдиреНрдп рд░реВрдк рд╕реЗ рдордЬрдмреВрдд), рдПрд▓рд░реНрдЬреА, рддрдВрддреНрд░рд┐рдХрд╛ рддрдВрддреНрд░ рдХреЗ рд╡рд┐рдХрд╛рд░реЛрдВ рдХреЗ рд▓рдХреНрд╖рдгреЛрдВ рдФрд░ рджреБрд░реНрд▓рдн рд░реВрдкреЛрдВ рдХреЗ рд╕рд╛рде рд╣реЛ рд╕рдХрддреА рд╣реИрдВред рдЯреАрдХрд╛рдХрд░рдг рдХреЗ рдмрд╛рдж рдХреА рдЬрдЯрд┐рд▓рддрд╛ рдХреЛ рдЯреАрдХрд╛рдХрд░рдг рдХреЗ рдмрд╛рдж рдХреА рдЕрд╡рдзрд┐ рдХреЗ рдЬрдЯрд┐рд▓ рдкрд╛рдареНрдпрдХреНрд░рдо рд╕реЗ рдЕрд▓рдЧ рдХрд┐рдпрд╛ рдЬрд╛рдирд╛ рдЪрд╛рд╣рд┐рдП, рдЬрдм рд╡рд┐рднрд┐рдиреНрди рд╡рд┐рдХреГрддрд┐ рдХрд╛ рдкрддрд╛ рдЪрд▓рддрд╛ рд╣реИ рдЬреЛ рдЯреАрдХрд╛рдХрд░рдг рдХреЗ рд╕рд╛рде рдПрдХ рд╕рд╛рде рд╣реЛрддреЗ рд╣реИрдВ, рд▓реЗрдХрд┐рди рдХрд┐рд╕реА рднреА рддрд░рд╣ рд╕реЗ рдЗрд╕рд╕реЗ рдЬреБрдбрд╝реЗ рдирд╣реАрдВ рд╣реЛрддреЗ рд╣реИрдВред

рдмрдЪреНрдЪреЛрдВ рдореЗрдВ рдЯреАрдХрд╛рдХрд░рдг рдХреЗ рдмрд╛рдж рдЬрдЯрд┐рд▓рддрд╛рдПрдВ

рдкреНрд░рддреНрдпреЗрдХ рдЯреАрдХрд╛ рдЕрдкрдиреА рдЬрдЯрд┐рд▓рддрд╛ рдкреИрджрд╛ рдХрд░ рд╕рдХрддрд╛ рд╣реИред рд▓реЗрдХрд┐рди рд╕рднреА рдЯреАрдХреЛрдВ рдореЗрдВ рдЬрдЯрд┐рд▓рддрд╛рдПрдВ рднреА рд╣реЛрддреА рд╣реИрдВ рдЬреЛ рдмрдЪреНрдЪреЛрдВ рдХреЛ рд╡рд┐рдХрд╕рд┐рдд рд╣реЛ рд╕рдХрддреА рд╣реИрдВред рдЗрдирдореЗрдВ рдирд┐рдореНрдирд▓рд┐рдЦрд┐рдд рд╢рд░реНрддреЗрдВ рд╢рд╛рдорд┐рд▓ рд╣реИрдВ:
  • рдПрдирд╛рдлрд┐рд▓реЗрдХреНрдЯрд┐рдХ рд╢реЙрдХ, рдЬреЛ рд╡реИрдХреНрд╕реАрди рдХреЗ рдкреНрд░рд╢рд╛рд╕рди рдХреЗ рдПрдХ рджрд┐рди рдХреЗ рднреАрддрд░ рд╡рд┐рдХрд╕рд┐рдд рд╣реЛрддрд╛ рд╣реИ;
  • рдкреВрд░реЗ рд╢рд░реАрд░ рд╕реЗ рдЬреБрдбрд╝реА рдПрд▓рд░реНрдЬреА рдкреНрд░рддрд┐рдХреНрд░рд┐рдпрд╛рдПрдВ - рдХреНрд╡рд┐рдиреНрдХреЗ рдХреА рдПрдбрд┐рдорд╛, рд╕реНрдЯреАрд╡рди-рдЬреЙрдирд╕рди рд╕рд┐рдВрдбреНрд░реЛрдо, рд▓рд┐рдПрд▓ рд╕рд┐рдВрдбреНрд░реЛрдо, рдЖрджрд┐;
  • рд╕реАрд░рдо рдмреАрдорд╛рд░реА;
  • рдорд╕реНрддрд┐рд╖реНрдХрд╛рд╡рд░рдг рд╢реЛрде;
  • рдиреНрдпреВрд░рд┐рдЯрд┐рд╕;
  • рдкреЛрд▓рд┐рдиреЗрд░рд┐рдЯрд┐рд╕ - рдЧреБрдЗрд▓реЗрди-рдмреИрд░реЗ рд╕рд┐рдВрдбреНрд░реЛрдо;
  • рдЖрдХреНрд╖реЗрдк рдЬреЛ рдХрдо рд╢рд░реАрд░ рдХреЗ рддрд╛рдкрдорд╛рди рдХреА рдкреГрд╖реНрдарднреВрдорд┐ рдХреЗ рдЦрд┐рд▓рд╛рдл рд╡рд┐рдХрд╕рд┐рдд рд╣реЛрддреЗ рд╣реИрдВ - 38.5 o рд╕реЗ рдХрдо, рдЬреЛ рдЯреАрдХрд╛рдХрд░рдг рдХреЗ рдмрд╛рдж рдПрдХ рд╡рд░реНрд╖ рдХреЗ рднреАрддрд░ рддрдп рд╣реЛрддреЗ рд╣реИрдВ;
  • рд╕рдВрд╡реЗрджрдирд╢реАрд▓рддрд╛ рдХрд╛ рдЙрд▓реНрд▓рдВрдШрди;
  • рд╡реИрдХреНрд╕реАрди рд╕реЗ рдЬреБрдбрд╝реЗ рдкреЛрд▓рд┐рдпреЛрдорд╛рдЗрд▓рд╛рдЗрдЯрд┐рд╕;
  • рдкреВрд░реЗ рд╢рд░реАрд░ рдХреА рдЫреЛрдЯреА рд░рдХреНрдд рдзрдордирд┐рдпреЛрдВ рдореЗрдВ рд░рдХреНрдд рдХреЗ рдердХреНрдХреЗ рдЬрдордирд╛;
  • рд╣рд╛рдЗрдкреЛрдкреНрд▓рд╛рд╕реНрдЯрд┐рдХ;
  • рдХреЛрд▓реЗрдЬрдиреЛрдЬрд╝;
  • рд░рдХреНрдд рдореЗрдВ рд▓реНрдпреВрдХреЛрд╕рд╛рдЗрдЯреНрд╕ рдХреА рд╕рдВрдЦреНрдпрд╛ рдореЗрдВ рдХрдореА;
  • рдЗрдВрдЬреЗрдХреНрд╢рди рд╕реНрдерд▓ рдкрд░ рдПрдХ рдлреЛрдбрд╝рд╛ рдпрд╛ рдЕрд▓реНрд╕рд░;
  • рд▓рд┐рдореНрдлреИрдбреЗрдирд╛рдЗрдЯрд┐рд╕ - рд▓рд╕реАрдХрд╛ рдирд▓рд┐рдХрд╛рдУрдВ рдХреА рд╕реВрдЬрди;
  • рдУрд╕реНрдЯрд┐рдЯрд┐рд╕ - рд╣рдбреНрдбрд┐рдпреЛрдВ рдХреА рд╕реВрдЬрди;
  • рдХреЗрд▓реЛрдЗрдб рдирд┐рд╢рд╛рди;
  • рд▓рдЧрд╛рддрд╛рд░ рдХрдо рд╕реЗ рдХрдо 3 рдШрдВрдЯреЗ рддрдХ рдмрдЪреНрдЪреЗ рдХрд╛ рд░реЛрдирд╛;
  • рдЕрдЪрд╛рдирдХ рдореМрддред
рдпреЗ рдЬрдЯрд┐рд▓рддрд╛рдПрдВ рд╡рд┐рднрд┐рдиреНрди рдЯреАрдХрд╛рдХрд░рдгреЛрдВ рдХреЗ рдмрд╛рдж рд╡рд┐рдХрд╕рд┐рдд рд╣реЛ рд╕рдХрддреА рд╣реИрдВред рдЯреАрдХрд╛рдХрд░рдг рдХреЗ рдкрд░рд┐рдгрд╛рдорд╕реНрд╡рд░реВрдк рдЙрдирдХреА рдЙрдкрд╕реНрдерд┐рддрд┐ рд╕реАрдорд┐рдд рд╕рдордп рдХреЗ рднреАрддрд░ рд╣реА рд╕рдВрднрд╡ рд╣реИ, рдЬрд┐рд╕реЗ рд╡рд┐рд╢реНрд╡ рд╕реНрд╡рд╛рд╕реНрдереНрдп рд╕рдВрдЧрдарди рджреНрд╡рд╛рд░рд╛ рд╕рд╛рд╡рдзрд╛рдиреАрдкреВрд░реНрд╡рдХ рд╕рддреНрдпрд╛рдкрд┐рдд рдФрд░ рд╡рд┐рдирд┐рдпрдорд┐рдд рдХрд┐рдпрд╛ рдЬрд╛рддрд╛ рд╣реИред рдирд┐рд░реНрджрд┐рд╖реНрдЯ рд╕рдордп рдЕрд╡рдзрд┐ рдХреЗ рдмрд╛рд╣рд░ рдЙрдкрд░реЛрдХреНрдд рд╡рд┐рдХреГрддрд┐ рдХреЗ рдкреНрд░рдХрдЯ рд╣реЛрдиреЗ рдХрд╛ рдЕрд░реНрде рд╣реИ рдХрд┐ рдЙрдирдХрд╛ рдЯреАрдХрд╛рдХрд░рдг рд╕реЗ рдХреЛрдИ рд▓реЗрдирд╛-рджреЗрдирд╛ рдирд╣реАрдВ рд╣реИред

рдмрдЪреНрдЪреЛрдВ рдореЗрдВ рдЯреАрдХрд╛рдХрд░рдг рдХреА рдЬрдЯрд┐рд▓рддрд╛рдПрдБ рдФрд░ рджреБрд╖реНрдкреНрд░рднрд╛рд╡ - рд╡реАрдбрд┐рдпреЛ

рдЯреАрдХрд╛рдХрд░рдг рдХреЗ рдмрд╛рдж рдЬрдЯрд┐рд▓рддрд╛рдУрдВ рдХреЗ рдореБрдЦреНрдп рдХрд╛рд░рдг

рдЯреАрдХрд╛рдХрд░рдг рдХреЗ рдмрд╛рдж рдЬрдЯрд┐рд▓рддрд╛рдПрдВ рдирд┐рдореНрдирд▓рд┐рдЦрд┐рдд рдХрд╛рд░рдгреЛрдВ рдореЗрдВ рд╕реЗ рдПрдХ рдХреЗ рдХрд╛рд░рдг рд╣реЛ рд╕рдХрддреА рд╣реИрдВ:
  • contraindications рдХреА рдЙрдкрд╕реНрдерд┐рддрд┐ рдореЗрдВ рдПрдХ рдЯреАрдХрд╛ рдХрд╛ рдкреНрд░рд╢рд╛рд╕рди;
  • рдЕрдиреБрдЪрд┐рдд рдЯреАрдХрд╛рдХрд░рдг;
  • рдЯреАрдХреЗ рдХреА рддреИрдпрд╛рд░реА рдХреА рдЦрд░рд╛рдм рдЧреБрдгрд╡рддреНрддрд╛;
  • рдорд╛рдирд╡ рд╢рд░реАрд░ рдХреЗ рд╡реНрдпрдХреНрддрд┐рдЧрдд рдЧреБрдг рдФрд░ рдкреНрд░рддрд┐рдХреНрд░рд┐рдпрд╛рдПрдВред
рдЬреИрд╕рд╛ рдХрд┐ рдЖрдк рджреЗрдЦ рд╕рдХрддреЗ рд╣реИрдВ, рдЯреАрдХрд╛рдХрд░рдг рдХреЗ рдмрд╛рдж рдХреА рдЬрдЯрд┐рд▓рддрд╛рдУрдВ рдХреЗ рдХрд╛рд░рдг рдореБрдЦреНрдп рдХрд╛рд░рдХ рд╡рд┐рднрд┐рдиреНрди рд╕реБрд░рдХреНрд╖рд╛ рдЙрд▓реНрд▓рдВрдШрди рд╣реИрдВ, рджрд╡рд╛рдУрдВ рдХреЗ рдкреНрд░рд╢рд╛рд╕рди рдХреЗ рдирд┐рдпрдореЛрдВ рдХреА рдЙрдкреЗрдХреНрд╖рд╛, рдорддрднреЗрджреЛрдВ рдХреА рдЕрдирджреЗрдЦреА рдпрд╛ рдЙрдирдХреА рдЕрдкрд░реНрдпрд╛рдкреНрдд рд╕рдХреНрд░рд┐рдп рдкрд╣рдЪрд╛рди, рд╕рд╛рде рд╣реА рд╕рд╛рде рдЯреАрдХреЛрдВ рдХреА рдЕрд╕рдВрддреЛрд╖рдЬрдирдХ рдЧреБрдгрд╡рддреНрддрд╛ред рдХрд┐рд╕реА рд╡реНрдпрдХреНрддрд┐ рдХреЗ рд╡реНрдпрдХреНрддрд┐рдЧрдд рдЧреБрдгреЛрдВ рдХреЛ рдХреЗрд╡рд▓ рд╕реВрдЪреАрдмрджреНрдз рдХрд╛рд░рдХреЛрдВ рдкрд░ рдЖрд░реЛрдкрд┐рдд рдХрд┐рдпрд╛ рдЬрд╛ рд╕рдХрддрд╛ рд╣реИ, рдЬреЛ рдЬрдЯрд┐рд▓рддрд╛рдУрдВ рдХреЗ рд╡рд┐рдХрд╛рд╕ рдореЗрдВ рдпреЛрдЧрджрд╛рди рдХрд░рддреЗ рд╣реИрдВред

рдпрд╣реА рдХрд╛рд░рдг рд╣реИ рдХрд┐ рдЯреАрдХрд╛рдХрд░рдг рдХреА рдЬрдЯрд┐рд▓рддрд╛рдУрдВ рдХреА рд░реЛрдХрдерд╛рдо рдХрд╛ рдЖрдзрд╛рд░ рдорддрднреЗрджреЛрдВ рдХреА рдЧрд╣рди рдкрд╣рдЪрд╛рди, рдЯреАрдХреЛрдВ рдХреЗ рдЙрдкрдпреЛрдЧ рдХреА рддрдХрдиреАрдХ рдХрд╛ рдкрд╛рд▓рди, рджрд╡рд╛рдУрдВ рдХреА рдЧреБрдгрд╡рддреНрддрд╛ рдирд┐рдпрдВрддреНрд░рдг, рдЙрдирдХреЗ рднрдВрдбрд╛рд░рдг, рдкрд░рд┐рд╡рд╣рди рдФрд░ рдкрд░рд┐рд╡рд╣рди рдХреЗ рдирд┐рдпрдореЛрдВ рдХрд╛ рдкрд╛рд▓рди рд╣реИред рдЦрд░рд╛рдм рдЧреБрдгрд╡рддреНрддрд╛рд╣рд╛рд▓рд╛рдВрдХрд┐, рдЯреАрдХреЗ рд╢реБрд░реВ рдореЗрдВ рдЬрд░реВрд░реА рдирд╣реАрдВ рдХрд┐ рдЙрдирдореЗрдВ рдирд┐рд╣рд┐рдд рд╣реЛрдВред рдлрд╛рд░реНрдорд╛рд╕реНрдпреБрдЯрд┐рдХрд▓ рдкреНрд▓рд╛рдВрдЯ рд╕рд╛рдорд╛рдиреНрдп, рдЧреБрдгрд╡рддреНрддрд╛ рд╡рд╛рд▓реА рджрд╡рд╛рдУрдВ рдХрд╛ рдирд┐рд░реНрдорд╛рдг рдХрд░ рд╕рдХрддрд╛ рд╣реИред рд▓реЗрдХрд┐рди рдЙрдиреНрд╣реЗрдВ рд▓реЗ рдЬрд╛рдпрд╛ рдЧрдпрд╛, рдФрд░ рдлрд┐рд░ рдЧрд▓рдд рддрд░реАрдХреЗ рд╕реЗ рд╕рдВрдЧреНрд░рд╣реАрдд рдХрд┐рдпрд╛ рдЧрдпрд╛, рдЬрд┐рд╕рдХреЗ рдкрд░рд┐рдгрд╛рдорд╕реНрд╡рд░реВрдк рдЙрдиреНрд╣реЛрдВрдиреЗ рдирдХрд╛рд░рд╛рддреНрдордХ рдЧреБрдг рдкреНрд░рд╛рдкреНрдд рдХрд░ рд▓рд┐рдПред

рдбреАрдкреАрдЯреА, рдПрдбреАрдПрд╕-рдПрдо . рдХреЗ рд╕рд╛рде рдЯреАрдХрд╛рдХрд░рдг рдХреЗ рдмрд╛рдж рдЬрдЯрд┐рд▓рддрд╛рдПрдВ

рдХрд╛рд▓реА рдЦрд╛рдВрд╕реА, рдбрд┐рдкреНрдереАрд░рд┐рдпрд╛ рдФрд░ рдЯрд┐рдЯрдиреЗрд╕ рдХреЗ рдкреНрд░рддрд┐ рдкреНрд░рддрд┐рд░реЛрдзрдХ рдХреНрд╖рдорддрд╛ рд╡рд┐рдХрд╕рд┐рдд рдХрд░рдиреЗ рдХреЗ рд▓рд┐рдП рдбреАрдЯреАрдкреА рдЯреАрдХрд╛рдХрд░рдг рдХрд┐рдпрд╛ рдЬрд╛рддрд╛ рд╣реИред рд╡рд╣реАрдВ, K рдХрд╛рд▓реА рдЦрд╛рдВрд╕реА рдХреЗ рдЦрд┐рд▓рд╛рдл рдПрдХ рдШрдЯрдХ рд╣реИ, рд░рдХреНрддрдЪрд╛рдк рдбрд┐рдкреНрдереАрд░рд┐рдпрд╛ рдХреЗ рдЦрд┐рд▓рд╛рдл рд╣реИ, рдФрд░ AU рдЯрд┐рдЯрдиреЗрд╕ рдХреЗ рдЦрд┐рд▓рд╛рдл рд╣реИред рдЗрд╕реА рддрд░рд╣ рдХреЗ рдЯреАрдХреЗ рднреА рд╣реИрдВ: рдЯреЗрдЯреНрд░рд╛рдХреЛрдХ рдФрд░ рдЗрдиреНрдлреИрдирд░рд┐рдХреНрд╕ред рдЯреАрдХрд╛ рдмрдЪреНрдЪреЛрдВ рдХреЛ рджрд┐рдпрд╛ рдЬрд╛рддрд╛ рд╣реИ, рддреАрди рдЦреБрд░рд╛рдХреЗрдВ рджреА рдЬрд╛рддреА рд╣реИрдВ, рдФрд░ рддреАрд╕рд░реА рдХреЗ рдПрдХ рд╕рд╛рд▓ рдмрд╛рдж рдЪреМрдереАред рдлрд┐рд░ рдмрдЪреНрдЪреЛрдВ рдХреЛ рдХреЗрд╡рд▓ 6-7 рд╕рд╛рд▓ рдХреА рдЙрдореНрд░ рдореЗрдВ рдбрд┐рдкреНрдереАрд░рд┐рдпрд╛ рдФрд░ рдЯреЗрдЯрдирд╕ рдХреЗ рдЦрд┐рд▓рд╛рдл рдЯреАрдХрд╛ рд▓рдЧрд╛рдпрд╛ рдЬрд╛рддрд╛ рд╣реИ, рдФрд░ 14 рд╕рд╛рд▓ рдХреА рдЙрдореНрд░ рдореЗрдВ - рдПрдбреАрдПрд╕-рдПрдо рдЯреАрдХрд╛ рдХреЗ рд╕рд╛рдеред

рдбреАрдкреАрдЯреА рдХрд╛ рдЯреАрдХрд╛ 15,000 - 50,000 рдЯреАрдХрд╛рдХрд░рдг рдореЗрдВ рд╕реЗ 1 рдмрдЪреНрдЪреЗ рдореЗрдВ рд╡рд┐рднрд┐рдиреНрди рдЬрдЯрд┐рд▓рддрд╛рдУрдВ рдХреЗ рдЧрдарди рдХреЛ рднрдбрд╝рдХрд╛рддрд╛ рд╣реИред рдФрд░ рдЗрдиреНрдлреИрдирд░рд┐рдХреНрд╕ рд╡реИрдХреНрд╕реАрди рдореЗрдВ рдЬрдЯрд┐рд▓рддрд╛рдУрдВ рдХрд╛ рдЬреЛрдЦрд┐рдо рдХрд╛рдлреА рдХрдо рд╣реИ - рдкреНрд░рддрд┐ 100,000 рдореЗрдВ рдХреЗрд╡рд▓ 1 рдмрдЪреНрдЪреЗ рдореЗрдВ - 2,500,000 рдЯреАрдХрд╛рдХрд░рдгред рдПрдбреАрдПрд╕-рдПрдо рд╡реИрдХреНрд╕реАрди рд▓рдЧрднрдЧ рдХрднреА рднреА рдЬрдЯрд┐рд▓рддрд╛рдУрдВ рдХреА рдУрд░ рдирд╣реАрдВ рд▓реЗ рдЬрд╛рддрд╛ рд╣реИ, рдХреНрдпреЛрдВрдХрд┐ рдЗрд╕рдореЗрдВ рд╕рдмрд╕реЗ рдЕрдзрд┐рдХ рдкреНрд░рддрд┐рдХреНрд░рд┐рдпрд╛рд╢реАрд▓ рдкрд░реНрдЯреБрд╕рд┐рд╕ рдШрдЯрдХ рдХреА рдХрдореА рд╣реЛрддреА рд╣реИред

рдбреАрдкреАрдЯреА рд╡реИрдХреНрд╕реАрди рд╕реЗ рд╣реЛрдиреЗ рд╡рд╛рд▓реА рд╕рднреА рдЬрдЯрд┐рд▓рддрд╛рдУрдВ рдХреЛ рдЖрдорддреМрд░ рдкрд░ рд╕реНрдерд╛рдиреАрдп рдФрд░ рдкреНрд░рдгрд╛рд▓реАрдЧрдд рдореЗрдВ рд╡рд┐рднрд╛рдЬрд┐рдд рдХрд┐рдпрд╛ рдЬрд╛рддрд╛ рд╣реИред рддрд╛рд▓рд┐рдХрд╛ рдбреАрдкреАрдЯреА рдФрд░ рдПрдбреАрдПрд╕-рдПрдо рдХреА рд╕рднреА рд╕рдВрднрд╛рд╡рд┐рдд рдЬрдЯрд┐рд▓рддрд╛рдУрдВ рдФрд░ рдЯреАрдХрд╛рдХрд░рдг рдХреЗ рдмрд╛рдж рдЙрдирдХреЗ рд╡рд┐рдХрд╛рд╕ рдХреЗ рд╕рдордп рдХреЛ рджрд░реНрд╢рд╛рддреА рд╣реИ:

рдбреАрдкреАрдЯреА, рдПрдбреАрдПрд╕-рдПрдо . рдХреА рдЬрдЯрд┐рд▓рддрд╛рдУрдВ рдХреЗ рдкреНрд░рдХрд╛рд░ рдЬрдЯрд┐рд▓рддрд╛ рдкреНрд░рдХрд╛рд░ рдЬрдЯрд┐рд▓рддрд╛ рдкреНрд░рдХрд╛рд░
рдЗрдВрдЬреЗрдХреНрд╢рди рд╕реНрдерд▓ рдкрд░ рдорд╣рддреНрд╡рдкреВрд░реНрдг рд╡реГрджреНрдзрд┐ рдФрд░ рдЕрд╡рдзрд┐рд╕реНрдерд╛рдиреАрдп24 - 48 рдШрдВрдЯреЗ
рдЗрдВрдЬреЗрдХреНрд╢рди рд╕рд╛рдЗрдЯ рдХреА рд╕реВрдЬрди 8 рд╕реЗрдореА рд╕реЗ рдЕрдзрд┐рдХ рд╡реНрдпрд╛рд╕рд╕реНрдерд╛рдиреАрдп24 - 48 рдШрдВрдЯреЗ
рдПрд▓рд░реНрдЬреАрд╕реНрдерд╛рдиреАрдп24 - 48 рдШрдВрдЯреЗ
рддреНрд╡рдЪрд╛ рдХреА рд▓рд╛рд▓реАрд╕реНрдерд╛рдиреАрдп24 - 48 рдШрдВрдЯреЗ
3 рдпрд╛ рдЕрдзрд┐рдХ рдШрдВрдЯреЗ рддрдХ рд▓рдЧрд╛рддрд╛рд░ рд░реЛрдирд╛рдкреНрд░рдгрд╛рд▓реАрдЧрддрджреЛ рджрд┐рди рддрдХ
39.0 o . рд╕реЗ рдКрдкрд░ рд╢рд░реАрд░ рдХреЗ рддрд╛рдкрдорд╛рди рдореЗрдВ рд╡реГрджреНрдзрд┐рдкреНрд░рдгрд╛рд▓реАрдЧрдд72 рдШрдВрдЯреЗ рддрдХ
рдЬреНрд╡рд░ рд╕рдВрдмрдВрдзреА рдЖрдХреНрд╖реЗрдк (38.0 o C рдФрд░ рдЙрд╕рд╕реЗ рдЕрдзрд┐рдХ рдХреЗ рддрд╛рдкрдорд╛рди рдкрд░)рдкреНрд░рдгрд╛рд▓реАрдЧрдд24 - 72 рдШрдВрдЯреЗ
рдПрдлрдмреНрд░рд╛рдЗрд▓ рджреМрд░реЗ (рд╕рд╛рдорд╛рдиреНрдп рддрд╛рдкрдорд╛рди рдкрд░)рдкреНрд░рдгрд╛рд▓реАрдЧрддрдЯреАрдХрд╛рдХрд░рдг рдХреЗ 1 рд╕рд╛рд▓ рдмрд╛рдж
рддреАрд╡реНрд░рдЧрд╛рд╣рд┐рддрд╛ рд╕рдВрдмрдВрдзреА рд╕рджрдорд╛рдкреНрд░рдгрд╛рд▓реАрдЧрдд24 рдШрдВрдЯреЗ рддрдХ
рд▓рд┐рдореНрдлреИрдбреЗрдиреЛрдкреИрдереАрдкреНрд░рдгрд╛рд▓реАрдЧрдд7 рджрд┐рдиреЛрдВ рддрдХ
рд╕рд┐рд░рджрд░реНрджрдкреНрд░рдгрд╛рд▓реАрдЧрдд48 рдШрдВрдЯреЗ рддрдХ
рдЪрд┐рдбрд╝рдЪрд┐рдбрд╝рд╛рдкрдирдкреНрд░рдгрд╛рд▓реАрдЧрдд48 рдШрдВрдЯреЗ рддрдХ
рдЦрдЯреНрдЯреА рдбрдХрд╛рд░рдкреНрд░рдгрд╛рд▓реАрдЧрдд72 рдШрдВрдЯреЗ рддрдХ
рдЧрдВрднреАрд░ рдПрд▓рд░реНрдЬреА рдкреНрд░рддрд┐рдХреНрд░рд┐рдпрд╛рдПрдВ (рдХреНрд╡рд┐рдиреНрдХреЗ рдХреА рдПрдбрд┐рдорд╛, рдкрд┐рддреНрддреА, рдЖрджрд┐)рдкреНрд░рдгрд╛рд▓реАрдЧрдд72 рдШрдВрдЯреЗ рддрдХ
рджрдмрд╛рд╡ рдореЗрдВ рдХрдореА, рдорд╛рдВрд╕рдкреЗрд╢рд┐рдпреЛрдВ рдХреА рдЯреЛрдирдкреНрд░рдгрд╛рд▓реАрдЧрдд72 рдШрдВрдЯреЗ рддрдХ
рдмреЗрд╣реЛрд╢реАрдкреНрд░рдгрд╛рд▓реАрдЧрдд72 рдШрдВрдЯреЗ рддрдХ
рдореЗрдирд┐рдирдЬрд╛рдЗрдЯрд┐рд╕ рдпрд╛ рдПрдиреНрд╕реЗрдлрд▓рд╛рдЗрдЯрд┐рд╕рдкреНрд░рдгрд╛рд▓реАрдЧрдд1 рдорд╣реАрдиреЗ рддрдХ
рд╕рдВрд╡реЗрджреА рдХреНрд╖рддрд┐рдкреНрд░рдгрд╛рд▓реАрдЧрдд1 рдорд╣реАрдиреЗ рддрдХ
рдкреЙрд▓реАрд░реЗрдбрд┐рдХреБрд▓реЛрдиреНрдпреВрд░рд┐рдЯрд┐рд╕рдкреНрд░рдгрд╛рд▓реАрдЧрдд1 рдорд╣реАрдиреЗ рддрдХ
рдкреНрд▓реЗрдЯрд▓реЗрдЯ рдХрд╛рдЙрдВрдЯ рдореЗрдВ рдХрдореАрдкреНрд░рдгрд╛рд▓реАрдЧрдд1 рдорд╣реАрдиреЗ рддрдХ

рдбреАрдкреАрдЯреА рдФрд░ рдПрдбреАрдПрд╕-рдПрдо рдЯреАрдХрд╛рдХрд░рдг рдХреА рд╕реНрдерд╛рдиреАрдп рдЬрдЯрд┐рд▓рддрд╛рдПрдВ рдЖрдорддреМрд░ рдкрд░ рдХреБрдЫ рджрд┐рдиреЛрдВ рдХреЗ рднреАрддрд░ рдЕрдкрдиреЗ рдЖрдк рджреВрд░ рд╣реЛ рдЬрд╛рддреА рд╣реИрдВред рдмрдЪреНрдЪреЗ рдХреА рд╕реНрдерд┐рддрд┐ рдХреЛ рдХрдо рдХрд░рдиреЗ рдХреЗ рд▓рд┐рдП, рдЖрдк рдЗрдВрдЬреЗрдХреНрд╢рди рд╕рд╛рдЗрдЯ рдХреЛ Troxevasin рдорд░рд╣рдо рдХреЗ рд╕рд╛рде рдЪрд┐рдХрдирд╛рдИ рдХрд░ рд╕рдХрддреЗ рд╣реИрдВред рдпрджрд┐ рдмрдЪреНрдЪреЗ рдХреЛ рдбреАрдкреАрдЯреА рдЯреАрдХрд╛рдХрд░рдг рдХреЗ рдмрд╛рдж рдЬрдЯрд┐рд▓рддрд╛рдПрдВ рд╡рд┐рдХрд╕рд┐рдд рд╣реБрдИ рд╣реИрдВ, рддреЛ рдЕрдЧрд▓реА рдмрд╛рд░, рдХреЗрд╡рд▓ рдПрдВрдЯреА-рдбрд┐рдкреНрдереАрд░рд┐рдпрд╛ рдФрд░ рдПрдВрдЯреА-рдЯреЗрдЯрдирд╕ рдШрдЯрдХреЛрдВ рдХреЛ рдкрд░реНрдЯреБрд╕рд┐рд╕ рдХреЗ рдмрд┐рдирд╛ рдЗрдВрдЬреЗрдХреНрдЯ рдХрд┐рдпрд╛ рдЬрд╛рддрд╛ рд╣реИ, рдХреНрдпреЛрдВрдХрд┐ рдпрд╣ рд╡рд╣ рд╣реИ рдЬреЛ рдЕрдзрд┐рдХрд╛рдВрд╢ рдЬрдЯрд┐рд▓рддрд╛рдУрдВ рдХреЛ рднрдбрд╝рдХрд╛рддрд╛ рд╣реИред

рдЯрд┐рдЯрдиреЗрд╕ рдЯреАрдХрд╛рдХрд░рдг рдХреЗ рдмрд╛рдж рдЬрдЯрд┐рд▓рддрд╛рдПрдВ

рдЯреЗрдЯрдирд╕ рдЯреАрдХрд╛рдХрд░рдг рдирд┐рд░реНрджрд┐рд╖реНрдЯ рд╕рдордп рд╕реАрдорд╛ рдХреЗ рднреАрддрд░ рдирд┐рдореНрдирд▓рд┐рдЦрд┐рдд рдЬрдЯрд┐рд▓рддрд╛рдУрдВ рдХрд╛ рд╡рд┐рдХрд╛рд╕ рдХрд░ рд╕рдХрддрд╛ рд╣реИ:
  • 3 рджрд┐рдиреЛрдВ рдХреЗ рднреАрддрд░ рд╢рд░реАрд░ рдХреЗ рддрд╛рдкрдорд╛рди рдореЗрдВ рд╡реГрджреНрдзрд┐;
  • рдЗрдВрдЬреЗрдХреНрд╢рди рд╕реНрдерд▓ рдкрд░ рд▓рд╛рд▓рд┐рдорд╛ - 2 рджрд┐рдиреЛрдВ рддрдХ;
  • рд▓рд┐рдореНрдл рдиреЛрдбреНрд╕ рдХрд╛ рдЗрдЬрд╝рд╛рдлрд╝рд╛ рдФрд░ рд╡реНрдпрдерд╛ - рдПрдХ рд╕рдкреНрддрд╛рд╣ рддрдХ;
  • рдиреАрдВрдж рдореЗрдВ рдЦрд▓рд▓ - 2 рджрд┐рди рддрдХ;
  • - 2 рджрд┐рдиреЛрдВ рддрдХ;
  • рдкрд╛рдЪрди рдФрд░ рднреВрдЦ рд╡рд┐рдХрд╛рд░ - 3 рджрд┐рди рддрдХ;
  • рдПрд▓рд░реНрдЬреА рджрд╛рдиреЗ;
  • рд▓рдВрдмреЗ рд╕рдордп рддрдХ рд▓рдЧрд╛рддрд╛рд░ рд░реЛрдирд╛ - 3 рджрд┐рдиреЛрдВ рддрдХ;
  • рдКрдВрдЪреЗ рддрд╛рдкрдорд╛рди рдХреА рдкреГрд╖реНрдарднреВрдорд┐ рдХреЗ рдЦрд┐рд▓рд╛рдл рдЖрдХреНрд╖реЗрдк - 3 рджрд┐рдиреЛрдВ рддрдХ;
  • рдореЗрдирд┐рдирдЬрд╛рдЗрдЯрд┐рд╕ рдпрд╛ рдПрдиреНрд╕реЗрдлрд▓рд╛рдЗрдЯрд┐рд╕ - 1 рдорд╣реАрдиреЗ рддрдХ;
  • рд╢реНрд░рд╡рдг рдФрд░ рдСрдкреНрдЯрд┐рдХ рддрдВрддреНрд░рд┐рдХрд╛ - 1 рдорд╣реАрдиреЗ рддрдХред
рдЬрдЯрд┐рд▓рддрд╛рдУрдВ рдХреЗ рдЬреЛрдЦрд┐рдо рдХреЛ рдиреНрдпреВрдирддрдо рд╕рдВрднрд╡ рд╕реНрддрд░ рддрдХ рдХрдо рдХрд░рдиреЗ рдХреЗ рд▓рд┐рдП, рдЯреАрдХрд╛рдХрд░рдг рдХреЗ рдирд┐рдпрдореЛрдВ рдХрд╛ рдкрд╛рд▓рди рдХрд░рдирд╛, рдорддрднреЗрджреЛрдВ рдХреЛ рдзреНрдпрд╛рди рдореЗрдВ рд░рдЦрдирд╛ рдФрд░ рд╕реНрдерд╛рдкрд┐рдд рдорд╛рдирдХреЛрдВ рдХреЗ рдЙрд▓реНрд▓рдВрдШрди рдореЗрдВ рд╕рдВрдЧреНрд░рд╣реАрдд рджрд╡рд╛рдУрдВ рдХрд╛ рдЙрдкрдпреЛрдЧ рдирд╣реАрдВ рдХрд░рдирд╛ рдЖрд╡рд╢реНрдпрдХ рд╣реИред

рдбрд┐рдкреНрдереАрд░рд┐рдпрд╛ рдЯреАрдХрд╛рдХрд░рдг рдХреЗ рдмрд╛рдж рдЬрдЯрд┐рд▓рддрд╛рдПрдВ

рдЕрдХреЗрд▓реЗ рдбрд┐рдкреНрдереАрд░рд┐рдпрд╛ рдХреЗ рдЦрд┐рд▓рд╛рдл рдЯреАрдХрд╛рдХрд░рдг рдмрд╣реБрдд рдкреНрд░рддрд┐рдХреНрд░рд┐рдпрд╛рд╢реАрд▓ рдирд╣реАрдВ рд╣реИ, рдЗрд╕рд▓рд┐рдП рдЗрд╕реЗ рдЕрдкреЗрдХреНрд╖рд╛рдХреГрдд рдЖрд╕рд╛рдиреА рд╕реЗ рд╕рд╣рди рдХрд┐рдпрд╛ рдЬрд╛рддрд╛ рд╣реИред рдЬрдЯрд┐рд▓рддрд╛рдПрдВ рдПрдирд╛рдлрд┐рд▓реЗрдХреНрдЯрд┐рдХ рд╕рджрдореЗ рдХреЗ рд░реВрдк рдореЗрдВ, рдЗрдВрдЬреЗрдХреНрд╢рди рд╕реНрдерд▓ рдкрд░, рдЗрдВрдЬреЗрдХреНрд╢рди рд╕реНрдерд▓ рдХреА рд╡реНрдпрдерд╛ рдФрд░ рд╕рдВрдкреВрд░реНрдг рдЕрдВрдЧ, рддрдВрддреНрд░рд┐рдХрд╛ рд╕рдВрдмрдВрдзреА рд╡рд┐рдХрд╛рд░реЛрдВ рдХреЗ рд░реВрдк рдореЗрдВ рд╡рд┐рдХрд╕рд┐рдд рд╣реЛ рд╕рдХрддреА рд╣реИрдВред

рдЯреАрдХрд╛рдХрд░рдг рдХреЗ рдмрд╛рдж рдЬрдЯрд┐рд▓рддрд╛рдПрдВ рдкреЗрдВрдЯрд╛рдХреНрд╕рд┐рдо

рдкреЗрдВрдЯрд╛рдХреНрд╕рд┐рдо рд╡реИрдХреНрд╕реАрди рд╕рдВрдпреБрдХреНрдд рд╣реИ, рдЗрд╕реЗ рдкрд╛рдВрдЪ рдмреАрдорд╛рд░рд┐рдпреЛрдВ рдХреЗ рдЦрд┐рд▓рд╛рдл рдкреНрд░рд╢рд╛рд╕рд┐рдд рдХрд┐рдпрд╛ рдЬрд╛рддрд╛ рд╣реИ - рдбрд┐рдкреНрдереАрд░рд┐рдпрд╛, рдкрд░реНрдЯреБрд╕рд┐рд╕, рдЯреЗрдЯрдирд╕, рдкреЛрд▓рд┐рдпреЛрдорд╛рдЗрд▓рд╛рдЗрдЯрд┐рд╕ рдФрд░ рдПрдЪрдЖрдИрдмреА рд╕рдВрдХреНрд░рдордг, рдЬреЛ рд╣реАрдореЛрдлрд┐рд▓рд╕ рдЗрдиреНрдлреНрд▓реБрдПрдВрдЬрд╛ рдХреЗ рдХрд╛рд░рдг рд╣реЛрддрд╛ рд╣реИред рдкреЗрдВрдЯрд╛рдХреНрд╕рд┐рдо рд╡реИрдХреНрд╕реАрди рдХреА рд╕рднреА 4 рдЦреБрд░рд╛рдХ рдкреНрд░рд╛рдкреНрдд рдХрд░рдиреЗ рд╡рд╛рд▓реЗ рдмрдЪреНрдЪреЛрдВ рдХреА рдЯрд┐рдкреНрдкрдгрд┐рдпреЛрдВ рдХреЗ рдЕрдиреБрд╕рд╛рд░, рдЬрдЯрд┐рд▓рддрд╛рдПрдВ рдХреЗрд╡рд▓ 0.6% рдореЗрдВ рд╡рд┐рдХрд╕рд┐рдд рд╣реБрдИрдВред рдЗрди рдЬрдЯрд┐рд▓рддрд╛рдУрдВ рдХреЗ рд▓рд┐рдП рдпреЛрдЧреНрдпрддрд╛ рдХреА рдЖрд╡рд╢реНрдпрдХрддрд╛ рд╣реИ рдЪрд┐рдХрд┐рддреНрд╕рд╛ рджреЗрдЦрднрд╛рд▓, рд▓реЗрдХрд┐рди рдПрдХ рднреА рдореМрдд рджрд░реНрдЬ рдирд╣реАрдВ рдХреА рдЧрдИ рдереАред рдЪреВрдВрдХрд┐ рдкреЗрдВрдЯрд╛рдХреНрд╕рд┐рдо рдореЗрдВ рдкреЛрд▓рд┐рдпреЛ рд░реЛрдзреА рдШрдЯрдХ рд╣реЛрддрд╛ рд╣реИ, рдЗрд╕рд▓рд┐рдП рдЗрд╕ рдмреАрдорд╛рд░реА рдХрд╛ рдХреЛрдИ рдЦрддрд░рд╛ рдирд╣реАрдВ рд╣реИ, рд▓реЗрдХрд┐рди рдпрд╣ рдореМрдЦрд┐рдХ рдЯреАрдХреЗ рдХрд╛ рдЙрдкрдпреЛрдЧ рдХрд░рддреЗ рд╕рдордп рд╣реЛрддрд╛ рд╣реИред

рдкреЗрдВрдЯрд╛рдХреНрд╕рд┐рдо, рдкрд╛рдВрдЪ рдШрдЯрдХреЛрдВ рдХреЗ рдмрд╛рд╡рдЬреВрдж, рд╢рд╛рдпрдж рд╣реА рдХрднреА рдкреНрд░рддрд┐рдХреНрд░рд┐рдпрд╛рдУрдВ рдФрд░ рдЬрдЯрд┐рд▓рддрд╛рдУрдВ рдХрд╛ рдХрд╛рд░рдг рдмрдирддрд╛ рд╣реИ, рдЬреЛ рдореБрдЦреНрдп рд░реВрдк рд╕реЗ рдЗрдВрдЬреЗрдХреНрд╢рди рд╕реНрдерд▓ рдкрд░ рддреЗрдЬ рдмреБрдЦрд╛рд░, рдЪрд┐рдбрд╝рдЪрд┐рдбрд╝рд╛рдкрди, рд▓рдВрдмреЗ рд╕рдордп рддрдХ рд░реЛрдирд╛, рджрдмрд╛рд╡ рдФрд░ рдзрдХреНрдХреЛрдВ рдХреЗ рд░реВрдк рдореЗрдВ рдкреНрд░рдХрдЯ рд╣реЛрддрд╛ рд╣реИред рджреБрд░реНрд▓рдн рдорд╛рдорд▓реЛрдВ рдореЗрдВ, рдЖрдХреНрд╖реЗрдк, рд╣рд▓реНрдХреЗ рддрдВрддреНрд░рд┐рдХрд╛ рд╕рдВрдмрдВрдзреА рд▓рдХреНрд╖рдг, рдкрд╛рдЪрди рд╡рд┐рдХрд╛рд░, рдЗрдВрдЬреЗрдХреНрд╢рди рд╕реНрдерд▓ рдкрд░ рддреЗрдЬ рджрд░реНрдж рдФрд░ рдкреВрд░реЗ рдЕрдВрдЧ рд╡рд┐рдХрд╕рд┐рдд рд╣реЛ рд╕рдХрддреЗ рд╣реИрдВред рд╕рдмрд╕реЗ рдЧрдВрднреАрд░ рдкреНрд░рддрд┐рдХреНрд░рд┐рдпрд╛ рдЖрдорддреМрд░ рдкрд░ рджреВрд╕рд░реА рдЦреБрд░рд╛рдХ рдХреЗ рд╕рд╛рде рд╡рд┐рдХрд╕рд┐рдд рд╣реЛрддреА рд╣реИ, рдЬрдмрдХрд┐ рдкрд╣рд▓реА рдФрд░ рддреАрд╕рд░реА рдЖрд╕рд╛рди рд╣реЛрддреА рд╣реИред

рд╣реЗрдкреЗрдЯрд╛рдЗрдЯрд┐рд╕ рдмреА рдЯреАрдХрд╛рдХрд░рдг рдХреЗ рдмрд╛рдж рдЬрдЯрд┐рд▓рддрд╛рдПрдВ

рдмреА рдХреЗ рдЦрд┐рд▓рд╛рдл рдЯреАрдХрд╛рдХрд░рдг рдирд┐рдореНрдирд▓рд┐рдЦрд┐рдд рдЬрдЯрд┐рд▓рддрд╛рдУрдВ рдХрд╛ рдХрд╛рд░рдг рдмрди рд╕рдХрддрд╛ рд╣реИ рдЬреЛ рдирд┐рд░реНрджрд┐рд╖реНрдЯ рд╕рдордп рд╕реАрдорд╛ рдХреЗ рднреАрддрд░ рд╡рд┐рдХрд╕рд┐рдд рд╣реЛрддреЗ рд╣реИрдВ:
  • рд╢рд░реАрд░ рдХреЗ рддрд╛рдкрдорд╛рди рдореЗрдВ рд╡реГрджреНрдзрд┐ - 3 рджрд┐рдиреЛрдВ рддрдХред
  • рдЗрдВрдЬреЗрдХреНрд╢рди рд╕реНрдерд▓ рдкрд░ рдПрдХ рд╕реНрдкрд╖реНрдЯ рдкреНрд░рддрд┐рдХреНрд░рд┐рдпрд╛ (рджрд░реНрдж, 5 рд╕реЗрдореА рд╕реЗ рдЕрдзрд┐рдХ рд╕реВрдЬрди, 8 рд╕реЗрдореА рд╕реЗ рдЕрдзрд┐рдХ рд▓рд╛рд▓реА, 2 рд╕реЗрдореА рд╕реЗ рдЕрдзрд┐рдХ рдХреА рдЕрд╡рдзрд┐) - 2 рджрд┐рдиреЛрдВ рддрдХред

  • рдмреАрд╕реАрдЬреА рдЯреАрдХрд╛рдХрд░рдг рдХреЗ рдмрд╛рдж рдЬрдЯрд┐рд▓рддрд╛рдПрдВ

    рдпрд╣ рд╕рдордЭрд╛ рдЬрд╛рдирд╛ рдЪрд╛рд╣рд┐рдП рдХрд┐ рдмреАрд╕реАрдЬреА рдХрд╛ рдЙрджреНрджреЗрд╢реНрдп рд╢рд░реАрд░ рдХреЛ рддрдкреЗрджрд┐рдХ рд╕реЗ рдкреНрд░рддрд┐рд░рдХреНрд╖рд┐рдд рдХрд░рдирд╛ рдирд╣реАрдВ рд╣реИ, рдмрд▓реНрдХрд┐ рд╕рдВрдХреНрд░рдордг рдХреЗ рдорд╛рдорд▓реЗ рдореЗрдВ рд░реЛрдЧ рдХреА рдЧрдВрднреАрд░рддрд╛ рдХреЛ рдХрдо рдХрд░рдирд╛ рд╣реИред рдпрд╣ 1 рд╡рд░реНрд╖ рд╕реЗ рдХрдо рдЙрдореНрд░ рдХреЗ рдмрдЪреНрдЪреЛрдВ рдХреЗ рд▓рд┐рдП рд╡рд┐рд╢реЗрд╖ рд░реВрдк рд╕реЗ рд╕рдЪ рд╣реИ, рдЬрд┐рд╕рдореЗрдВ рддрдкреЗрджрд┐рдХ рд╕рдВрдХреНрд░рдордг рдлреЗрдлрдбрд╝реЛрдВ рдХреЛ рдкреНрд░рднрд╛рд╡рд┐рдд рдирд╣реАрдВ рдХрд░рддрд╛ рд╣реИ, рд▓реЗрдХрд┐рди рд╕рд╛рдорд╛рдиреНрдпреАрдХреГрдд рд░рдХреНрдд рд╡рд┐рд╖рд╛рдХреНрддрддрд╛ рдпрд╛ рдореЗрдирд┐рдирдЬрд╛рдЗрдЯрд┐рд╕ рджреЗрддрд╛ рд╣реИред рд╣рд╛рд▓рд╛рдВрдХрд┐, рдмреАрд╕реАрдЬреА рдЕрдкрдиреЗ рдЖрдк рдореЗрдВ рдПрдХ рдХрдо-рдкреНрд░рддрд┐рдХреНрд░рд┐рдпрд╛рд╢реАрд▓ рдЯреАрдХрд╛ рд╣реИ рдЬреЛ 2 рджрд┐рдиреЛрдВ рдХреЗ рднреАрддрд░ рддрд╛рдкрдорд╛рди рдореЗрдВ рд╡реГрджреНрдзрд┐, рдЗрдВрдЬреЗрдХреНрд╢рди рд╕реНрдерд▓ рдкрд░ рддреНрд╡рдЪрд╛ рдХреЗ рдиреАрдЪреЗ рдПрдХ рдлреЛрдбрд╝рд╛ рдпрд╛ 1.5-6 рдорд╣реАрдиреЛрдВ рдХреЗ рдмрд╛рдж 1 рд╕реЗрдореА рд╕реЗ рдЕрдзрд┐рдХ рдФрд░ 6-12 рдорд╣реАрдиреЛрдВ рдХреЗ рдмрд╛рдж рдХреЗрд▓реЛрдЗрдб рдирд┐рд╢рд╛рди рдкреИрджрд╛ рдХрд░ рд╕рдХрддрд╛ рд╣реИред . рдЗрд╕рдХреЗ рдЕрд▓рд╛рд╡рд╛, рдирд┐рдореНрдирд▓рд┐рдЦрд┐рдд рдХреЛ рдмреАрд╕реАрдЬреА рдХреА рдЬрдЯрд┐рд▓рддрд╛рдУрдВ рдХреЗ рд░реВрдк рдореЗрдВ рд╕реВрдЪрд┐рдд рдХрд┐рдпрд╛ рдЧрдпрд╛ рд╣реИ:
    • рд╕рд╛рдорд╛рдиреНрдпреАрдХреГрдд рдмреАрд╕реАрдЬреА рд╕рдВрдХреНрд░рдордг - 2-18 рдорд╣реАрдиреЛрдВ рдХреЗ рдмрд╛рдж;
    • рдСрд╕реНрдЯрд┐рдпреЛрдорд╛рдЗрд▓рд╛рдЗрдЯрд┐рд╕ - 2-18 рдорд╣реАрдиреЛрдВ рдХреЗ рдмрд╛рдж;
    • рдУрд╕реНрдЯрд┐рдЯрд┐рд╕ - 2-18 рдорд╣реАрдиреЛрдВ рдХреЗ рдмрд╛рдж;
    • рд▓рд╕реАрдХрд╛ рдирд▓рд┐рдХрд╛рдУрдВ рдХреА рд╕реВрдЬрди - 2 рд╕реЗ 6 рдорд╣реАрдиреЗ рдХреЗ рдмрд╛рджред

    рдлреНрд▓реВ рдХрд╛ рдЯреАрдХрд╛ рд▓рдЧрд╡рд╛рдиреЗ рдХреЗ рдмрд╛рдж рдЬрдЯрд┐рд▓рддрд╛рдПрдВ

    рдШрд░реЗрд▓реВ рдФрд░ рдЖрдпрд╛рддрд┐рдд рдЙрддреНрдкрд╛рджрди рдХреЗ рдЦрд┐рд▓рд╛рдл рдЯреАрдХреЗ рд░реВрд╕ рдореЗрдВ рдЙрдкрд▓рдмреНрдз рд╣реИрдВ, рдФрд░ рдЙрди рд╕рднреА рдореЗрдВ рд▓рдЧрднрдЧ рд╕рдорд╛рди рдЧреБрдг рд╣реИрдВ рдФрд░ рд╕рдорд╛рди рдЬрдЯрд┐рд▓рддрд╛рдПрдВ рдкреИрджрд╛ рдХрд░рдиреЗ рдореЗрдВ рд╕рдХреНрд╖рдо рд╣реИрдВред рд╕рд╛рдорд╛рдиреНрдп рддреМрд░ рдкрд░, рдЗрдиреНрдлреНрд▓реВрдПрдВрдЬрд╛ рдЯреАрдХрд╛рдХрд░рдг рд╢рд╛рдпрдж рд╣реА рдХрднреА рдЬрдЯрд┐рд▓рддрд╛рдУрдВ рдХреЗ рд╕рд╛рде рд╣реЛрддрд╛ рд╣реИ, рдЬрд┐рд╕рдХрд╛ рд╕реНрдкреЗрдХреНрдЯреНрд░рдо рдмрд╣реБрдд рд╕рдВрдХреАрд░реНрдг рд╣реЛрддрд╛ рд╣реИред рд╕рдмрд╕реЗ рдЖрдо рдЬрдЯрд┐рд▓рддрд╛рдПрдВ рдПрд▓рд░реНрдЬреА рдХреЗ рд░реВрдк рдореЗрдВ рд╣реЛрддреА рд╣реИрдВ, рдЦрд╛рд╕рдХрд░ рдЙрди рд▓реЛрдЧреЛрдВ рдореЗрдВ рдЬрд┐рдирдХреЗ рдкрд╛рд╕ рджрд╡рд╛ рдирд┐рдпреЛрдорд╛рдЗрд╕рд┐рди рдпрд╛ рдЪрд┐рдХрди рдЕрдВрдбреЗ рдХрд╛ рд╕рдлреЗрдж рднрд╛рдЧ рд╣реЛрддрд╛ рд╣реИред рд░рдХреНрддрд╕реНрд░рд╛рд╡реА рд╡рд╛рд╕реНрдХреБрд▓рд┐рдЯрд┐рд╕ рдХреЗ рдЧрдарди рдХреЗ рдХрдИ рдорд╛рдорд▓реЗ рджрд░реНрдЬ рдХрд┐рдП рдЧрдП рд╣реИрдВ, рд▓реЗрдХрд┐рди рдлреНрд▓реВ рдХреЗ рдЯреАрдХреЗ рдХреЗ рд╕рд╛рде рдЗрд╕ рд╡рд┐рдХреГрддрд┐ рдХрд╛ рд╕рдВрдмрдВрдз рдирд┐рд╢реНрдЪрд┐рдд рд░реВрдк рд╕реЗ рд╕реНрдерд╛рдкрд┐рдд рдирд╣реАрдВ рдХрд┐рдпрд╛ рдЧрдпрд╛ рд╣реИред

    рдЪрд┐рдХрдирдкреЙрдХреНрд╕, рдЦрд╕рд░рд╛, рд░реВрдмреЗрд▓рд╛, рд╕рдВрдпреБрдХреНрдд рдХреЗ рдЦрд┐рд▓рд╛рдл рдЯреАрдХрд╛рдХрд░рдг рдХреЗ рдмрд╛рдж рдЬрдЯрд┐рд▓рддрд╛рдПрдВ
    рдкреАрдбреАрдП рдФрд░ рдкреНрд░рд╛рдпрд░рд┐рдХреНрд╕ рдЯреАрдХреЗ

    рдкреНрд░рд╛рдпрд░рд┐рдХреНрд╕ рдПрдХ рд╕рдВрдпреЛрдЬрди рдЦрд╕рд░рд╛, рдХрдгреНрдардорд╛рд▓рд╛ рдФрд░ рд░реВрдмреЗрд▓рд╛ рд╡реИрдХреНрд╕реАрди рд╣реИред рдЗрди рд╕рдВрдХреНрд░рдордгреЛрдВ рдХреЗ рдЦрд┐рд▓рд╛рдл рдЯреАрдХрд╛рдХрд░рдг рд▓рдЧрднрдЧ рд╕рдорд╛рди рдкреНрд░рддрд┐рдХреНрд░рд┐рдпрд╛рдУрдВ рдФрд░ рдЬрдЯрд┐рд▓рддрд╛рдУрдВ рдХрд╛ рдХрд╛рд░рдг рдмрдирддрд╛ рд╣реИред рддреЛ, рдЯреАрдХрд╛рдХрд░рдг рдХреЗ 4-15 рджрд┐рдиреЛрдВ рдХреЗ рдмрд╛рдж рд╣реА рддрд╛рдкрдорд╛рди рдореЗрдВ рд╡реГрджреНрдзрд┐ рджреЗрдЦреА рдЬрд╛ рд╕рдХрддреА рд╣реИ, рдФрд░ рдкрд╣рд▓реЗ рджреЛ рджрд┐рдиреЛрдВ рдореЗрдВ рдПрдХ рдордЬрдмреВрдд рд╕реНрдерд╛рдиреАрдп рдкреНрд░рддрд┐рдХреНрд░рд┐рдпрд╛ рджреЗрдЦреА рдЬрд╛рддреА рд╣реИ, рдФрд░ 5 рд╕реЗрдореА рд╕реЗ рдЕрдзрд┐рдХ рдЧрдВрднреАрд░ рдПрдбрд┐рдорд╛ рдХреЗ рдЧрдарди рдореЗрдВ рд╡реНрдпрдХреНрдд рдХреА рдЬрд╛рддреА рд╣реИ, 8 рд╕реЗрдореА рд╕реЗ рдЕрдзрд┐рдХ рд▓рд╛рд▓реА, рдФрд░ рдЕрд╡рдзрд┐ - 2 рд╕реЗрдореА рд╕реЗ рдЕрдзрд┐рдХред рдЗрд╕рдХреЗ рдЕрд▓рд╛рд╡рд╛, рдЪрд┐рдХрдирдкреЙрдХреНрд╕, рдЦрд╕рд░рд╛, рд░реВрдмреЗрд▓рд╛ рдФрд░ рд╕рдВрдпреБрдХреНрдд рдПрдордПрдордЖрд░ рдХреЗ рдЦрд┐рд▓рд╛рдл рдЯреАрдХрд╛рдХрд░рдг рдЙрдЪрд┐рдд рд╕рдордп рдкрд░ рдирд┐рдореНрдирд▓рд┐рдЦрд┐рдд рдЬрдЯрд┐рд▓рддрд╛рдУрдВ рдХрд╛ рдХрд╛рд░рдг рдмрди рд╕рдХрддрд╛ рд╣реИ:
    - 4 рд╕реЗ 15 рджрд┐рдиреЛрдВ рддрдХред
рдпреЗ рдЬрдЯрд┐рд▓рддрд╛рдПрдВ рдмрд╣реБрдд рдХрдо рд╡рд┐рдХрд╕рд┐рдд рд╣реЛрддреА рд╣реИрдВ, рдФрд░ рджрд╡рд╛рдУрдВ рдХреЗ рдЯреАрдХрд╛рдХрд░рдг, рднрдВрдбрд╛рд░рдг рдФрд░ рдкрд░рд┐рд╡рд╣рди рдХреЗ рдирд┐рдпрдореЛрдВ рдХрд╛ рдкрд╛рд▓рди рдХрд░рдХреЗ рдЙрдиреНрд╣реЗрдВ рд░реЛрдХрд╛ рдЬрд╛ рд╕рдХрддрд╛ рд╣реИред

рд░реЗрдмреАрдЬ рдЯреАрдХрд╛рдХрд░рдг рдХреЗ рдмрд╛рдж рдЬрдЯрд┐рд▓рддрд╛рдПрдВ

рд░реЗрдмреАрдЬ рдЯреАрдХрд╛рдХрд░рдг рдмрд╣реБрдд рдХрдо рд╣реА рдЬрдЯрд┐рд▓рддрд╛рдУрдВ рдХреЗ рд╡рд┐рдХрд╛рд╕ рдХреЛ рднрдбрд╝рдХрд╛рддрд╛ рд╣реИ, рдФрд░ рд╡реЗ рдореБрдЦреНрдп рд░реВрдк рд╕реЗ рдПрд▓рд░реНрдЬреА рд╕реЗ рдкреНрд░рдХрдЯ рд╣реЛрддреЗ рд╣реИрдВ, рд╡рд┐рд╢реЗрд╖ рд░реВрдк рд╕реЗ рдЪрд┐рдХрди рдЕрдВрдбреЗ рдХреА рд╕рдлреЗрджреА рдХреА рдкреНрд░рддрд┐рдХреНрд░рд┐рдпрд╛ рд╕реЗ рдкреАрдбрд╝рд┐рдд рд▓реЛрдЧреЛрдВ рдореЗрдВред рддрдВрддреНрд░рд┐рдХрд╛ рд╕рдВрдмрдВрдзреА рд▓рдХреНрд╖рдг рднреА рдиреЛрдЯ рдХрд┐рдП рдЬрд╛рддреЗ рд╣реИрдВ, рдЬреИрд╕реЗ рдХрд┐ рдирд╕реЛрдВ рдХрд╛ рджрд░реНрдж, рдЪрдХреНрдХрд░ рдЖрдирд╛, рдиреНрдпреВрд░реЛрдкреИрдереА, рдЬреЛ, рд╣рд╛рд▓рд╛рдВрдХрд┐, рдереЛрдбрд╝реЗ рд╕рдордп рдХреЗ рдмрд╛рдж рдЕрдкрдиреЗ рдЖрдк рдФрд░ рдмрд┐рдирд╛ рдХрд┐рд╕реА рдирд┐рд╢рд╛рди рдХреЗ рдЪрд▓реЗ рдЬрд╛рддреЗ рд╣реИрдВред

рдордВрдЯреМрдХреНрд╕ рдкрд░реАрдХреНрд╖рдг рдХреЗ рдмрд╛рдж рдЬрдЯрд┐рд▓рддрд╛рдПрдВ

рдордВрдЯреМрдХреНрд╕ рдПрдХ рдЬреИрд╡рд┐рдХ рдкрд░реАрдХреНрд╖рдг рд╣реИ рдЬреЛ рдПрдХ рд░реЛрдЧрдЬрдирдХ - рдХреЛрдЪ рдХреЗ рдмреЗрд╕рд┐рд▓рд╕ рд╡рд╛рд▓реЗ рдмрдЪреНрдЪреЗ рдХреЗ рд╕рдВрдХреНрд░рдордг рдХрд╛ рдкрддрд╛ рд▓рдЧрд╛рдиреЗ рдХреЗ рд▓рд┐рдП рдЖрд╡рд╢реНрдпрдХ рд╣реИред рдордВрдЯреМрдХреНрд╕ рдкрд░реАрдХреНрд╖рдг рдХрд╛ рдЙрдкрдпреЛрдЧ рдлреНрд▓реЛрд░реЛрдЧреНрд░рд╛рдлреА рдХреЗ рдмрдЬрд╛рдп рдмрдЪреНрдЪреЛрдВ рдореЗрдВ рдХрд┐рдпрд╛ рдЬрд╛рддрд╛ рд╣реИ, рдЬреЛ рд╡рдпрд╕реНрдХреЛрдВ рдХреЗ рд▓рд┐рдП рдХрд┐рдпрд╛ рдЬрд╛рддрд╛ рд╣реИред рдЬрдЯрд┐рд▓рддрд╛рдУрдВ рдХреЗ рд░реВрдк рдореЗрдВ, рдордВрдЯреМрдХреНрд╕ рдкрд░реАрдХреНрд╖рдг рд▓рд┐рдореНрдл рдиреЛрдбреНрд╕ рдФрд░ рдирд▓рд┐рдХрд╛рдУрдВ рдХреА рд╕реВрдЬрди рдХреЗ рд╕рд╛рде-рд╕рд╛рде рдЕрд╕реНрд╡рд╕реНрдерддрд╛, рд╕рд┐рд░рджрд░реНрдж, рдХрдордЬреЛрд░реА рдпрд╛ рдмреБрдЦрд╛рд░ рдХреЗ рд╕рд╛рде рд╣реЛ рд╕рдХрддрд╛ рд╣реИред рдордВрдЯреМрдХреНрд╕ рдкрд░реАрдХреНрд╖рдг рдХреЗ рд▓рд┐рдП рдкреНрд░рддрд┐рдХреНрд░рд┐рдпрд╛рдУрдВ рдХреА рдЧрдВрднреАрд░рддрд╛ рдорд╛рдирд╡ рд╢рд░реАрд░ рдХреА рд╡реНрдпрдХреНрддрд┐рдЧрдд рд╡рд┐рд╢реЗрд╖рддрд╛рдУрдВ рдкрд░ рдирд┐рд░реНрднрд░ рдХрд░рддреА рд╣реИред рдЙрджрд╛рд╣рд░рдг рдХреЗ рд▓рд┐рдП, рдХреБрдЫ рдмрдЪреНрдЪреЛрдВ рдХреЛ рд╣рд╛рде рдореЗрдВ рддреЗрдЬ рджрд░реНрдж рдпрд╛ рдЙрд▓реНрдЯреА рд╣реЛрддреА рд╣реИред

рдЯреАрдХрд╛рдХрд░рдг рдЬрдЯрд┐рд▓рддрд╛рдУрдВ рдХреЗ рдЖрдБрдХрдбрд╝реЗ

рдЖрдЬ рддрдХ, рд░реВрд╕ рдореЗрдВ, рдЯреАрдХрд╛рдХрд░рдг рдХреЗ рдкрд░рд┐рдгрд╛рдорд╕реНрд╡рд░реВрдк рдЬрдЯрд┐рд▓рддрд╛рдУрдВ рдХреА рд╕рдВрдЦреНрдпрд╛ рдкрд░ рдЖрдзрд┐рдХрд╛рд░рд┐рдХ рдкрдВрдЬреАрдХрд░рдг рдФрд░ рдирд┐рдпрдВрддреНрд░рдг рдХреЗрд╡рд▓ 1998 рд╕реЗ рдХрд┐рдпрд╛ рдЧрдпрд╛ рд╣реИред рдЗрд╕ рддрд░рд╣ рдХреЗ рдХрд╛рдо рд░рд╛рд╖реНрдЯреНрд░реАрдп рд╡рд┐рд╢рд┐рд╖реНрдЯ рд╡реИрдЬреНрдЮрд╛рдирд┐рдХ рд╕рдВрд╕реНрдерд╛рдиреЛрдВ рдФрд░ рд╡рд┐рд╢реНрд╡ рд╕реНрд╡рд╛рд╕реНрдереНрдп рд╕рдВрдЧрдарди рдХреЗ рд╡рд┐рд╢реЗрд╖рдЬреНрдЮреЛрдВ рджреНрд╡рд╛рд░рд╛ рдХрд┐рдП рдЬрд╛рддреЗ рд╣реИрдВ, рд▓реЗрдХрд┐рди рд╡реЗ рд╕реАрдорд┐рдд рд╕рдВрдЦреНрдпрд╛ рдореЗрдВ рдмрд╕реНрддрд┐рдпреЛрдВ рдореЗрдВ рд╣реА рд╕реНрдерд┐рддрд┐ рдХрд╛ рдЕрдзреНрдпрдпрди рдХрд░рдиреЗ рдореЗрдВ рд╕рдХреНрд╖рдо рд╣реИрдВ, рдореБрдЦреНрдпрддрдГ рдмрдбрд╝реЗ рд╢рд╣рд░реЛрдВ рдореЗрдВред рдЕрдореЗрд░рд┐рдХреА рдЖрдВрдХрдбрд╝реЛрдВ рдХреЗ рдЕрдиреБрд╕рд╛рд░, рд╣рд░ рд╕рд╛рд▓ 50 рдмрдЪреНрдЪреЛрдВ рдореЗрдВ рдЧрдВрднреАрд░ рдиреНрдпреВрд░реЛрд▓реЙрдЬрд┐рдХрд▓ рд▓рдХреНрд╖рдг рдФрд░ рдХреЗрдВрджреНрд░реАрдп рддрдВрддреНрд░рд┐рдХрд╛ рддрдВрддреНрд░ рд╡рд┐рдХрд╛рд░ рд╡рд┐рдХрд╕рд┐рдд рд╣реЛрддреЗ рд╣реИрдВ, рдЬреЛ рдЯреАрдХреЗ рдХреА рдЬрдЯрд┐рд▓рддрд╛рдУрдВ рдХреЗ рдкрд░рд┐рдгрд╛рдорд╕реНрд╡рд░реВрдк рд╣реЛрддреЗ рд╣реИрдВред рддрд╛рд▓рд┐рдХрд╛ рд╡рд┐рд╢реНрд╡ рд╕реНрд╡рд╛рд╕реНрдереНрдп рд╕рдВрдЧрдарди рдХреЗ рдЕрдиреБрд╕рд╛рд░ рд╡рд┐рднрд┐рдиреНрди рдЯреАрдХрд╛рдХрд░рдгреЛрдВ рд╕реЗ рдЯреАрдХрд╛рдХрд░рдг рдХреЗ рдмрд╛рдж рдХреА рд╡рд┐рднрд┐рдиреНрди рдЧрдВрднреАрд░ рдЬрдЯрд┐рд▓рддрд╛рдУрдВ рдХреЛ рджрд░реНрд╢рд╛рддреА рд╣реИ:
рдЯреАрдХрд╛ рдЙрд▓рдЭрди рд╡рд┐рдХрд╛рд╕ рдЖрд╡реГрддреНрддрд┐
рдЬрдЯрд┐рд▓рддрд╛рдУрдВ
рдмреАрд╕реАрдЬреАрд▓рд╕реАрдХрд╛ рд╡рд╛рд╣рд┐рдХрд╛рдУрдВ рдХреА рд╕реВрдЬрди1000 рдореЗрдВ 1 - 10,000
рдУрд╕реНрдЯрд┐рдЕрдЯрд┐рд╕3,000 рдореЗрдВ 1 - 100,000,000
рд╕рд╛рдорд╛рдиреНрдпреАрдХреГрдд рдмреАрд╕реАрдЬреА рд╕рдВрдХреНрд░рдордг1,000,000 . рдореЗрдВ 1
рд╣реЗрдкреЗрдЯрд╛рдЗрдЯрд┐рд╕ рдмреАрддреАрд╡реНрд░рдЧрд╛рд╣рд┐рддрд╛ рд╕рдВрдмрдВрдзреА рд╕рджрдорд╛600,000 рдореЗрдВ 1 - 900,000
рдЦрд╕рд░рд╛ рдХрдгреНрдардорд╛рд▓рд╛ рд░реВрдмреЗрд▓рд╛рддрд╛рдкрдорд╛рди рдХреА рдкреГрд╖реНрдарднреВрдорд┐ рдкрд░ рдЖрдХреНрд╖реЗрдк3000 . рдореЗрдВ 1
рд░рдХреНрдд рдореЗрдВ рдкреНрд▓реЗрдЯрд▓реЗрдЯреНрд╕ рдХреА рд╕рдВрдЦреНрдпрд╛ рдореЗрдВ рдХрдореА30,000 рдореЗрдВ 1
рдЧрдВрднреАрд░ рдПрд▓рд░реНрдЬреА100,000 . рдореЗрдВ 1
рддреАрд╡реНрд░рдЧрд╛рд╣рд┐рддрд╛ рд╕рдВрдмрдВрдзреА рд╕рджрдорд╛1,000,000 . рдореЗрдВ 1
рдорд╕реНрддрд┐рд╖реНрдХ рд╡рд┐рдХреГрддрд┐1,000,000 рдореЗрдВ 1 рд╕реЗ рдХрдо
рдХреЗ рдЦрд┐рд▓рд╛рдл рдореМрдЦрд┐рдХ рдЯреАрдХрд╛
рдкреЛрд▓рд┐рдпреЛ (рдореБрдВрд╣ рдореЗрдВ рдмреВрдБрджреЗрдВ)
рд╡реИрдХреНрд╕реАрди рд╕реЗ рдЬреБрдбрд╝реЗ рдкреЛрд▓рд┐рдпреЛрдорд╛рдЗрд▓рд╛рдЗрдЯрд┐рд╕2,000,000 . рдореЗрдВ 1
рдзрдиреБрд╕реНрддрдВрднрдмреНрд░реЗрдХрд┐рдпрд▓ рддрдВрддреНрд░рд┐рдХрд╛ рдиреНрдпреВрд░рд┐рдЯрд┐рд╕100,000 . рдореЗрдВ 1
рддреАрд╡реНрд░рдЧрд╛рд╣рд┐рддрд╛ рд╕рдВрдмрдВрдзреА рд╕рджрдорд╛100,000 . рдореЗрдВ 1
рдбреАрдЯреАрдкреАрд▓рдВрдмреЗ рд╕рдордп рддрдХ рд▓рдЧрд╛рддрд╛рд░ рд░реЛрдирд╛1000 рдореЗрдВ 1
рдЖрдХреНрд╖реЗрдк1750 рдореЗрдВ 1 - 12500
рджрдмрд╛рд╡ рдореЗрдВ рдХрдореА, рдорд╛рдВрд╕рдкреЗрд╢рд┐рдпреЛрдВ рдХреА рдЯреЛрди, рдЪреЗрддрдирд╛ рдХреА рд╣рд╛рдирд┐1,000 рдореЗрдВ 1 - 33,000
рддреАрд╡реНрд░рдЧрд╛рд╣рд┐рддрд╛ рд╕рдВрдмрдВрдзреА рд╕рджрдорд╛50,000 рдореЗрдВ 1
рдорд╕реНрддрд┐рд╖реНрдХ рд╡рд┐рдХреГрддрд┐1,000,000 . рдореЗрдВ 1

рд╕рд╛рдорд╛рдиреНрдп рдЬрдЯрд┐рд▓рддрд╛рдУрдВ рдХреА рд╢реНрд░реЗрдгреА рдореЗрдВ рдЕрдВрддрд░ рдХреЗ рдХрд╛рд░рдг рд╣реЛрддрд╛ рд╣реИ рд╡рд┐рднрд┐рдиреНрди рджреЗрд╢... рдмрдбрд╝реА рд╕рдВрдЦреНрдпрд╛ рдореЗрдВ рдЬрдЯрд┐рд▓рддрд╛рдПрдВ рдЯреАрдХрд╛рдХрд░рдг рдирд┐рдпрдореЛрдВ рдХреА рдЙрдкреЗрдХреНрд╖рд╛, рдорддрднреЗрджреЛрдВ рдХреА рдЕрдирджреЗрдЦреА, рдЯреАрдХреЛрдВ рдХреЗ рдЕрдиреБрдЪрд┐рдд рднрдВрдбрд╛рд░рдг рдФрд░ рдкрд░рд┐рд╡рд╣рди, рджрд╡рд╛рдУрдВ рдХреЗ рдЦрд░рд╛рдм рдмреИрдЪреЛрдВ рдХреЗ рдЙрдкрдпреЛрдЧ рдФрд░ рдЕрдиреНрдп рд╕рдорд╛рди рдХрд╛рд░рдХреЛрдВ рдХреЗ рдХрд╛рд░рдг рд╣реЛрддреА рд╣реИрдВред
  • рд╕рд╛рдЗрдЯ рдХреЗ рдЕрдиреБрднрд╛рдЧ